You are on page 1of 215

Buyer: Jan Hiller (janhiller@gmx.

net)
Transaction ID: 8BE25485KM511652L

Humanities, Cultural Studies and


Social Sciences Module

TestAS, die Gesellschaft für Akademische Studienvorbereitung und Testentwicklung e.V., and ITB Consulting
GmbH do not endorse, nor are affiliated with this product.

© 2017 edulink GmbH www.edu-link.de ALL RIGHTS RESERVED 1


Buyer: Jan Hiller (janhiller@gmx.net)
Transaction ID: 8BE25485KM511652L

Humanities, Cultural Studies and


Social Sciences Module

FOREWORD

I am as passionate about education as I am about learning. Holding BA/BS degrees from the
Wharton School in the United States and an MBA from Insead in France and Singapore, I
have found that, over the course of my career, my education has opened many doors for
me. I wrote these test preparation books to support students with similar passions.

Over the years, with our company edulink, I have been privileged to help young men and
women navigate what can be a complicated university application process. It is a first step
toward the realization of their educational dreams, and a crticial one. I find it very rewarding
to be a part of this very important decision, helping students secure placement at German
universities that will meet their long-term goals.

Due to the highly competitive nature of admission to these schools, applicants must always
be looking for ways to differentiate their application. One of the most effective ways that they
can do this is by performing well on the TestAS exam, an aptitude test for applicants from
non-European countries who intend to pursue their studies at a German university. More
than ever before, German universities are using the results of this exam to determine a
student’s qualification for university-level courses.

We created this preparation book based on our own first-hand experience taking the exam
and detailed interviews with dozens of students who have taken the exam in Indonesia,
Vietnam, China, Turkey, Russia, and Ukraine. This ebook gives a comprehensive overview
of each of the tested topics, complete with test-taking tips and many practice questions,
each designed to help you take the TestAS with confidence.

I wish you the best of success on the exam and in your applications.

Ozveri Bauschmid

© 2017 edulink GmbH www.edu-link.de ALL RIGHTS RESERVED 2


Buyer: Jan Hiller (janhiller@gmx.net)
Transaction ID: 8BE25485KM511652L

Humanities, Cultural Studies and


Social Sciences Module

Özveri Bauschmid, Peter Bauschmid

All rights reserved

Edition 2 April 2017

Copyright © 2017, edulink GmbH


Munich, Germany

This work and all its parts are subject to copyright. Any unauthorized use outside the narrow
limits laid down by copyright law is unlawful and liable for prosecution. This applies in
particular to reproductions, translations, and microfilming, as well as to storing and
processing by electronic means.

© 2017 edulink GmbH www.edu-link.de ALL RIGHTS RESERVED 3


Buyer: Jan Hiller (janhiller@gmx.net)
Transaction ID: 8BE25485KM511652L

Humanities, Cultural Studies and


Social Sciences Module

CONTENTS
Foreword .…………………………………………………………………………………………..... 2
1 About the Humanities, Cultural Studies and Social Sciences Module ............................. 6
1.1 Introduction ............................................................................................................. 6
1.2 Working with the question booklet .......................................................................... 7
1.3 How should I fill out the answer sheet? .................................................................. 8
1.4 How to guess? ........................................................................................................ 9
1.5 Practical tips for the test day ................................................................................. 10
2 Understanding and Interpreting Texts ........................................................................... 11
2.1 Tips for success .................................................................................................... 14
2.2 Start with reading the question statements ........................................................... 15
2.3 Can the answer be frequently D "none of the above"? ......................................... 15
2.4 Practice questions ................................................................................................ 17
2.4.1 Exam 1 .................................................................................................... 17
2.4.2 Exam 2 .................................................................................................... 29
2.5 Answer key ........................................................................................................... 40
2.6 Detailed answers .................................................................................................. 41
2.6.1 Exam 1 .................................................................................................... 41
2.6.2 Exam 2 .................................................................................................... 58
3 Using Representation Systems Flexibly ........................................................................ 75
3.1 What am I being tested on? .................................................................................. 75
3.2 Things to keep in mind .......................................................................................... 76
3.3 Types of diagrams encountered in the exam ........................................................ 76
3.3.1 Dependencies.......................................................................................... 76
3.3.2 Chronological processes ......................................................................... 78
3.3.3 Positive and negative effects ................................................................... 82
3.4 Summary: Ten essential facts, tips and tricks – read this (again) before the exam
.............................................................................................................................. 86
3.5 Practice questions ................................................................................................ 87
3.5.1 Exam ....................................................................................................... 87
3.5.2 Answer key ............................................................................................ 107
3.5.3 Detailed answers ................................................................................... 108
4 Recognising Linguistic Structures ................................................................................ 139
4.1 General rules for this section .............................................................................. 142
4.1.1 There are no irregularities in the fictitious languages. ........................... 142
4.2 This question type can be mastered ................................................................... 145
4.3 Time management is essential ........................................................................... 145

© 2017 edulink GmbH www.edu-link.de ALL RIGHTS RESERVED 4


Buyer: Jan Hiller (janhiller@gmx.net)
Transaction ID: 8BE25485KM511652L

Humanities, Cultural Studies and


Social Sciences Module

4.4 Top-down approach ............................................................................................ 146


4.5 Our suggestion for solution approach ................................................................. 154
4.6 The process of elimination is your friend in this section...................................... 161
4.7 Practice questions .............................................................................................. 165
4.7.1 Exam 1 ................................................................................................. 165
4.7.2 Exam 2 ................................................................................................. 174
4.8 Answer key ......................................................................................................... 183
4.9 Detailed answers ................................................................................................ 184
4.9.1 Exam 1 ................................................................................................. 184
4.9.2 Exam 2 ................................................................................................. 200

© 2017 edulink GmbH www.edu-link.de ALL RIGHTS RESERVED 5


Buyer: Jan Hiller (janhiller@gmx.net)
Transaction ID: 8BE25485KM511652L

Humanities, Cultural Studies and


Social Sciences Module

1 ABOUT THE HUMANITIES, CULTURAL STUDIES AND


SOCIAL SCIENCES MODULE

1.1 INTRODUCTION

The TestAS exam consists of 2 parts:

○ a Core Test lasting 110 minutes and assessing skills necessary for successful
studies in all university-level subjects, and

○ a Subject-Specific Module pertaining to your chosen field of study and lasting 145-
150 minutes.

This ebook is focussed on the subject-specific module of Humanities, Cultural Studies and
Social Sciences Module.

Each university freely determines which subject module is relevant for a given major. In
many cases, applicants to the abovelisted faculties are asked to take this subject test. If you
are unsure which subject-specific module will be relevant for you, please consult directly with
the university that you are applying to. Usually this subject module is relevant for students
who have future plans to study in fields offered under the following faculties:

○ Faculty of Law (Rechtswissenschaftliche Fakultät)

○ Faculty of Arts and Humanities (Philosophische Fakultät)

○ Faculty of Human Sciences (Humanwissenschaftliche Fakultät)

Examples for the majors where this module is relevant are:

Languages / Linguistics Law

Politics Sociology

If you are uncertain about which specialised test module is right for you, contact the
university you wish to attend directly.

© 2017 edulink GmbH www.edu-link.de ALL RIGHTS RESERVED 6


Buyer: Jan Hiller (janhiller@gmx.net)
Transaction ID: 8BE25485KM511652L

Humanities, Cultural Studies and


Social Sciences Module

The humanities module is made up of three groups of questions:

○ Understanding and Interpreting Texts (22 questions; 45 minutes)

○ Using Representation Systems Flexibly (26 questions; 55 minutes)

○ Recognising Linguistic Structures (22 questions; 50 minutes).

In each subtest the questions are in increasing order of difficulty. The first few questions will
be easy – please quickly solve these and proceed to the next question. Many students report
being suspicious by the easiness of the first questions and unnecessarily doublechecking
the answers, leading to an unfortunate loss of time.

1.2 WORKING WITH THE QUESTION BOOKLET

At the beginning of the exam you will receive a booklet with the questions and a separate
one-page answer sheet where you need to mark your answers. You have 60 minutes for the
first subtest "Analysing scientific interrelationships". By the end of this time, you need to
have marked all your answers in the answer sheet. The answers marked in the question
booklet are not graded.

Even if you finish earlier, you need to wait until the end of the 60 minutes before you can
proceed to the 2nd subtest. And once the 60 minutes are over, you need to move to the next
subtest.

The following symbols are in the question booklet at the lower right side of the pages. These
show you where each subtest ends.

Please go on to the next page and


continue to work immediately

Do not turn the page!


Wait for the sign of the test conductor!
STOP

Important: If you see the stop sign, it means that you have reached the end of the subtest.
You can go back to the prior pages, but are not allowed to switch to the next section. Do not

© 2017 edulink GmbH www.edu-link.de ALL RIGHTS RESERVED 7


Buyer: Jan Hiller (janhiller@gmx.net)
Transaction ID: 8BE25485KM511652L

Humanities, Cultural Studies and


Social Sciences Module

turn the page here. This could be grounds to disqualify you from the entire test.

1.3 HOW SHOULD I FILL OUT THE ANSWER SHEET?

For the core test and the subject specific modules you will be given a question booklet along
with a separate one-page answer sheet. All answers need to be marked on the separate
answer sheet using a ball-point pen.

You need to use a blue or black pen to fill in the answers. Please do not use a pencil. Your
answer choice must be marked with an X. In the example below, B is marked.

A B C D

In case you need to change an answer that you already marked, fill out the entire wrong
circle and put an X in your new answer choice. In the example below, D is marked.

A B C D

In the unlikely event that you would like to make a further change, please fill out the wrong
circles and cross the correct answer. In the example below, C is marked.

A B C D

If you change your mind again and you would like to reenter an answer which you have
already crossed out, you need to fill out the wrong circles and manually write the correct
letter (A, B, C, D) at the end of the answer row. In the example below, B is marked.

© 2017 edulink GmbH www.edu-link.de ALL RIGHTS RESERVED 8


Buyer: Jan Hiller (janhiller@gmx.net)
Transaction ID: 8BE25485KM511652L

Humanities, Cultural Studies and


Social Sciences Module

A B C D

But given a computer does the grading, we urge you not to make too many corrections on
the answer sheet.

Please make sure that all of your answers are in the answer sheet before the time is up.

1.4 HOW TO GUESS?

We recommend the following approach to increase your likelihood of getting the skipped
questions right.

Step 1 Step 2 Step 3


• Answer first • Use the • Determine
the questions process of which letter
that you are elimination. you have been
familiar with. picked the
least and use
this as your
letter of the
day.

STEP 1: Answer first the questions that you are familiar with.

On the first pass, focus on these questions and skip the questions that seem to require more
thinking or the ones that you need to read two or three times before you understand them.
The questions increase in difficulty as you progress (this is true for all subtests of the
TestAS). But even the difficult ones are not difficult if you know the topic from your studies.

Circle the questions that you skipped for easy identification.

STEP 2: Use the process of elimination.

Once you have gone through all the questions, you will go back to the ones that you skipped
before. This two-round strategy may be especially suitable for you if you have not taken a

© 2017 edulink GmbH www.edu-link.de ALL RIGHTS RESERVED 9


Buyer: Jan Hiller (janhiller@gmx.net)
Transaction ID: 8BE25485KM511652L

Humanities, Cultural Studies and


Social Sciences Module

certain natural science course before (biology, chemistry, or physics). This will also boost
your self-confidence as you will see that – despite the skipped questions – you can solve
many questions correctly.

Use the process of elimination to rule out the answer choices that you know or believe are
wrong. Please mark these ruled-out choices clearly on the question booklet.

Next you need to guess from the remaining. (There is no penalty for guessing. A blank
answer and a wrong answer have the same impact on your score.) Step 3 will help you with
this.

STEP 3: Determine which letter you have been picked the least and use this as your
letter of the day.

Look which of the answer choice A to D you crossed the least on the answer sheet for that
subtest. This will then be your letter of the day. (In past years each of the 4 answer letters
were roughly equally selected.) Select the same answer choice each time you guess. This
will increase your likelihood of answering more questions correctly.

Skip step 2 in case you are running out of time so that at least you have marked all the
answers.

1.5 PRACTICAL TIPS FOR THE TEST DAY

The test can last more than 4 hours. Make sure to bring drinks and a snack to maintain
your energy during the test.

Also, make sure that you get enough sleep during the night before. This might sound like a
matter of course, but it must be understood by all that it is of great importance because you
need to concentrate for more than 4 hours!

© 2017 edulink GmbH www.edu-link.de ALL RIGHTS RESERVED 10


Buyer: Jan Hiller (janhiller@gmx.net)
Transaction ID: 8BE25485KM511652L

Humanities, Cultural Studies and


Social Sciences Module

2 UNDERSTANDING AND INTERPRETING TEXTS

In the task group "Understanding and Interpreting Texts", you need to understand the
content of texts and interpret it correctly to be able to answer 2-3 questions about what you
read.

There is only one rule to keep in mind: it only matters what is in the text. You may come
across texts about topics that you know a lot about. But in this section, you are being tested
on your ability to reason and think through each individual problem that you encounter, and
not on what you may already know about a subject. Do not allow your knowledge of a
subject to influence your search for answers. You must focus solely on what is written in
the text to arrive at the correct answer.

On the test, you will be given a series of brief texts to read, each of which will contain about
100-150 words. In these practice exercises, the texts are often slightly longer. This is
intended to make sure that you are really well prepared, so do not worry if you struggle a
little with the length or complexity at first. Accompanying each text on the test will be 2-4
comprehension exercises.

EXAMPLES 1-3

A MORAL DILEMMA

Two friends were walking down the street when they came upon a lost wallet. When they
opened it up, they found two hundred and fifty dollars in bank notes, a credit card, and a
driver’s licence - presumably that of the wallet’s owner. The first friend, Peter, looked
excited. "What a find!" he said. "Let’s use the money to go somewhere exotic." The second
friend, James, wasn’t so sure. "Shouldn’t we try to return it to its owner?" he asked, worried.
"After all, we have the licence and we know his name and address. It would be wrong to
keep it. Maybe the owner needs the money." Peter couldn’t believe what he was hearing.
"Don’t be such a bore!" he said, exasperated. "If the owner needed the money so much, he
would have been more careful not to lose it in the first place. Why don’t we just have some
fun? Nobody will ever know it’s us." James looked miserable. "This isn’t right at all," he
thought to himself.

© 2017 edulink GmbH www.edu-link.de ALL RIGHTS RESERVED 11


Buyer: Jan Hiller (janhiller@gmx.net)
Transaction ID: 8BE25485KM511652L

Humanities, Cultural Studies and


Social Sciences Module

EXAMPLE 1

Which of the following statements is/are correct according to the text?

I. The wallet contained more than two hundred pounds in cash.

II. Peter wanted to use the money for shopping.

(A) Only statement (I) is correct.

(B) Only statement (II) is correct.

(C) Both statements are correct.

(D) Neither of the two statements is correct.

Answer D

The first statement is designed to test your attention to detail – everything is correct except
the currency (the text mentions "two hundred and fifty dollars in bank notes"), which means
the statement overall is incorrect. Likewise, Statement II is also incorrect – the text clearly
states that the first friend wanted to use the money "to go somewhere exotic". Since both
statements are incorrect, Answer D is the right choice.

EXAMPLE 2

How did James probably feel after the conversation ended?

(A) He was scared of being found out.

(B) He was looking forward to spending the money.

(C) He was angry with his friend.

(D) He was disappointed in his friend’s reaction.

Answer D

To reach a conclusion about how James was feeling, we must look at the final two
sentences: "James looked miserable. 'This isn’t right at all,' he thought to himself." His
reaction makes clear that he wasn’t feeling positive about the situation, which rules out
option B. We now have to examine which of the "negative" options (A, C or D) is most

© 2017 edulink GmbH www.edu-link.de ALL RIGHTS RESERVED 12


Buyer: Jan Hiller (janhiller@gmx.net)
Transaction ID: 8BE25485KM511652L

Humanities, Cultural Studies and


Social Sciences Module

probable. There is nothing anywhere in the text to suggest that he was scared of being found
out – rather, his motivation for giving back the wallet was that he felt it was wrong to keep it
– so we can also rule out option A. It possible that he was angry with his friend (option C),
but the use of the word "miserable" to describe his demeanour makes it much more likely
that he was sad than angry. Therefore, option D is the most likely choice.

EXAMPLE 3

Which of the following statements is/are correct according to the text?

I. James has empathy with the owner of the wallet.

II. Peter was convinced by James’ argument.

(A) Only statement (I) is correct.

(B) Only statement (II) is correct.

(C) Both statements are correct.

(D) Neither of the two statements is correct.

Answer A

James points out that "he might need the money" and is trying to understand the situation of
the owner. Statement I is correct. Statement II, however, is incorrect. If we look at Peter’s
response to James’ argument that keeping the wallet is wrong, there is nothing to show that
he agrees with James. Peter’s words ("Don’t be such a bore!... If the owner needed the
money so much, he would have been more careful not to lose it in the first place") and his
tone ("exasperated") demonstrate clearly that he does not see any moral problem with
keeping the money. Hence, Statement II is incorrect and Answer A is the correct choice.

© 2017 edulink GmbH www.edu-link.de ALL RIGHTS RESERVED 13


Buyer: Jan Hiller (janhiller@gmx.net)
Transaction ID: 8BE25485KM511652L

Humanities, Cultural Studies and


Social Sciences Module

2.1 TIPS FOR SUCCESS

There are some important tips for this task group:

1.

• Time management. It is very important to keep the time in mind. You will
have to answer 22 questions within 45 minutes during the real test. That
means you will have 2 minutes (120 seconds) for every task. Remember
not to waste too much time on any one task. We believe that all tasks are
given the same score or weight - no matter the difficulty. The makers of the
test do not disclose this information, but equal weighting applies in
comparable tests created by the same company.

2.

• Read texts and tasks thoroughly. You can only make an educated
decision about your answer if you've read the text and the corresponding
answers thoroughly. In all standard tests, people tend to make mistakes
because they have not read the text or the task properly. These are
mistakes that can be avoided.

3.

• Choose an answer for all questions - there is no penalty for incorrect


answers. If you are running out of time, answer all the remaining answers
quickly - even if you have to guess. There is no penalty for wrong answers
and no points will be deducted. Pick one letter and stick to that.

4.

• Intelligent guessing: If you are only sure about one statement, e.g. that
statement (I) is correct, you can rule out B "Only (II) can be recognized" and
D "Neither I nor II can be recognized". Now you have a 50% chance of
answering the question correctly instead of only a 25% chance.

5.

• Don't panic: Should you notice that the day on which you are taking the
test is just not your day, remain calm and continue to work with
concentration. Just do your best. You might be doing better than you think
you are.

© 2017 edulink GmbH www.edu-link.de ALL RIGHTS RESERVED 14


Buyer: Jan Hiller (janhiller@gmx.net)
Transaction ID: 8BE25485KM511652L

Humanities, Cultural Studies and


Social Sciences Module

6.

• Your language skills has a direct impact on your final score. Practicing the
langauge is the best preparation. The range of content in the exam is quite
wide. Read as much as you can – newspapers, fiction and non-fiction – to
prepare yourself for the test. However, it is equally important to remember
that you won’t always need to know every word of vocabulary to
comprehend a text successfully. You will often be able to “read around” the
words you do not know to arrive at the correct answer.

2.2 START WITH READING THE QUESTION STATEMENTS

In this test, texts are very long. It is best to first read the question statement and then the
lengthy text, so you know what to look for in the paragraph. This can save you a lot of time.

2.3 CAN THE ANSWER BE FREQUENTLY D "NONE OF THE ABOVE"?

If you tend to choose D (none of the statements are correct) too frequently, consider
skimming the paragraph one more time. D is chosen roughly on every fourth question.

EXAMPLE 4

Geese are very loyal to their families: They choose one partner for life and are very
protective of that partner and their children. If the partner of a goose or if the children
become ill or injured, the goose will often refuse to leave their side even if winter is coming
and the other geese are flying south as a flock. Those of us who have taken the time to
observe geese, say that they feel emotion just as we do: If the partner of a goose is killed or
eggs become damaged, then the goose will retreat from the remaining group of geese while
it mourns. If the partner dies, some geese will remain a widow for the rest of their lives and
will refuse to take on a new partner. And this can be for quite some time since geese can live
for up to 25 years.

Geese love to clean their feathers, dig through grass and branches for food and to collect
bark and leaves in order to make their nests comfy cosy. They lay spring eggs once a year
and the female will sit on them for 30 days while her partner protects their well hidden home.

© 2017 edulink GmbH www.edu-link.de ALL RIGHTS RESERVED 15


Buyer: Jan Hiller (janhiller@gmx.net)
Transaction ID: 8BE25485KM511652L

Humanities, Cultural Studies and


Social Sciences Module

And, corresponding to their loyal nature, geese love using the same nest every year when
possible. (Source: Peta.de)

Which of the following two statements is or are correct?

I. Geese care for their families quite intensively.

II. If the partner of a goose dies, the widow goose will never take on a new partner.

(A) Only statement (I) is correct.

(B) Only statement (II) is correct.

(C) Both statements are correct.

(D) Neither of the two statements is correct.

Answer A

It can be derived from the text that geese care quite intensively for their families.

However, it is only stated that "some" geese do not take on a new partner when their partner
dies, but not all.

© 2017 edulink GmbH www.edu-link.de ALL RIGHTS RESERVED 16


Buyer: Jan Hiller (janhiller@gmx.net)
Transaction ID: 8BE25485KM511652L

Humanities, Cultural Studies and


Social Sciences Module

2.4 PRACTICE QUESTIONS

You have a total of 45 minutes during the TestAS exam for the 22 questions belonging to
this section.

2.4.1 EXAM 1

"THE LIFE OF MORITZ" (EXERCISES 1.1-1.4)

Five years ago, Moritz, now a 17-year-old student, developed a passion for tennis and
began playing as often as he could. All went smoothly until three months ago, when he got
to know a charming, bright and very pretty young lady named Stephanie. Moritz now had to
find a way to divide his time between Stephanie and all of his hobbies. Furthermore,
Stephanie wanted to become an engineer. It was her greatest dream to help people by
making her innovative ideas a reality. June had already arrived. In July, Stephanie wanted to
gain a deeper insight into the engineering world by completing a work experience placement
at the Volkswagen car company. This would make Moritz’s situation even more precarious.

1.1.

Which of the following statements is/are correct according to the text?

I. Stephanie is definitely going to become an engineer.

II. Moritz plays tennis very well.

(A) Only statement (I) is correct.

(B) Only statement (II) is correct.

(C) Both statements are correct.

(D) Neither of the two statements is correct.

1.2.

Which of the following statements is/are correct according to the text?

I. Stephanie’s interests must take a back seat while she is in a relationship with Moritz.

© 2017 edulink GmbH www.edu-link.de ALL RIGHTS RESERVED 17


Buyer: Jan Hiller (janhiller@gmx.net)
Transaction ID: 8BE25485KM511652L

Humanities, Cultural Studies and


Social Sciences Module

II. Moritz is trying to make his relationship with Stephanie compatible with other parts of
his life.

(A) Only statement (I) is correct.

(B) Only statement (II) is correct.

(C) Both statements are correct.

(D) Neither of the two statements is correct.

1.3.

Which of the following statements is/are correct according to the text?

I. Stephanie does not care at all about her career.

II. Stephanie has been accepted to do a work experience placement at Volkswagen.

(A) Only statement (I) is correct.

(B) Only statement (II) is correct.

(C) Both statements are correct.

(D) Neither of the two statements is correct.

1.4.

How was Moritz likely to be feeling about Stephanie’s plans?

(A) He will break up with her.

(B) He was worried.

(C) He didn’t care.

(D) He was happy.

"THE LITTLE TIGER" (EXERCISES 1.5-1.8)

There was once a little tiger whose mother died soon after he was born. A herd of sheep
living nearby took charge of him and accepted him as one of their own.

© 2017 edulink GmbH www.edu-link.de ALL RIGHTS RESERVED 18


Buyer: Jan Hiller (janhiller@gmx.net)
Transaction ID: 8BE25485KM511652L

Humanities, Cultural Studies and


Social Sciences Module

The little tiger soon turned into a big, noble creature. But he continued to behave like a
sheep. He ate grass, bleated like a sheep and sought protection from the herd. Although the
power of a tiger slumbered deep within him, he held fast to the belief that he was a sheep.

One day, an old tiger crept towards the herd with the aim of seizing one of the sheep. When
he saw the young tiger grazing amidst the sheep, he could hardly believe his eyes. He
chased after him, grabbed him by the fur on his neck and dragged him to a watering hole.
The young tiger smelled like a sheep and struggled anxiously against the insistence of the
older tiger that he was, in fact, a big cat – not a sheep as he believed.

The old tiger was insistent that the young tiger should look at his reflection in the water.
When he did so, he realised that he really wasn’t a sheep, but a tiger!

At that moment, a powerful roar erupted from deep inside of him, and his true nature
became clear to him at last.

(translation of a slightly amended excerpt from the German story "Sei dein bester Freund" by
Ulrike Dahm and Erich Keller; http://www.engelbrecht-media.de/s_der_kleine_tiger.html)

1.5.

We can assume that from now on, the tiger will behave like a...

(A) Tiger

(B) Sheep

(C) Deer

(D) Beaver

1.6.

Which of the following statements is/are correct according to the text?

I. It is impossible to hide one’s true nature from oneself forever.

II. One can carry a certain nature within oneself, but behave in a way that does not
reflect it.

(A) Only statement (I) is correct.

(B) Only statement (II) is correct.

© 2017 edulink GmbH www.edu-link.de ALL RIGHTS RESERVED 19


Buyer: Jan Hiller (janhiller@gmx.net)
Transaction ID: 8BE25485KM511652L

Humanities, Cultural Studies and


Social Sciences Module

(C) Both statements are correct.

(D) Neither of the two statements is correct.

1.7.

Which of the following statements is/are correct according to the text?

I. The "little tiger" ate grass, bleated like a sheep and ate meat.

II. The "little tiger" had lost the power he once had.

(A) Only statement (I) is correct.

(B) Only statement (II) is correct.

(C) Both statements are correct.

(D) Neither of the two statements is correct.

1.8.

Imagine that the old tiger had not spotted the young tiger and had instead carried out his
plan of attacking one of the sheep. Based on what you read in the passage, which of these
outcomes would have been most likely?

I. The young tiger would have instinctively recognised himself as the strongest member
of the herd and jumped to the sheep’s defence.

II. The young tiger would have felt scared and tried to hide like the rest of the herd.

(A) Only statement (I) is correct.

(B) Only statement (II) is correct.

(C) Both statements are correct.

(D) Neither of the two statements is correct.

© 2017 edulink GmbH www.edu-link.de ALL RIGHTS RESERVED 20


Buyer: Jan Hiller (janhiller@gmx.net)
Transaction ID: 8BE25485KM511652L

Humanities, Cultural Studies and


Social Sciences Module

"PROTEIN – THE SOURCE OF LIFE" (EXERCISES 1.9-1.11)

Virtually all processes occurring in living things are dependent on protein (Greek: proteios =
"of prime importance"). Therefore, they are of great importance for every single living
creature in this world. Proteins are biological macromolecules consisting of one or more
polypeptide chains, each of which is made from up to 22 proteinogenic amino acids. Eight of
these amino acids are what is known as "essential amino acids", which cannot be
synthesised internally and must therefore be taken in by living organisms in their food. The
amino acids are connected to each other with peptide bonds (-CONH-). The specific role of
each protein within an organism is determined by its spatial structure, which can be
described at four distinct structural levels (primary, secondary, tertiary and quaternary).

As enzymes (biocatalysts), proteins can be responsible for speeding up certain chemical


reactions or for enabling them to happen at all. For example, digestive enzymes catalyse the
hydrolysis of polymers. When behaving as structural proteins, proteins provide support and
give structure; for example, collagen and elastin form a fibrous mesh in the connective
tissues of animals. As hormones, proteins coordinate the various processes happening
within a living organism. For example, insulin – a hormone produced by the pancreas –
regulates the blood sugar concentration of vertebrates.

1.9.

Which of the following statements is/are correct according to the text?

I. Humans require protein to stay alive.

II. Elastin speeds up chemical reactions.

(A) Only statement (I) is correct.

(B) Only statement (II) is correct.

(C) Both statements are correct.

(D) Neither of the two statements is correct.

1.10.

Which of the following statements is/are correct according to the text?

I. The spatial structure of a protein determines its effects.

II. Insulin regulates the blood sugar concentration of slugs.

© 2017 edulink GmbH www.edu-link.de ALL RIGHTS RESERVED 21


Buyer: Jan Hiller (janhiller@gmx.net)
Transaction ID: 8BE25485KM511652L

Humanities, Cultural Studies and


Social Sciences Module

(A) Only statement (I) is correct.

(B) Only statement (II) is correct.

(C) Both statements are correct.

(D) Neither of the two statements is correct.

1.11.

Which of the following statements is/are correct according to the text?

I. "Proteins" are also referred to as "biocatalysts".

II. There are 22 essential amino acids.

(A) Only statement (I) is correct.

(B) Only statement (II) is correct.

(C) Both statements are correct.

(D) Neither of the two statements is correct.

"EXCERPT FROM A CONSTITUTIONAL COMPLAINT" (EXERCISES 1.12-1.14)

Pursuant to Article 93 (1) No. 4a GG (Basic Law for the Federal Republic of Germany) in
conjunction with §§13 No. 8a, 90 ff. BVerfG (Law on the Federal Constitutional Court of
Germany), the Federal Constitutional Court is responsible for the constitutional complaint by
the complainant. A complaint is deemed to be lawful if it is submitted in the correct form and
in a timely manner. Pursuant to §23 (1) 1 BVerfGG, a complaint must be submitted to the
Federal Constitutional Court in writing in order to satisfy the formal requirements. Any type of
written form is deemed to be admissible. Telex and telefax are objectionable. While, as a
basic principle, emails are not admissible as evidence, the literature indicates that emails are
sometimes admitted or acknowledged to be admissible.

In addition, according to § 92 BVerfGG, the constitutional complaint must be justified through


the naming of the basic right that has been breached.

Furthermore, the time limits set forth in §93 BVerfGG must be adhered to. In the case of a
constitutional complaint regarding acts of the administration (e.g. an administrative act) and
the judicial system (e.g. an incriminating judgement), the one-month time limit set forth in

© 2017 edulink GmbH www.edu-link.de ALL RIGHTS RESERVED 22


Buyer: Jan Hiller (janhiller@gmx.net)
Transaction ID: 8BE25485KM511652L

Humanities, Cultural Studies and


Social Sciences Module

§93 (1) 1 BverfGG shall apply; for a constitutional complaint against a law, a time limit of one
year from the date the law comes into force shall apply in accordance with § 93 (3) BverfGG.

1.12.

Which of the following statements is/are correct according to the text?

I. A complaint is deemed to be lawful if it is submitted in the correct form and in a timely


manner.

II. It is compulsory to submit the constitutional complaint in writing.

(A) Only statement (I) is correct.

(B) Only statement (II) is correct.

(C) Both statements are correct.

(D) Neither of the two statements is correct.

1.13.

Which of the following statements is/are correct according to the text?

I. § 93 BverfGG describes some of the time limits for a constitutional complaint.

II. The one-month time limit applies for judgements of the Court.

(A) Only statement (I) is correct.

(B) Only statement (II) is correct.

(C) Both statements are correct.

(D) Neither of the two statements is correct.

1.14.

Which of the following statements is/are correct according to the text?

I. Constitutional complaints can be sent via telex.

II. Time limits can be extended if there is good cause.

© 2017 edulink GmbH www.edu-link.de ALL RIGHTS RESERVED 23


Buyer: Jan Hiller (janhiller@gmx.net)
Transaction ID: 8BE25485KM511652L

Humanities, Cultural Studies and


Social Sciences Module

(A) Only statement (I) is correct.

(B) Only statement (II) is correct.

(C) Both statements are correct.

(D) Neither of the two statements is correct.

"THE FEDERAL PRESIDENT OF GERMANY" (EXERCISES 1.15-1.18)

"I swear that I will dedicate my efforts to the well-being of the German people, promote their
welfare, protect them from harm, uphold and defend the Basic Law and the laws of the
Federation, perform my duties conscientiously, and do justice to all.” – The oath of office of
the Federal President of Germany

By taking this oath, the Federal President of Germany indicates - in accordance with Article
56 of the Basic Law - that in taking up office, he or she commits themselves to high goals on
behalf of the German people and assumes a great responsibility towards them. Based on
this part of the Basic Law and on the Federal President’s manifestation as a permanent
constitutional body and designation as head of state, one could assume that the Federal
President enjoys a strong, influential position in the political process. However, although the
Federal Presidents of Germany do function as head of state and occupy what was formally
the highest position in the German political system, their national-political power is very
limited by the constitution, and they perform mainly representative duties. For this reason,
they are known as the highest representative of the Federal Republic of Germany. In return
for undertaking these duties, the Federal President receives 10/9 of the official salary of the
Federal Chancellor, which corresponds to an amount of 199,000€. After leaving office, the
Federal President is paid an Ehrensold (presidential pension) in the amount of their salary
every year for the rest of their life. This salary and pension are a lot higher than those
received by the average German person. In light of the above facts, many Germans are
questioning whether the Office of the Federal President still fulfills a necessary function for
the Federal Republic of Germany.

1.15.

Which of the following statements is/are correct according to the text?

I. The Federal President swears to serve the well-being of the EU.

II. The Federal Presidents takes their oath of office in accordance with Article 46 of the
Basic Law.

© 2017 edulink GmbH www.edu-link.de ALL RIGHTS RESERVED 24


Buyer: Jan Hiller (janhiller@gmx.net)
Transaction ID: 8BE25485KM511652L

Humanities, Cultural Studies and


Social Sciences Module

(A) Only statement (I) is correct.

(B) Only statement (II) is correct.

(C) Both statements are correct.

(D) Neither of the two statements is correct.

1.16.

Which of the following statements is/are correct according to the text?

I. The Federal President earns more than the Federal Chancellor.

II. The Federal President undertakes duties of a primarily representative nature.

(A) Only statement (I) is correct.

(B) Only statement (II) is correct.

(C) Both statements are correct.

(D) Neither of the two statements is correct.

1.17.

Which of the following statements is/are correct according to the text?

I. The Federal Presidents are not limited in their powers.

II. The Federal President is formally ranked above the Federal Chancellor in the Federal
Republic of Germany.

(A) Only statement (I) is correct.

(B) Only statement (II) is correct.

(C) Both statements are correct.

(D) Neither of the two statements is correct.

© 2017 edulink GmbH www.edu-link.de ALL RIGHTS RESERVED 25


Buyer: Jan Hiller (janhiller@gmx.net)
Transaction ID: 8BE25485KM511652L

Humanities, Cultural Studies and


Social Sciences Module

1.18.

Based on the text, how do you think the average German person would be likely to react if
the Federal President’s salary was increased?

(A) They would not care.

(B) They would be annoyed.

(C) They would be approving.

(D) They would be sad.

"THE FUTURE OF EUROPE" (EXERCISES 1.19.-1.22.)

In fifty years’ time, Europe and the USA will make up only seven per cent of the world’s
population. This will represent the most fundamental change in global population distribution
that humanity has ever seen. Europe must accept that it is a community of values and that
the USA, as the world’s oldest democracy, is an essential partner - regardless of the socio-
political differences that came to light during the NSA affair. People in Europe have been
fighting for their freedom for the last 500 years; milestones like the French Revolution, the
German Revolution of 1848, the Weimar Republic and the fall of the Berlin Wall in 1989 are
just a few of the events that have paved the way for the current system of democracy.
Europe has not only the choice but also the moral duty to preserve these values for the
future and to assert themselves in disputes. The unified stance of Europe in the world is
therefore not an option but an obligation.

1.19.

Which of the following statements is/are correct according to the text?

I. Over the next 50 years, 93 per cent of the world’s population will come to be located
outside of the USA and Europe.

II. Population distribution has never before altered so drastically.

(A) Only statement (I) is correct.

(B) Only statement (II) is correct.

(C) Both statements are correct.

(D) Neither of the two statements is correct.

© 2017 edulink GmbH www.edu-link.de ALL RIGHTS RESERVED 26


Buyer: Jan Hiller (janhiller@gmx.net)
Transaction ID: 8BE25485KM511652L

Humanities, Cultural Studies and


Social Sciences Module

1.20.

Which of the following statements is/are correct according to the text?

I. The German Revolution of 1848 and the fall of the Berlin Wall contributed to
establishing the current system of democracy.

II. In the future, Europe will be able to decide freely whether it wants to operate as a
community.

(A) Only statement (I) is correct.

(B) Only statement (II) is correct.

(C) Both statements are correct.

(D) Neither of the two statements is correct.

1.21.

Which of the following statements is/are correct according to the text?

I. The NSA affair was not beneficial for the relationship between the USA and
Germany.

II. Europe must further improve its relationship with the USA.

(A) Only statement (I) is correct.

(B) Only statement (II) is correct.

(C) Both statements are correct.

(D) Neither of the two statements is correct.

1.22.

The text states that Europe has a duty to preserve democratic values. Based on the text,
what would be the likely effect on Europe’s efforts to preserve democratic values if it did not
cooperate with America?

(A) It would make no difference.

(B) Europe’s efforts would be slightly less effective.

© 2017 edulink GmbH www.edu-link.de ALL RIGHTS RESERVED 27


Buyer: Jan Hiller (janhiller@gmx.net)
Transaction ID: 8BE25485KM511652L

Humanities, Cultural Studies and


Social Sciences Module

(C) Europe’s efforts would be much less effective.

(D) Europe’s efforts would be more effective.

© 2017 edulink GmbH www.edu-link.de ALL RIGHTS RESERVED 28


Buyer: Jan Hiller (janhiller@gmx.net)
Transaction ID: 8BE25485KM511652L

Humanities, Cultural Studies and


Social Sciences Module

2.4.2 EXAM 2

"THE ORIGINS OF THE OLYMPICS" (EXERCISES 2.1.-2.3.)

The Olympic Games originated in Olympia, Greece, with most sources agreeing that the first
official games took place in 776 BCE. This figure is based on inscriptions in Olympia that list
the winners of a footrace held in 776 BCE and every four years afterwards; however, many
scholars believe that the games had actually been happening in some form for many years
before that. According to tradition, the first champion of the games was a cook named
Coroebus, who took the title by winning the sole event at the time – a 192-metre (210-yard)
run. Held at the sanctuary of Zeus in Olympia, the games continued to grow for the next
1200 years and eventually came to feature running events, a pentathlon, boxing, wrestling,
pankration, and equestrian events. As well as being a sporting event, the Ancient Olympics
also featured many ceremonies and ritual sacrifices to honour the god Zeus. In 393 CE, a
Roman emperor named Theodosius I, who was a Christian, abolished the games due to
their connection with pagan beliefs. The Olympic tradition was revived in the 1800s, and the
first modern games were held in 1896.

2.1.

Which of the following statements is/are correct according to the text?

I. Most experts believe that the idea for the Olympic games was thought up in 776 BC.

II. Running was the only sport in the first official Olympic games.

(A) Only statement (I) is correct.

(B) Only statement (II) is correct.

(C) Both statements are correct.

(D) Neither of the two statements is correct.

2.2.

Which of the following statements is/are correct according to the text?

I. The Ancient Olympics was both a sporting and a religious event.

II. Like the modern Olympics, the ancient Olympics featured athletes from all over the
world.

© 2017 edulink GmbH www.edu-link.de ALL RIGHTS RESERVED 29


Buyer: Jan Hiller (janhiller@gmx.net)
Transaction ID: 8BE25485KM511652L

Humanities, Cultural Studies and


Social Sciences Module

(A) Only statement (I) is correct.

(B) Only statement (II) is correct.

(C) Both statements are correct.

(D) Neither of the two statements is correct.

2.3.

Which of the following statements is/are correct according to the text?

I. The emperor Theodosius I got rid of the games because they clashed with his
religious beliefs.

II. It was more than a thousand years before the games were brought back to life.

(A) Only statement (I) is correct.

(B) Only statement (II) is correct.

(C) Both statements are correct.

(D) Neither of the two statements is correct.

"THE FREIBURG THESES OF LIBERALISM" (EXERCISES 2.4.-2.6.)

The rationale behind liberal thinking is illustrated clearly in the following (originally German)
quote: "Liberalism leaves people in peace, without abandoning them completely." The four
Freiburg Theses were adopted on 27 October 1971 at the Federal Party Convention of the
FDP (Free Democratic Party, a classical liberal party in Germany) after the FDP received
only 5.8% of the vote at the 1969 Bundestag elections (German parliamentary elections).

The first thesis states that people should be given precedence over institutions and that the
preservation and development of the individual and the concept of plurality should be
promoted within human societies. The second thesis says that liberalism stands for progress
and reason. According to the third thesis, we are all part of society, but "society must not be
permitted to be everything". The fourth thesis supports a reform of the capitalistic system to
abolish trends for the accumulation of wealth in only a few hands. At its core, a liberal
society should serve people, promote an increase in output and ensure the protection of the
needy and the environment.

© 2017 edulink GmbH www.edu-link.de ALL RIGHTS RESERVED 30


Buyer: Jan Hiller (janhiller@gmx.net)
Transaction ID: 8BE25485KM511652L

Humanities, Cultural Studies and


Social Sciences Module

2.4.

Which of the following statements is/are correct according to the text?

I. At the 1969 Bundestag elections, the FDP received only 5.8% of the vote.

II. One of the functions of a liberal society is to protect the needy.

(A) Only statement (I) is correct.

(B) Only statement (II) is correct.

(C) Both statements are correct.

(D) Neither of the two statements is correct.

2.5.

Which of the following statements is/are correct according to the text?

I. According to the third thesis, the German federal chancellor Angela Merkel is a part
of society.

II. The result of the 1969 Bundestag election was the worst in the FDP’s history.

(A) Only statement (I) is correct.

(B) Only statement (II) is correct.

(C) Both statements are correct.

(D) Neither of the two statements is correct.

2.6.

Which of the following statements is/are correct according to the text?

I. According to the second thesis, the FDP is an advocate of science.

II. Environmental protection is considered to be of subordinate importance within a


liberal society.

(A) Only statement (I) is correct.

(B) Only statement (II) is correct.

© 2017 edulink GmbH www.edu-link.de ALL RIGHTS RESERVED 31


Buyer: Jan Hiller (janhiller@gmx.net)
Transaction ID: 8BE25485KM511652L

Humanities, Cultural Studies and


Social Sciences Module

(C) Both statements are correct.

(D) Neither of the two statements is correct.

"ANIMAL SPECIES FACING EXTINCTION" (EXERCISES 2.7.-2.9.)

The impact of human activity on the planet is putting an increasing number of animal species
at risk of extinction. Though extinction is a natural process that has occurred throughout the
earth’s history, studies show that man-made factors such as climate change, hunting and
poaching, fishing practices, pollution and loss of habitat are accelerating the process and
creating unprecedented numbers of endangered species. Some of the most at-risk species
include tigers, polar bears, penguins and various kinds of fish. Many insects are also
threatened – a recent assessment of European wild bee species found that 9.2% were in
danger becoming extinct. Of all the ways that humans are changing the planet, temperature
increases due to global warming are thought to be the most dangerous. If the planet warms
by an average of 3 degrees C, scientists predict that 8.5% of the world’s species will be in
danger of ceasing to exist.

2.7.

Which of the following statements is/are correct according to the text?

I. Extinction is a man-made problem.

II. A reduction in the size of natural habitats is one of the factors putting animals at
increased risk.

(A) Only statement (I) is correct.

(B) Only statement (II) is correct.

(C) Both statements are correct.

(D) Neither of the two statements is correct.

2.8.

Which of the following statements is/are correct according to the text?

I. Just over 9% of all wild bee species worldwide are thought to be at risk of extinction.

© 2017 edulink GmbH www.edu-link.de ALL RIGHTS RESERVED 32


Buyer: Jan Hiller (janhiller@gmx.net)
Transaction ID: 8BE25485KM511652L

Humanities, Cultural Studies and


Social Sciences Module

II. Bad fishing practices are responsible for the fact that many kinds of fish are now
endangered.

(A) Only statement (I) is correct.

(B) Only statement (II) is correct.

(C) Both statements are correct.

(D) Neither of the two statements is correct.

2.9.

Which of the following statements is/are correct according to the text?

I. Temperature rises due to global warming are generally considered to be a more


significant threat to animals than hunting and poaching.

II. If the earth’s temperature rises by an average of 3 degrees C, the number of animal
species will be reduced by 8.5%.

(A) Only statement (I) is correct.

(B) Only statement (II) is correct.

(C) Both statements are correct.

(D) Neither of the two statements is correct.

"THE LION AND THE MOUSE" (EXERCISES 2.10.-2.12.)

Once upon a time, a lion was sleeping deeply and peacefully in the jungle. As he slept, a
small mouse passed by. Without realising what he was doing, the mouse ran over the lion,
tickled him and woke him up. Immediately, the lion trapped the mouse under his big paw and
opened his mouth to swallow him whole. "Pardon me, Mr Lion!" squeaked the mouse.
"Please forgive me. I’ll make sure never to do such a thing again, and I’ll never forget your
kindness. Perhaps one day I’ll be able to repay you." The lion smiled to himself. He didn’t
see how such a small creature could ever be of any use to a lion, but he was so amused by
the mouse’s request that he let him go. It just so happened that some hours later, some
hunters passed by, captured the lion and tied him to a tree while they looked for a wagon to
carry him home. The mouse passed by once again, saw the sad plight of the lion, and set
about chewing through the ropes. Soon, the lion was free and the mouse was beaming with

© 2017 edulink GmbH www.edu-link.de ALL RIGHTS RESERVED 33


Buyer: Jan Hiller (janhiller@gmx.net)
Transaction ID: 8BE25485KM511652L

Humanities, Cultural Studies and


Social Sciences Module

pride. "Did I not tell you I’d pay you back someday?" he asked the lion. "Yes," agreed the
lion, smiling widely, "you were right!"

2.10.

Which of the following statements is/are correct according to the text?

I. The mouse didn’t mean to annoy the lion.

II. The lion let the mouse go because he was expecting to be repaid.

(A) Only statement (I) is correct.

(B) Only statement (II) is correct.

(C) Both statements are correct.

(D) Neither of the two statements is correct.

2.11.

Which of the following statements is/are correct according to the text?

I. The lion would definitely have died if the mouse hadn’t saved him.

II. The lion will never think of eating the mouse again.

(A) Only statement (I) is correct.

(B) Only statement (II) is correct.

(C) Both statements are correct.

(D) Neither of the two statements is correct.

2.12.

How did the lion probably feel after being rescued by the mouse?

(A) He was humiliated.

(B) He was shocked.

© 2017 edulink GmbH www.edu-link.de ALL RIGHTS RESERVED 34


Buyer: Jan Hiller (janhiller@gmx.net)
Transaction ID: 8BE25485KM511652L

Humanities, Cultural Studies and


Social Sciences Module

(C) He was grateful.

(D) He was confused.

"THE STUDY" (EXERCISES 2.13.-2.15.)

The (originally German-language) article "Student Writing Guidance is Professional and


Personalised" discusses a research project from the Writing Centre at the European
University Viadrina in Frankfurt (Oder), Germany. Within the student writing guidance
scheme, students seeking guidance were offered the help of other students. The goal of the
scheme was to bring about an increase in writing competence. Various concepts were
deployed in an attempt to achieve this, including "help with self-help", the exchange of
experiences, emotions and knowledge, and active engagement with individual writing
projects. The study was compiled from qualitative content analyses and problem-focused
interviews. The goal of qualitative content analysis is the systematic processing of
communication content that must adhere to a certain form. It does not reflect the content but
is instead aimed at representing formal aspects and latent meaning. A category system was
developed on the basis of the qualitative content analysis and used to measure how the
student writing guidance was received by the guidance-seeking students. The guidance-
seekers were asked how they entered and exited the guidance session, how they perceived
the session, what occurred to them, how they felt during and after the session and whether
the session triggered anything new in relation to them as individuals or their writing.

2.13.

Which of the following statements is/are correct according to the text?

I. The student writing guidance scheme requires students to help other students.

II. The student writing guidance scheme was intended to bring about an increase in
writing competence.

(A) Only statement (I) is correct.

(B) Only statement (II) is correct.

(C) Both statements are correct.

(D) Neither of the two statements is correct.

© 2017 edulink GmbH www.edu-link.de ALL RIGHTS RESERVED 35


Buyer: Jan Hiller (janhiller@gmx.net)
Transaction ID: 8BE25485KM511652L

Humanities, Cultural Studies and


Social Sciences Module

2.14.

Which of the following statements is/are correct according to the text?

I. The study was predominantly compiled from problem-focused interviews.

II. Qualitative content analysis arose out of the category system.

(A) Only statement (I) is correct.

(B) Only statement (II) is correct.

(C) Both statements are correct.

(D) Neither of the two statements is correct.

2.15.

Which of the following statements is/are correct according to the text?

I. The purpose of qualitative content analysis is the graphical processing of


communication content.

I. Qualitative content analysis goes beyond merely compiling information about the
actual content.

(A) Only statement (I) is correct.

(B) Only statement (II) is correct.

(C) Both statements are correct.

(D) Neither of the two statements is correct.

"THE PUPIL" (EXERCISES 2.16.-2.19.)

At the beginning of an assignment, David always reads the instructions carefully and then
skims through the text to get an overview before starting the first question. When he comes
across a problem he cannot solve, he simply skips over the problem and goes to the next
question. When he finishes the work, he looks at the problem once more and, if necessary,
he asks the teacher for help. When learning new things, he relies primarily on memory and
observation – I can tell that he is better able to memorise information when he sees things.
When it comes to German, David is quiet during classes and follows the lesson – quite in
contrast to his behaviour during natural history. Though he also follows the lesson here, he

© 2017 edulink GmbH www.edu-link.de ALL RIGHTS RESERVED 36


Buyer: Jan Hiller (janhiller@gmx.net)
Transaction ID: 8BE25485KM511652L

Humanities, Cultural Studies and


Social Sciences Module

talks much more with his fellow pupils and shares jokes, most of which come from him. The
subject of the "central nervous system" is one that particularly appeals to him. David finds it
easy to draw logical conclusions, which helps him to excel in fact-based subjects like maths
and science. However, he finds it difficult to utilise his creativity.

2.16.

Based on the teacher’s assessment of David’s strengths and weaknesses, which of these
four subjects is he likely to be strongest in?

(A) English

(B) Art

(C) Chemistry

(D) Music

2.17.

Which of the following statements is/are correct according to the text?

I. David finds it easy to make use of his musical talent.

II. If David is stuck, he asks the teacher for help.

(A) Only statement (I) is correct.

(B) Only statement (II) is correct.

(C) Both statements are correct.

(D) Neither of the two statements is correct.

2.18.

Which of the following statements is/are correct according to the text?

I. David is always quiet during German classes.

II. David knows exactly what the central nervous system is.

© 2017 edulink GmbH www.edu-link.de ALL RIGHTS RESERVED 37


Buyer: Jan Hiller (janhiller@gmx.net)
Transaction ID: 8BE25485KM511652L

Humanities, Cultural Studies and


Social Sciences Module

(A) Only statement (I) is correct.

(B) Only statement (II) is correct.

(C) Both statements are correct.

(D) Neither of the two statements is correct.

2.19.

Which of the following statements is/are correct according to the text?

I. David usually skims a text before reading the instructions that go with it.

II. David prefers to see and hear things in order to remember them.

(A) Only statement (I) is correct.

(B) Only statement (II) is correct.

(C) Both statements are correct.

(D) Neither of the two statements is correct.

"MORALS IN BUSINESS" (EXERCISES 2.20.-2.22.)

Time and again, we hear of the illegal and immoral acts committed by individuals within
companies and between companies themselves. This raises questions regarding the
motives for such acts. We must question whether the market economy promotes moral
values, whether it creates incentives for immoral behaviour, whether it drives immoral
behaviour in favour of profit maximisation and to the detriment of ethics, whether it is
possible to curb this behaviour in companies using internally prescribed moral codes and
whether companies have an interest in adhering to their own moral codes or will always
decide in favour of profit when faced with a moral dilemma.

2.20.

Which of the following statements is/are correct according to the text?

I. Banks are denounced as being especially immoral.

II. The market economy promotes moral values.

© 2017 edulink GmbH www.edu-link.de ALL RIGHTS RESERVED 38


Buyer: Jan Hiller (janhiller@gmx.net)
Transaction ID: 8BE25485KM511652L

Humanities, Cultural Studies and


Social Sciences Module

(A) Only statement (I) is correct.

(B) Only statement (II) is correct.

(C) Both statements are correct.

(D) Neither of the two statements is correct.

2.21.

Which of the following statements is/are correct according to the text?

I. There are no obvious approaches to solving the issue mentioned in the text.

II. The problem relates only to dealings between companies.

(A) Only statement (I) is correct.

(B) Only statement (II) is correct.

(C) Both statements are correct.

(D) Neither of the two statements is correct.

2.22.

Which of the following statements is/are correct according to the text?

I. The maximisation of profit takes top priority.

II. It has not yet been clarified whether or not companies care about their employees
behaving morally when faced with a choice between morals and profit.

(A) Only statement (I) is correct.

(B) Only statement (II) is correct.

(C) Both statements are correct.

(D) Neither of the two statements is correct.

© 2017 edulink GmbH www.edu-link.de ALL RIGHTS RESERVED 39


Buyer: Jan Hiller (janhiller@gmx.net)
Transaction ID: 8BE25485KM511652L

Humanities, Cultural Studies and


Social Sciences Module

2.5 ANSWER KEY

Exam 1 Exam 2

Question Answer Question Answer

1.1. D 2.1. B

1.2. B 2.2. A

1.3. D 2.3. C

1.4. B 2.4. C

1.5. A 2.5. A

1.6. B 2.6. D

1.7. D 2.7. B

1.8. B 2.8. D

1.9. A 2.9. A

1.10. A 2.10. A

1.11. D 2.11. D

1.12. C 2.12. C

1.13. C 2.13. C

1.14. A 2.14. D

1.15. D 2.15. B

1.16. C 2.16. C

1.17. B 2.17. B

1.18. B 2.18. A

1.19. C 2.19. D

1.20. A 2.20. D

1.21. D 2.21. A

1.22. C 2.22. B

© 2017 edulink GmbH www.edu-link.de ALL RIGHTS RESERVED 40


Buyer: Jan Hiller (janhiller@gmx.net)
Transaction ID: 8BE25485KM511652L

Humanities, Cultural Studies and


Social Sciences Module

2.6 DETAILED ANSWERS

2.6.1 EXAM 1

"THE LIFE OF MORITZ" (EXERCISES 1.1-1.4)

Five years ago, Moritz, now a 17-year-old student, developed a passion for tennis and
began playing as often as he could. All went smoothly until three months ago, when he got
to know a charming, bright and very pretty young lady named Stephanie. Moritz now had to
find a way to divide his time between Stephanie and all of his hobbies. Furthermore,
Stephanie wanted to become an engineer. It was her greatest dream to help people by
making her innovative ideas a reality. June had already arrived. In July, Stephanie wanted to
gain a deeper insight into the engineering world by completing a work experience placement
at the Volkswagen car company. This would make Moritz’s situation even more precarious.

1.1.

Which of the following statements is/are correct according to the text?

I. Stephanie is definitely going to become an engineer.

II. Moritz plays tennis very well.

(A) Only statement (I) is correct.

(B) Only statement (II) is correct.

(C) Both statements are correct.

(D) Neither of the two statements is correct.

Answer D

With this type of exercise, the first (I) and second (II) statements must be considered
independently of one another. Regarding statement (I): Stephanie’s current ambition is to
become an engineer. However, she is planning work experience to learn more about the
profession before making firm plans. It cannot, therefore, be said with certainty that
Stephanie will actually become an engineer. While the probability is high, it is not 100%. This
means that statement (I) is incorrect.

The text clearly states that Moritz is a passionate tennis player. However, it is possible to
pursue a sport passionately for five years without actually becoming good at it. In other

© 2017 edulink GmbH www.edu-link.de ALL RIGHTS RESERVED 41


Buyer: Jan Hiller (janhiller@gmx.net)
Transaction ID: 8BE25485KM511652L

Humanities, Cultural Studies and


Social Sciences Module

words, passion does not imply success, and the text gives no indication about whether
Moritz plays well or badly. This means that statement (II) is also incorrect.

Since both statements are incorrect, Answer D must be correct.

1.2.

Which of the following statements is/are correct according to the text?

I. Stephanie’s interests must take a back seat while she is in a relationship with Moritz.

II. Moritz is trying to make his relationship with Stephanie compatible with other parts of
his life.

(A) Only statement (I) is correct.

(B) Only statement (II) is correct.

(C) Both statements are correct.

(D) Neither of the two statements is correct.

Answer B

The text gives no indication whatsoever that Stephanie is prioritising her relationship with
Moritz ahead of her other interests. Rather, it says exactly the opposite: She will soon begin
a work experience placement at car company Volkswagen. Therefore, (I) is incorrect.

It is clearly stated in the passage that Moritz got to know Stephanie 3 months ago,
whereupon he began trying to divide his time fairly between his hobbies and Stephanie. In
other words, he wanted to integrate her into his life.

Only statement (II) agrees with the text, which means that Answer B is correct.

1.3.

Which of the following statements is/are correct according to the text?

I. Stephanie does not care at all about her career.

© 2017 edulink GmbH www.edu-link.de ALL RIGHTS RESERVED 42


Buyer: Jan Hiller (janhiller@gmx.net)
Transaction ID: 8BE25485KM511652L

Humanities, Cultural Studies and


Social Sciences Module

II. Stephanie has been accepted to do a work experience placement at Volkswagen.

(A) Only statement (I) is correct.

(B) Only statement (II) is correct.

(C) Both statements are correct.

(D) Neither of the two statements is correct.

Answer D

Since Stephanie has a high general interest in engineering and wants to do work experience
in the engineering industry, we can assume that Stephanie is career-oriented. Therefore, (I)
is incorrect.

The text does not convey that Stephanie has already been offered a work experience
position at Volkswagen, only that she wants to do work experience there. This means that
statement (II) is incorrect. It is irrelevant that the date on which she wishes to begin her work
experience is only one month away.

Since none of the statements agrees with the text, answer D must be right.

1.4.

How was Moritz likely to be feeling about Stephanie’s plans?

(A) He will break up with her.

(B) He was worried.

(C) He didn’t care.

(D) He was happy.

Answer B

The last sentence of the passage ("This would make Moritz’s situation even more
precarious") indicates that Moritz was not feeling secure in his relationship with Stephanie
and was unsure about the future (= worried). Therefore, answer B is correct.

© 2017 edulink GmbH www.edu-link.de ALL RIGHTS RESERVED 43


Buyer: Jan Hiller (janhiller@gmx.net)
Transaction ID: 8BE25485KM511652L

Humanities, Cultural Studies and


Social Sciences Module

"THE LITTLE TIGER" (EXERCISES 1.5-1.8)

There was once a little tiger whose mother died soon after he was born. A herd of sheep
living nearby took charge of him and accepted him as one of their own.

The little tiger soon turned into a big, noble creature. But he continued to behave like a
sheep. He ate grass, bleated like a sheep and sought protection from the herd. Although the
power of a tiger slumbered deep within him, he held fast to the belief that he was a sheep.

One day, an old tiger crept towards the herd with the aim of seizing one of the sheep. When
he saw the young tiger grazing amidst the sheep, he could hardly believe his eyes. He
chased after him, grabbed him by the fur on his neck and dragged him to a watering hole.
The young tiger smelled like a sheep and struggled anxiously against the insistence of the
older tiger that he was, in fact, a big cat – not a sheep as he believed.

The old tiger was insistent that the young tiger should look at his reflection in the water.
When he did so, he realised that he really wasn’t a sheep, but a tiger!

At that moment, a powerful roar erupted from deep inside of him, and his true nature
became clear to him at last.

(translation of a slightly amended excerpt from the German story "Sei dein bester Freund"
by Ulrike Dahm and Erich Keller; http://www.engelbrecht-media.de/s_der_kleine_tiger.html)

1.5.

We can assume that from now on, the tiger will behave like a...

(A) Tiger

(B) Sheep

(C) Deer

(D) Beaver

Answer A

Tiger (Answer A), since the last sentence (At that moment, a powerful roar erupted from
deep inside of him, and his true nature became clear to him at last) implies that the tiger has
now recognised that he is, in fact, a tiger and not a sheep. Because of this, we can assume
that his future behaviour will be that of a tiger. We might also mistakenly assume that the
tiger will continue to protect the sheep, since they are the ones who raised him. But this is

© 2017 edulink GmbH www.edu-link.de ALL RIGHTS RESERVED 44


Buyer: Jan Hiller (janhiller@gmx.net)
Transaction ID: 8BE25485KM511652L

Humanities, Cultural Studies and


Social Sciences Module

not likely because the "true nature" of the tiger can be seen in the example of the old tiger,
who wants to capture the sheep.

1.6.

Which of the following statements is/are correct according to the text?

I. It is impossible to hide one’s true nature from oneself forever.

II. One can carry a certain nature within oneself, but behave in a way that does not
reflect it.

(A) Only statement (I) is correct.

(B) Only statement (II) is correct.

(C) Both statements are correct.

(D) Neither of the two statements is correct.

Answer B

The story does not state explicitly that a person or creature cannot hide their true nature
from themselves forever. If the old tiger had not passed by the herd, the young tiger might
never have discovered his true nature. Therefore, statement (I) is incorrect.

Statement (II) correctly states that the nature of a tiger was present within the small tiger, but
that he behaved like a sheep until the old tiger convinced him of his true nature.

Only statement (II) is correct, which means that B is the correct answer.

1.7.

Which of the following statements is/are correct according to the text?

I. The "little tiger" ate grass, bleated like a sheep and ate meat.

II. The "little tiger" had lost the power he once had.

(A) Only statement (I) is correct.

© 2017 edulink GmbH www.edu-link.de ALL RIGHTS RESERVED 45


Buyer: Jan Hiller (janhiller@gmx.net)
Transaction ID: 8BE25485KM511652L

Humanities, Cultural Studies and


Social Sciences Module

(B) Only statement (II) is correct.

(C) Both statements are correct.

(D) Neither of the two statements is correct.

Answer D

The "little tiger" did eat grass and bleat like a sheep, but there is no indication in the text that
he ate meat. Therefore, statement (I) is incorrect.

The phrase "Although the power of a tiger slumbered deep within him…" shows clearly that
the power of the "small tiger" had not been lost. This means that (II) is incorrect.

Since both statements are incorrect, the correct answer is D.

1.8.

Imagine that the old tiger had not spotted the young tiger and had instead carried out his
plan of attacking one of the sheep. Based on what you read in the passage, which of these
outcomes would have been most likely?

I. The young tiger would have instinctively recognised himself as the strongest member
of the herd and jumped to the sheep’s defence.

II. The young tiger would have felt scared and tried to hide like the rest of the herd.

(A) Only statement (I) is correct.

(B) Only statement (II) is correct.

(C) Both statements are correct.

(D) Neither of the two statements is correct.

Answer B

The key piece of information here comes earlier in the paragraph: "… and (he) sought
protection from the herd". Since he felt the need to seek protection from the herd, we can
assume that his urge to behave like a sheep was stronger than the power that was
slumbering within him. Throughout the paragraph, we see further evidence of how firmly the
young tiger believed he was a sheep ("The young tiger…struggled anxiously against the
insistence of the older tiger that he was, in fact, a big cat").

© 2017 edulink GmbH www.edu-link.de ALL RIGHTS RESERVED 46


Buyer: Jan Hiller (janhiller@gmx.net)
Transaction ID: 8BE25485KM511652L

Humanities, Cultural Studies and


Social Sciences Module

"PROTEIN – THE SOURCE OF LIFE" (EXERCISES 1.9-1.11)

Virtually all processes occurring in living things are dependent on protein (Greek: proteios =
"of prime importance"). Therefore, they are of great importance for every single living
creature in this world. Proteins are biological macromolecules consisting of one or more
polypeptide chains, each of which is made from up to 22 proteinogenic amino acids. Eight of
these amino acids are what is known as "essential amino acids", which cannot be
synthesised internally and must therefore be taken in by living organisms in their food. The
amino acids are connected to each other with peptide bonds (-CONH-). The specific role of
each protein within an organism is determined by its spatial structure, which can be
described at four distinct structural levels (primary, secondary, tertiary and quaternary).

As enzymes (biocatalysts), proteins can be responsible for speeding up certain chemical


reactions or for enabling them to happen at all. For example, digestive enzymes catalyse the
hydrolysis of polymers. When behaving as structural proteins, proteins provide support and
give structure; for example, collagen and elastin form a fibrous mesh in the connective
tissues of animals. As hormones, proteins coordinate the various processes happening
within a living organism. For example, insulin – a hormone produced by the pancreas –
regulates the blood sugar concentration of vertebrates.

1.9.

Which of the following statements is/are correct according to the text?

I. Humans require protein to stay alive.

II. Elastin speeds up chemical reactions.

(A) Only statement (I) is correct.

(B) Only statement (II) is correct.

(C) Both statements are correct.

(D) Neither of the two statements is correct.

Answer A

Statement (I) is correct, as the introductory text clearly mentions that it is the basis of life –
and therefore humans require protein to stay alive.

© 2017 edulink GmbH www.edu-link.de ALL RIGHTS RESERVED 47


Buyer: Jan Hiller (janhiller@gmx.net)
Transaction ID: 8BE25485KM511652L

Humanities, Cultural Studies and


Social Sciences Module

Statement (II) is incorrect, since elastin is a structural protein.

Only statement (I) is correct, so the correct answer is A.

1.10.

Which of the following statements is/are correct according to the text?

I. The spatial structure of a protein determines its effects.

II. Insulin regulates the blood sugar concentration of slugs.

(A) Only statement (I) is correct.

(B) Only statement (II) is correct.

(C) Both statements are correct.

(D) Neither of the two statements is correct.

Answer A

Statement (I) is correct. It is mentioned in the text passage that the role of the protein is
determined by its spatial structure.

Statement (II) is incorrect, since a slug is not a vertebrate.

Only statement (I) is correct, which means that A is the correct answer.

1.11.

Which of the following statements is/are correct according to the text?

I. "Proteins" are also referred to as "biocatalysts".

II. There are 22 essential amino acids.

(A) Only statement (I) is correct.

(B) Only statement (II) is correct.

© 2017 edulink GmbH www.edu-link.de ALL RIGHTS RESERVED 48


Buyer: Jan Hiller (janhiller@gmx.net)
Transaction ID: 8BE25485KM511652L

Humanities, Cultural Studies and


Social Sciences Module

(C) Both statements are correct.

(D) Neither of the two statements is correct.

Answer D

No, proteins in general are not referred to as biocatalysts. Only enzymes can be referred to
as biocatalysts. Therefore, statement (I) is incorrect.

There are only 8 essential amino acids. Therefore, statement II is also incorrect.

Statements (I) and (II) are both incorrect, which means that Answer D is correct.

"EXCERPT FROM A CONSTITUTIONAL COMPLAINT" (EXERCISES 1.12-1.14)

Pursuant to Article 93 (1) No. 4a GG (Basic Law for the Federal Republic of Germany) in
conjunction with §§13 No. 8a, 90 ff. BVerfG (Law on the Federal Constitutional Court of
Germany), the Federal Constitutional Court is responsible for the constitutional complaint by
the complainant. A complaint is deemed to be lawful if it is submitted in the correct form and
in a timely manner. Pursuant to §23 (1) 1 BVerfGG, a complaint must be submitted to the
Federal Constitutional Court in writing in order to satisfy the formal requirements. Any type of
written form is deemed to be admissible. Telex and telefax are objectionable. While, as a
basic principle, emails are not admissible as evidence, the literature indicates that emails are
sometimes admitted or acknowledged to be admissible.

In addition, according to § 92 BVerfGG, the constitutional complaint must be justified through


the naming of the basic right that has been breached.

Furthermore, the time limits set forth in §93 BVerfGG must be adhered to. In the case of a
constitutional complaint regarding acts of the administration (e.g. an administrative act) and
the judicial system (e.g. an incriminating judgement), the one-month time limit set forth in
§93 (1) 1 BverfGG shall apply; for a constitutional complaint against a law, a time limit of one
year from the date the law comes into force shall apply in accordance with § 93 (3) BverfGG.

1.12.

Which of the following statements is/are correct according to the text?

I. A complaint is deemed to be lawful if it is submitted in the correct form and in a timely


manner.

© 2017 edulink GmbH www.edu-link.de ALL RIGHTS RESERVED 49


Buyer: Jan Hiller (janhiller@gmx.net)
Transaction ID: 8BE25485KM511652L

Humanities, Cultural Studies and


Social Sciences Module

II. It is compulsory to submit the constitutional complaint in writing.

(A) Only statement (I) is correct.

(B) Only statement (II) is correct.

(C) Both statements are correct.

(D) Neither of the two statements is correct.

Answer C

Statements (I) and (II) are clearly mentioned in the text and are therefore correct.

Since both statements are correct, Answer C is correct.

1.13.

Which of the following statements is/are correct according to the text?

I. § 93 BverfGG describes some of the time limits for a constitutional complaint.

II. The one-month time limit applies for judgements of the Court.

(A) Only statement (I) is correct.

(B) Only statement (II) is correct.

(C) Both statements are correct.

(D) Neither of the two statements is correct.

Answer C

Statement (I) is correct, since §93 BVerfGG describes some of the time limits for a
constitutional complaint.

Statement (II) is correct since an incriminating judgement is classified as an act of the


judicial system.

Statement (I) and statement (II) both are correct, which means that C is the correct answer
choice.

© 2017 edulink GmbH www.edu-link.de ALL RIGHTS RESERVED 50


Buyer: Jan Hiller (janhiller@gmx.net)
Transaction ID: 8BE25485KM511652L

Humanities, Cultural Studies and


Social Sciences Module

1.14.

Which of the following statements is/are correct according to the text?

I. Constitutional complaints can be sent via telex.

II. Time limits can be extended if there is good cause.

(A) Only statement (I) is correct.

(B) Only statement (II) is correct.

(C) Both statements are correct.

(D) Neither of the two statements is correct.

Answer A

Statement (I) is correct, since "any type of written form is deemed to be admissible".

Statement (II) is not correct as the text does not comment on this.

Only statement (I) is correct, which means that Answer A is correct.

"THE FEDERAL PRESIDENT OF GERMANY" (EXERCISES 1.15-1.18)

"I swear that I will dedicate my efforts to the well-being of the German people, promote their
welfare, protect them from harm, uphold and defend the Basic Law and the laws of the
Federation, perform my duties conscientiously, and do justice to all. " – The oath of office of
the Federal President of Germany

By taking this oath, the Federal President of Germany indicates - in accordance with Article
56 of the Basic Law - that in taking up office, he or she commits themselves to high goals on
behalf of the German people and assumes a great responsibility towards them. Based on
this part of the Basic Law and on the Federal President’s manifestation as a permanent
constitutional body and designation as head of state, one could assume that the Federal
President enjoys a strong, influential position in the political process. However, although the
Federal Presidents of Germany do function as head of state and occupy what was formally
the highest position in the German political system, their national-political power is very
limited by the constitution, and they perform mainly representative duties. For this reason,
they are known as the highest representative of the Federal Republic of Germany. In return

© 2017 edulink GmbH www.edu-link.de ALL RIGHTS RESERVED 51


Buyer: Jan Hiller (janhiller@gmx.net)
Transaction ID: 8BE25485KM511652L

Humanities, Cultural Studies and


Social Sciences Module

for undertaking these duties, the Federal President receives 10/9 of the official salary of the
Federal Chancellor, which corresponds to an amount of 199,000€. After leaving office, the
Federal President is paid an Ehrensold (presidential pension) in the amount of their salary
every year for the rest of their life. This salary and pension are a lot higher than those
received by the average German person. In light of the above facts, many Germans are
questioning whether the Office of the Federal President still fulfills a necessary function for
the Federal Republic of Germany.

1.15.

Which of the following statements is/are correct according to the text?

I. The Federal President swears to serve the well-being of the EU.

II. The Federal Presidents takes their oath of office in accordance with Article 46 of the
Basic Law.

(A) Only statement (I) is correct.

(B) Only statement (II) is correct.

(C) Both statements are correct.

(D) Neither of the two statements is correct.

Answer D

Statement (I) is incorrect. The Federal President swears that he will dedicate his efforts to
the well-being of the German people.

Statement (II) is also incorrect, since the Federal President takes their oath of office in
accordance with Article 56 of the Basic Law.

Both statements are incorrect, which means that D is correct.

1.16.

Which of the following statements is/are correct according to the text?

I. The Federal President earns more than the Federal Chancellor.

II. The Federal President undertakes duties of a primarily representative nature.

© 2017 edulink GmbH www.edu-link.de ALL RIGHTS RESERVED 52


Buyer: Jan Hiller (janhiller@gmx.net)
Transaction ID: 8BE25485KM511652L

Humanities, Cultural Studies and


Social Sciences Module

(A) Only statement (I) is correct.

(B) Only statement (II) is correct.

(C) Both statements are correct.

(D) Neither of the two statements is correct.

Answer C

Statement (I) is correct, since the Federal President earns 10/9 of the Federal Chancellor’s
salary.

Statement (II) is also correct. Although the Federal President is the head of state, the text
clearly states that they fulfill a primarily representative role.

Both statements are correct, which means that Answer C is correct.

1.17.

Which of the following statements is/are correct according to the text?

I. The Federal Presidents are not limited in their powers.

II. The Federal President is formally ranked above the Federal Chancellor in the Federal
Republic of Germany.

(A) Only statement (I) is correct.

(B) Only statement (II) is correct.

(C) Both statements are correct.

(D) Neither of the two statements is correct.

Answer B

Statement (I) is incorrect, since the text states that the powers of the Federal President are
limited by the constitution.

Statement (II) is correct, since the Federal President is the head of state.

Only statement (II) is correct, which means than Answer B is correct.

© 2017 edulink GmbH www.edu-link.de ALL RIGHTS RESERVED 53


Buyer: Jan Hiller (janhiller@gmx.net)
Transaction ID: 8BE25485KM511652L

Humanities, Cultural Studies and


Social Sciences Module

1.18.

Based on the text, how do you think the average German person would be likely to react if
the Federal President’s salary was increased?

(A) They wouldn’t care.

(B) They would be annoyed.

(C) They would be approving.

(D) They would be sad.

Answer B

The text tells us that even though the Federal President performs mainly representative
duties, he/she is already considerably wealthier than the average German person. We also
know that many German people are questioning the necessity of the Federal President
precisely because of this fact. Therefore, it is reasonable to assume that the average
German person would have negative feelings towards any increase in the Federal
President’s salary (thus the answer cannot be A or C). Of the two "negative" options listed,
Answer B ("annoyed") is the most likely given the context.

"THE FUTURE OF EUROPE" (EXERCISES 1.19.-1.22.)

In fifty years’ time, Europe and the USA will make up only seven per cent of the world’s
population. This will represent the most fundamental change in global population distribution
that humanity has ever seen. Europe must accept that it is a community of values and that
the USA, as the world’s oldest democracy, is an essential partner - regardless of the socio-
political differences that came to light during the NSA affair. People in Europe have been
fighting for their freedom for the last 500 years; milestones like the French Revolution, the
German Revolution of 1848, the Weimar Republic and the fall of the Berlin Wall in 1989 are
just a few of the events that have paved the way for the current system of democracy.
Europe has not only the choice but also the moral duty to preserve these values for the
future and to assert themselves in disputes. The unified stance of Europe in the world is
therefore not an option but an obligation.

1.19.

Which of the following statements is/are correct according to the text?

© 2017 edulink GmbH www.edu-link.de ALL RIGHTS RESERVED 54


Buyer: Jan Hiller (janhiller@gmx.net)
Transaction ID: 8BE25485KM511652L

Humanities, Cultural Studies and


Social Sciences Module

I. Over the next 50 years, 93 per cent of the world’s population will come to be located
outside of the USA and Europe.

II. Population distribution has never before altered so drastically.

(A) Only statement (I) is correct.

(B) Only statement (II) is correct.

(C) Both statements are correct.

(D) Neither of the two statements is correct.

Answer C

Statement (I) is correct, since Europe and the USA will come to represent 7 per cent of the
world’s population over the next fifty years. This means that the rest of the world will contain
93% of its population. Statement (II) is correct, because the text clearly states that this is the
most significant change in population distribution that has ever occurred.

Both statements are correct, which means that C is correct.

1.20.

Which of the following statements is/are correct according to the text?

I. The German Revolution of 1848 and the fall of the Berlin Wall contributed to
establishing the current system of democracy.

II. In the future, Europe will be able to decide freely whether it wants to operate as a
community.

(A) Only statement (I) is correct.

(B) Only statement (II) is correct.

(C) Both statements are correct.

(D) Neither of the two statements is correct.

Answer A

Statement (I) is correct, because the text states that both of these events paved the way for
the current system of democracy. Statement (II) is incorrect, since the text expresses the

© 2017 edulink GmbH www.edu-link.de ALL RIGHTS RESERVED 55


Buyer: Jan Hiller (janhiller@gmx.net)
Transaction ID: 8BE25485KM511652L

Humanities, Cultural Studies and


Social Sciences Module

opinion that it is Europe’s duty to present a united front. Only statement (I) is correct, which
means that Answer A is correct.

1.21.

Which of the following statements is/are correct according to the text?

I. The NSA affair was not beneficial for the relationship between the USA and
Germany.

II. Europe must further improve its relationship with the USA.

(A) Only statement (I) is correct.

(B) Only statement (II) is correct.

(C) Both statements are correct.

(D) Neither of the two statements is correct.

Answer D

Statement (I) is incorrect, since the text doesn't state that the NSA affair was not beneficial
for the relationship between the USA and Europe. While it is certain that German-American
relations also suffered, there is nothing to confirm this in the text. Hence, statement (I) is
incorrect.

Statement (II) is also incorrect, since the text does not explicitly state that the relationship
between Europe and the USA should be improved. According to the text, it is only necessary
for Europe to recognise that the USA is an important partner.

Both statements are incorrect. Therefore, Answer D is correct.

1.22.

The text states that Europe has a duty to preserve democratic values. Based on the text,
what would be the likely effect on Europe’s efforts to preserve democratic values if it did not
cooperate with America?

(A) It would make no difference.

© 2017 edulink GmbH www.edu-link.de ALL RIGHTS RESERVED 56


Buyer: Jan Hiller (janhiller@gmx.net)
Transaction ID: 8BE25485KM511652L

Humanities, Cultural Studies and


Social Sciences Module

(B) Europe’s efforts would be slightly less effective.

(C) Europe’s efforts would be much less effective.

(D) Europe’s efforts would be more effective.

Answer C

The clue lies in the part of the text that states "Europe must accept that… the USA, as the
world’s oldest democracy, is an essential partner - regardless of the socio-political
differences that came to light during the NSA affair". The mention of "partner" clearly
suggests that America’s support is important for ensuring that democracy is preserved. This
rules out Answers A and D. It also rules out Answer B, since the use of the adjective
"essential" implies that America will play a large role rather than a smaller one.

© 2017 edulink GmbH www.edu-link.de ALL RIGHTS RESERVED 57


Buyer: Jan Hiller (janhiller@gmx.net)
Transaction ID: 8BE25485KM511652L

Humanities, Cultural Studies and


Social Sciences Module

2.6.2 EXAM 2

"THE ORIGINS OF THE OLYMPICS" (EXERCISES 2.1.-2.3.)

The Olympic Games originated in Olympia, Greece, with most sources agreeing that the first
official games took place in 776 BCE. This figure is based on inscriptions in Olympia that list
the winners of a footrace held in 776 BCE and every four years afterwards; however, many
scholars believe that the games had actually been happening in some form for many years
before that. According to tradition, the first champion of the games was a cook named
Coroebus, who took the title by winning the sole event at the time – a 192-metre (210-yard)
run. Held at the sanctuary of Zeus in Olympia, the games continued to grow for the next
1200 years and eventually came to feature running events, a pentathlon, boxing, wrestling,
pankration, and equestrian events. As well as being a sporting event, the Ancient Olympics
also featured many ceremonies and ritual sacrifices to honour the god Zeus. In 393 CE, a
Roman emperor named Theodosius I, who was a Christian, abolished the games due to
their connection with pagan beliefs. The Olympic tradition was revived in the 1800s, and the
first modern games were held in 1896.

2.1.

Which of the following statements is/are correct according to the text?

I. Most experts believe that the idea for the Olympic games was thought up in 776 BC.

II. Running was the only sport in the first official Olympic games.

(A) Only statement (I) is correct.

(B) Only statement (II) is correct.

(C) Both statements are correct.

(D) Neither of the two statements is correct.

Answer B

Statement (I) is incorrect. The text states that whilst the first official games is thought to have
taken place in 776 BC, "many scholars (=experts) believe that the games had actually been
happening in some form for many years before that". Statement (II) is correct, since the text
informs us about "inscriptions in Olympia that list the winners of a footrace held in 776 BCE
and every four years afterwards" and about the first ever champion of the games, "who took
the title by winning the sole (=only) event at the time – a 192-metre (210-yard) run". Since
only statement (II) is true, Answer B is the correct choice.

© 2017 edulink GmbH www.edu-link.de ALL RIGHTS RESERVED 58


Buyer: Jan Hiller (janhiller@gmx.net)
Transaction ID: 8BE25485KM511652L

Humanities, Cultural Studies and


Social Sciences Module

2.2.

Which of the following statements is/are correct according to the text?

I. The Ancient Olympics was both a sporting and a religious event.

II. Like the modern Olympics, the ancient Olympics featured athletes from all over the
world.

(A) Only statement (I) is correct.

(B) Only statement (II) is correct.

(C) Both statements are correct.

(D) Neither of the two statements is correct.

Answer A

Statement (I) is true – we know that the games had a religious element from the part of the
text that says, "As well as being a sporting event, the Ancient Olympics also featured many
ceremonies and ritual sacrifices to honour the god Zeus. " However, there is no information
in the text to support statement (II). Thus, even though we might suspect it to be true based
on what we know about the modern-day Olympics, we have to conclude that - according to
the text - it is incorrect. (In actual fact, it is incorrect according to other historical sources too
– the ancient games took place among participants from different city-states and kingdoms
of ancient Greece).

2.3.

Which of the following statements is/are correct according to the text?

I. The emperor Theodosius I got rid of the games because they clashed with his
religious beliefs.

II. It was more than a thousand years before the games were brought back to life.

(A) Only statement (I) is correct.

(B) Only statement (II) is correct.

(C) Both statements are correct.

(D) Neither of the two statements is correct.

© 2017 edulink GmbH www.edu-link.de ALL RIGHTS RESERVED 59


Buyer: Jan Hiller (janhiller@gmx.net)
Transaction ID: 8BE25485KM511652L

Humanities, Cultural Studies and


Social Sciences Module

Answer C

Statement (I) is correct. We know this because the text states, "In 393 CE, a Roman
emperor named Theodosius I, who was a Christian, abolished (=got rid of) the games due to
their connection with pagan beliefs." Here, the crucial information is that Theodosius I was
Christian. Even if we would not know the word "pagan" – which describes an ancient,
polytheistic (many-god) religion – we can deduce that the emperor disliked the games
because of their connection with beliefs that were different from his own.

To reach a conclusion about statement (II), we must consider the final sentence ("The
Olympic tradition was revived in the 1800s and the first modern games were held in 1896")
in connection with the previous sentence about Theodosius I abolishing the games in 393
CE. It is obvious that there are more than one thousand years between the year 393 CE and
the holding of the first modern games in 1896 (CE). Hence, statement (II) is correct.

Both statements (I) and (II) are true, which means that Answer C is the correct choice.

"THE FREIBURG THESES OF LIBERALISM" (EXERCISES 2.4.-2.6.)

The rationale behind liberal thinking is illustrated clearly in the following (originally German)
quote: "Liberalism leaves people in peace, without abandoning them completely." The four
Freiburg Theses were adopted on 27 October 1971 at the Federal Party Convention of the
FDP (Free Democratic Party, a classical liberal party in Germany) after the FDP received
only 5.8% of the vote at the 1969 Bundestag elections (German parliamentary elections).

The first thesis states that people should be given precedence over institutions and that the
preservation and development of the individual and the concept of plurality should be
promoted within human societies. The second thesis says that liberalism stands for progress
and reason. According to the third thesis, we are all part of society, but "society must not be
permitted to be everything". The fourth thesis supports a reform of the capitalistic system to
abolish trends for the accumulation of wealth in only a few hands. At its core, a liberal
society should serve people, promote an increase in output and ensure the protection of the
needy and the environment.

2.4.

Which of the following statements is/are correct according to the text?

I. At the 1969 Bundestag elections, the FDP received only 5.8% of the vote.

© 2017 edulink GmbH www.edu-link.de ALL RIGHTS RESERVED 60


Buyer: Jan Hiller (janhiller@gmx.net)
Transaction ID: 8BE25485KM511652L

Humanities, Cultural Studies and


Social Sciences Module

II. One of the functions of a liberal society is to protect the needy.

(A) Only statement (I) is correct.

(B) Only statement (II) is correct.

(C) Both statements are correct.

(D) Neither of the two statements is correct.

Answer C

Both statements are clearly communicated by the text, which means that Answer C is
correct.

2.5.

Which of the following statements is/are correct according to the text?

I. According to the third thesis, the German federal chancellor Angela Merkel is a part
of society.

II. The result of the 1969 Bundestag election was the worst in the FDP’s history.

(A) Only statement (I) is correct.

(B) Only statement (II) is correct.

(C) Both statements are correct.

(D) Neither of the two statements is correct.

Answer A

The third theory states that we all are part of society, which means that Angela Merkel is
included. Therefore, statement (I) is correct. However, statement (II) is incorrect, since there
is nothing in the text to support it.

Only statement (I) is correct, which means that A is the right answer.

© 2017 edulink GmbH www.edu-link.de ALL RIGHTS RESERVED 61


Buyer: Jan Hiller (janhiller@gmx.net)
Transaction ID: 8BE25485KM511652L

Humanities, Cultural Studies and


Social Sciences Module

2.6.

Which of the following statements is/are correct according to the text?

I. According to the second thesis, the FDP is an advocate of science.

II. Environmental protection is considered to be of subordinate importance within a


liberal society.

(A) Only statement (I) is correct.

(B) Only statement (II) is correct.

(C) Both statements are correct.

(D) Neither of the two statements is correct.

Answer D

Statement (I) is incorrect. The second thesis informs us that liberalism advocates progress
and reason, from which we can derive that it also advocates science. However, although the
FDP is a liberal party, we cannot derive from the text whether it preserves all liberal ideals,
particularly since the party operates as part of a political system. Statement (II) is also
incorrect. Even though it may be true that environmental protection is considered an issue of
secondary importance in liberal societies, the text gives no indication of this.

Both statements are incorrect, which means that Answer D is correct.

"ANIMAL SPECIES FACING EXTINCTION" (EXERCISES 2.7.-2.9.)

The impact of human activity on the planet is putting an increasing number of animal species
at risk of extinction. Though extinction is a natural process that has occurred throughout the
earth’s history, studies show that man-made factors such as climate change, hunting and
poaching, fishing practices, pollution and loss of habitat are accelerating the process and
creating unprecedented numbers of endangered species. Some of the most at-risk species
include tigers, polar bears, penguins and various kinds of fish. Many insects are also
threatened – a recent assessment of European wild bee species found that 9.2% were in
danger becoming extinct. Of all the ways that humans are changing the planet, temperature
increases due to global warming are thought to be the most dangerous. If the planet warms
by an average of 3 degrees C, scientists predict that 8.5% of the world’s species will be in
danger of ceasing to exist.

© 2017 edulink GmbH www.edu-link.de ALL RIGHTS RESERVED 62


Buyer: Jan Hiller (janhiller@gmx.net)
Transaction ID: 8BE25485KM511652L

Humanities, Cultural Studies and


Social Sciences Module

2.7.

Which of the following statements is/are correct according to the text?

I. Extinction is a man-made problem.

II. A reduction in the size of natural habitats is one of the factors putting animals at
increased risk.

(A) Only statement (I) is correct.

(B) Only statement (II) is correct.

(C) Both statements are correct.

(D) Neither of the two statements is correct.

Answer B

Statement (I) is incorrect. The text states that "extinction is a natural process that has
occurred throughout the earth’s history", but makes the point that humans are responsible
for putting some species at increased risk. Statement (II) is correct, since the text informs us
that "man-made factors such as…loss of habitat" are responsible for creating increased
numbers of endangered (=at risk) species. Since only statement (II) is correct, Answer B is
the right choice.

2.8.

Which of the following statements is/are correct according to the text?

I. Just over 9% of all wild bee species worldwide are thought to be at risk of extinction.

II. Bad fishing practices are responsible for the fact that many kinds of fish are now
endangered.

(A) Only statement (I) is correct.

(B) Only statement (II) is correct.

(C) Both statements are correct.

(D) Neither of the two statements is correct.

Answer D

© 2017 edulink GmbH www.edu-link.de ALL RIGHTS RESERVED 63


Buyer: Jan Hiller (janhiller@gmx.net)
Transaction ID: 8BE25485KM511652L

Humanities, Cultural Studies and


Social Sciences Module

Statement (I) is designed to test your attention to detail. Those of you who read the text
closely will notice that it actually mentioned "a recent assessment of European wild bee
species" rather than wild bee species worldwide. This means that statement (I) is false. As
for statement (II), while the text does mention that fishing practices are an issue and that
many species of fish are at risk, we do not have enough information to say with certainty that
statement (II) is correct. While it would be natural to assume some connection between
fishing and endangered fish species, the text gives us no reason to suspect that fishing
practices are solely responsible. Other man-made problems (such as a climate change,
pollution and habitat loss) could easily be playing a role as well, and some endangered
species of fish might not be valued as food at all. In light of all this, we must conclude that
statement (II) is incorrect. Since both statements are incorrect, Answer D is the right choice.

2.9.

Which of the following statements is/are correct according to the text?

I. Temperature rises due to global warming are generally considered to be a more


significant threat to animals than hunting and poaching.

II. If the earth’s temperature rises by an average of 3 degrees C, the number of animal
species will be reduced by 8.5%.

(A) Only statement (I) is correct.

(B) Only statement (II) is correct.

(C) Both statements are correct.

(D) Neither of the two statements is correct.

Answer A

Statement (I) is correct, since the text tells us that "of all the ways that humans are changing
the planet, temperature increases due to global warming are thought to be the most
dangerous". Statement (II), however, is incorrect, since the text states that "if the planet
warms by an average of 3 degrees C, scientists predict that 8.5% of the world’s species will
be in danger of ceasing to exist." It tells us that there is a risk of 8.5% of the world’s animal
species becoming extinct, but it does not say for sure that this will happen. Since only
Statement (I) is correct, Answer A is the right choice.

© 2017 edulink GmbH www.edu-link.de ALL RIGHTS RESERVED 64


Buyer: Jan Hiller (janhiller@gmx.net)
Transaction ID: 8BE25485KM511652L

Humanities, Cultural Studies and


Social Sciences Module

"THE LION AND THE MOUSE" (EXERCISES 2.10.-2.12.)

Once upon a time, a lion was sleeping deeply and peacefully in the jungle. As he slept, a
small mouse passed by. Without realising what he was doing, the mouse ran over the lion,
tickled him and woke him up. Immediately, the lion trapped the mouse under his big paw and
opened his mouth to swallow him whole. "Pardon me, Mr Lion!" squeaked the mouse.
"Please forgive me. I’ll make sure never to do such a thing again, and I’ll never forget your
kindness. Perhaps one day I’ll be able to repay you." The lion smiled to himself. He didn’t
see how such a small creature could ever be of any use to a lion, but he was so amused by
the mouse’s request that he let him go. It just so happened that some hours later, some
hunters passed by, captured the lion and tied him to a tree while they looked for a wagon to
carry him home. The mouse passed by once again, saw the sad plight of the lion, and set
about chewing through the ropes. Soon, the lion was free and the mouse was beaming with
pride. "Did I not tell you I’d pay you back someday?" he asked the lion. "Yes," agreed the
lion, smiling widely, "you were right!"

2.10.

Which of the following statements is/are correct according to the text?

I. The mouse didn’t mean to annoy the lion.

II. The lion let the mouse go because he was expecting to be repaid.

(A) Only statement (I) is correct.

(B) Only statement (II) is correct.

(C) Both statements are correct.

(D) Neither of the two statements is correct.

Answer A

Statement (I) is correct. We know this from the part of the text that says, "Without realising
what he was doing, the mouse ran over the lion, tickled him and woke him up." This conveys
clearly that the mouse ran over the lion by accident, not by intention.

Statement (II) is incorrect. To reach this conclusion, we must look at the part of the text that
says: "He didn’t see how such a small creature could ever be of any use to a lion, but he
was so amused by the mouse’s request that he let him go." The first part of this sentence
communicates clearly that the lion didn’t believe the mouse would ever be able to repay him,
but that he chose to let him go anyway. The second part of the sentence tells us why –
because he found the mouse’s request amusing.

© 2017 edulink GmbH www.edu-link.de ALL RIGHTS RESERVED 65


Buyer: Jan Hiller (janhiller@gmx.net)
Transaction ID: 8BE25485KM511652L

Humanities, Cultural Studies and


Social Sciences Module

Only statement (I) is correct, so Answer A is the right choice.

2.11.

Which of the following statements is/are correct according to the text?

I. The lion would definitely have died if the mouse hadn’t saved him.

II. The lion will never think of eating the mouse again.

(A) Only statement (I) is correct.

(B) Only statement (II) is correct.

(C) Both statements are correct.

(D) Neither of the two statements is correct.

Answer D

Here, our job is to identify whether these statements are explicitly given as facts in the text.
Although both of them seem very likely after reading the text, we cannot say for sure –
based on the information we have – that either statement is definitely correct. In the case of
statement (I), the lion might have found another way to free himself, or the hunters might
have decided to give him to a zoo. With regard to statement (II), we know that the lion is
happy about the mouse saving him, but this is not sufficient to guarantee that the lion will
never see the mouse as prey again.

For these reasons, we have to deduce that both statements are incorrect, hence Answer D
is correct.

2.12.

How did the lion probably feel after being rescued by the mouse?

(A) He was humiliated.

(B) He was shocked.

(C) He was grateful.

© 2017 edulink GmbH www.edu-link.de ALL RIGHTS RESERVED 66


Buyer: Jan Hiller (janhiller@gmx.net)
Transaction ID: 8BE25485KM511652L

Humanities, Cultural Studies and


Social Sciences Module

(D) He was confused.

Answer C

The text does not explicitly state how the lion was feeling, so we have to work out the
answer by eliminating the most unlikely options. The relevant part of the text comes in the
last two lines: "'Did I not tell you I’d pay you back someday?' he asked the lion. 'Yes,' agreed
the lion, smiling widely, 'you were right!'" We know that the lion was smiling widely and that
he agreed readily with the mouse, so we can rule out the possibilities that he was humiliated
or confused. This leaves us to choose between "shocked" and "grateful". Although it could
be possible that he was feeling shocked – since he wasn’t expecting the mouse to be able to
save him – the fact that he was smiling widely makes it more likely that he was feeling a
positive emotion than a negative one. Therefore, grateful is the most likely choice.

"THE STUDY" (EXERCISES 2.13.-2.15.)

The (originally German-language) article "Student Writing Guidance is Professional and


Personalised" discusses a research project from the Writing Centre at the European
University Viadrina in Frankfurt (Oder), Germany. Within the student writing guidance
scheme, students seeking guidance were offered the help of other students. The goal of the
scheme was to bring about an increase in writing competence. Various concepts were
deployed in an attempt to achieve this, including "help with self-help", the exchange of
experiences, emotions and knowledge, and active engagement with individual writing
projects. The study was compiled from qualitative content analyses and problem-focused
interviews. The goal of qualitative content analysis is the systematic processing of
communication content that must adhere to a certain form. It does not reflect the content but
is instead aimed at representing formal aspects and latent meaning. A category system was
developed on the basis of the qualitative content analysis and used to measure how the
student writing guidance was received by the guidance-seeking students. The guidance-
seekers were asked how they entered and exited the guidance session, how they perceived
the session, what occurred to them, how they felt during and after the session and whether
the session triggered anything new in relation to them as individuals or their writing.

2.13.

Which of the following statements is/are correct according to the text?

I. The student writing guidance scheme requires students to help other students.

© 2017 edulink GmbH www.edu-link.de ALL RIGHTS RESERVED 67


Buyer: Jan Hiller (janhiller@gmx.net)
Transaction ID: 8BE25485KM511652L

Humanities, Cultural Studies and


Social Sciences Module

II. The student writing guidance scheme was intended to bring about an increase in
writing competence.

(A) Only statement (I) is correct.

(B) Only statement (II) is correct.

(C) Both statements are correct.

(D) Neither of the two statements is correct.

Answer C

Statements (I) and (II) are clearly evident from the text and are therefore correct. Since both
statements are correct, Answer C is correct.

2.14.

Which of the following statements is/are correct according to the text?

I. The study was predominantly compiled from problem-focused interviews.

II. Qualitative content analysis arose out of the category system.

(A) Only statement (I) is correct.

(B) Only statement (II) is correct.

(C) Both statements are correct.

(D) Neither of the two statements is correct.

Answer D

Statement (I) is incorrect. While the text did state that the study’s results were partially
comprised of problem-focused interviews, the proportion of the study that was accounted for
by the interviews is not made clear to us. Statement (II) is incorrect, since the text indicates
that the category system is based on qualitative content analysis. Both statements are
incorrect, which means that Answer D is correct.

© 2017 edulink GmbH www.edu-link.de ALL RIGHTS RESERVED 68


Buyer: Jan Hiller (janhiller@gmx.net)
Transaction ID: 8BE25485KM511652L

Humanities, Cultural Studies and


Social Sciences Module

2.15.

Which of the following statements is/are correct according to the text?

I. The purpose of qualitative content analysis is the graphical processing of


communication content.

I. Qualitative content analysis goes beyond merely compiling information about the
actual content.

(A) Only statement (I) is correct.

(B) Only statement (II) is correct.

(C) Both statements are correct.

(D) Neither of the two statements is correct.

Answer B

Statement (I) is incorrect, since the purpose of qualitative content analysis is the systematic
processing of communication content. Statement (II) is correct, since according to the text,
qualitative content analysis also looks at formal aspects and latent meaning.

Only statement (II) is correct, which means that B is correct.

"THE PUPIL" (EXERCISES 2.16.-2.19.)

At the beginning of an assignment, David always reads the instructions carefully and then
skims through the text to get an overview before starting the first question. When he comes
across a problem he cannot solve, he simply skips over the problem and goes to the next
question. When he finishes the work, he looks at the problem once more and, if necessary,
he asks the teacher for help. When learning new things, he relies primarily on memory and
observation – I can tell that he is better able to memorise information when he sees things.
When it comes to German, David is quiet during classes and follows the lesson – quite in
contrast to his behaviour during natural history. Though he also follows the lesson here, he
talks much more with his fellow pupils and shares jokes, most of which come from him. The
subject of the "central nervous system" is one that particularly appeals to him. David finds it
easy to draw logical conclusions, which helps him to excel in fact-based subjects like maths
and science. However, he finds it difficult to utilise his creativity.

© 2017 edulink GmbH www.edu-link.de ALL RIGHTS RESERVED 69


Buyer: Jan Hiller (janhiller@gmx.net)
Transaction ID: 8BE25485KM511652L

Humanities, Cultural Studies and


Social Sciences Module

2.16.

Based on the teacher’s assessment of David’s strengths and weaknesses, which of these
four subjects is he likely to be strongest in?

(A) English

(B) Art

(C) Chemistry

(D) Music

Answer C

To arrive at this answer, we need to pay close attention to the final two sentences: "David
finds it easy to draw logical conclusions, which helps him to excel in fact-based subjects like
maths and science. However, he finds it difficult to utilise his creativity." Since the text
specifically mentions his problems with creativity, we can immediately eliminate the two
"creative" subjects – art and music – from the list. This leaves us with A and C. Though the
text gives no indication of his ability in English – which means it could, hypothetically, be
very good – we are looking here for the most likely answer based on what we know. Since
chemistry is well-known as the more "logical" of the two remaining subjects, and since the
text explicitly mentions that David excels (= is strong in) scientific subjects, we can safely
choose C as the most probable option.

2.17.

Which of the following statements is/are correct according to the text?

I. David finds it easy to make use of his musical talent.

II. If David is stuck, he asks the teacher for help.

(A) Only statement (I) is correct.

(B) Only statement (II) is correct.

(C) Both statements are correct.

(D) Neither of the two statements is correct.

Answer B

© 2017 edulink GmbH www.edu-link.de ALL RIGHTS RESERVED 70


Buyer: Jan Hiller (janhiller@gmx.net)
Transaction ID: 8BE25485KM511652L

Humanities, Cultural Studies and


Social Sciences Module

Statement (I) is incorrect, since the last sentence of the text makes clear that David finds it
difficult to take advantage of his musical (=artistic, creative) talents.

Statement (II) is correct, since the excerpt states that David asks the teacher for help when
he is unable to make any further progress on a difficult question by himself.

Only statement (II) is correct, which means that B is the right answer.

2.18.

Which of the following statements is/are correct according to the text?

I. David is always quiet during German classes.

II. David knows exactly what the central nervous system is.

(A) Only statement (I) is correct.

(B) Only statement (II) is correct.

(C) Both statements are correct.

(D) Neither of the two statements is correct.

Answer A

Statement (I) is correct, since the text states that David is always quiet during German
lessons but tends to chat with his fellow pupils during natural history.

Statement (II) is incorrect. The text makes clear that David is especially interested in the
central nervous system. However, the text does not say that David really knows what the
central nervous system is.

Only statement (I) is correct, which means that A is the right answer.

2.19.

Which of the following statements is/are correct according to the text?

I. David usually skims a text before reading the instructions that go with it.

© 2017 edulink GmbH www.edu-link.de ALL RIGHTS RESERVED 71


Buyer: Jan Hiller (janhiller@gmx.net)
Transaction ID: 8BE25485KM511652L

Humanities, Cultural Studies and


Social Sciences Module

II. David prefers to see and hear things in order to remember them.

(A) Only statement (I) is correct.

(B) Only statement (II) is correct.

(C) Both statements are correct.

(D) Neither of the two statements is correct.

Answer D

Statement (I) is incorrect, since the text states that David first looks at the instructions and
then skims the text.

Statement (II) is also incorrect, because David tends especially to use visual means to help
him remember things. No mention is made of hearing things.

Both statements are incorrect, which means that D is correct.

"MORALS IN BUSINESS" (EXERCISES 2.20.-2.22.)

Time and again, we hear of the illegal and immoral acts committed by individuals within
companies and between companies themselves. This raises questions regarding the
motives for such acts. We must question whether the market economy promotes moral
values, whether it creates incentives for immoral behaviour, whether it drives immoral
behaviour in favour of profit maximisation and to the detriment of ethics, whether it is
possible to curb this behaviour in companies using internally prescribed moral codes and
whether companies have an interest in adhering to their own moral codes or will always
decide in favour of profit when faced with a moral dilemma.

2.20.

Which of the following statements is/are correct according to the text?

I. Banks are denounced as being especially immoral.

II. The market economy promotes moral values.

(A) Only statement (I) is correct.

(B) Only statement (II) is correct.

© 2017 edulink GmbH www.edu-link.de ALL RIGHTS RESERVED 72


Buyer: Jan Hiller (janhiller@gmx.net)
Transaction ID: 8BE25485KM511652L

Humanities, Cultural Studies and


Social Sciences Module

(C) Both statements are correct.

(D) Neither of the two statements is correct.

Answer D

Statement (I) is incorrect, since banks are not the main subject of discussion. Statement (II)
is also incorrect, since the text explicitly states that it is questionable whether the market
economy promotes moral values.

Both statements are incorrect according to the text, which means that Answer D is correct.

2.21.

Which of the following statements is/are correct according to the text?

I. There are no obvious approaches to solving the issue mentioned in the text.

II. The problem relates only to dealings between companies.

(A) Only statement (I) is correct.

(B) Only statement (II) is correct.

(C) Both statements are correct.

(D) Neither of the two statements is correct.

Answer A

Statement (I) is correct, since the text highlights the issues but does not formulate any
concrete suggestions for solving them. Statement (II) is incorrect, because the text also
addresses the actions of employees within a company.

Only statement (I) is correct, which means that A is correct.

© 2017 edulink GmbH www.edu-link.de ALL RIGHTS RESERVED 73


Buyer: Jan Hiller (janhiller@gmx.net)
Transaction ID: 8BE25485KM511652L

Humanities, Cultural Studies and


Social Sciences Module

2.22.

Which of the following statements is/are correct according to the text?

I. The maximisation of profit takes top priority.

II. It has not yet been clarified whether or not companies care about their employees
behaving morally when faced with a choice between morals and profit.

(A) Only statement (I) is correct.

(B) Only statement (II) is correct.

(C) Both statements are correct.

(D) Neither of the two statements is correct.

Answer B

Statement (I) is incorrect, since it has not yet been clarified whether profit maximisation or
morals are more important for businesses. Statement (II) is correct, since it has not yet been
clarified whether businesses care more about the moral behaviour of their employees or
about making a profit.

Only statement II can be seen in the text. This means that Answer B is correct.

© 2017 edulink GmbH www.edu-link.de ALL RIGHTS RESERVED 74


Buyer: Jan Hiller (janhiller@gmx.net)
Transaction ID: 8BE25485KM511652L

Humanities, Cultural Studies and


Social Sciences Module

3 USING REPRESENTATION SYSTEMS FLEXIBLY

The test contains 22 questions and is designed to start off easy and become progressively
more difficult. With 55 minutes to answer all the questions, you can afford to spend an
average of 2.5 minutes on each question. However, average is the operative word here,
since the harder questions at the end of the test are likely to require slightly longer than the
easier questions at the beginning. Keep this in mind when you’re planning your time!

As with the rest of the test, you won’t lose marks for wrong answers, so if you find yourself
stuck on a particular question, you should make your best guess at the answer and move
on. If you have time, you can come back to it at the end.

There are two types of questions in this section of the test.

In some questions, you’ll see a short text and then a diagram that represents the information
contained in the text. Usually (but not always), some of the information from the text will be
correctly represented, and some will be incorrectly represented. Your job will be to read the
text, look at the diagram and judge which relationships the diagram gets right and which
it gets wrong.

In other questions, you’ll see a short text and a choice of two or more diagrams. Your job
will be to decide which diagram correctly represents the information from the text.

The test will contain sets of up to three questions on the same topic. You may sometimes
need to use information from question 1 to answer questions 2 or 3.

3.1 WHAT AM I BEING TESTED ON?

This section of the test is measuring a skill called inductive reasoning. This is just the
technical term for something most of us do every day – drawing conclusions about what
would be likely to happen in a specific situation or case based on other comparable
situations or cases we’ve seen or heard about.

This section is also testing your ability to understand information in both diagrammatic and
written form.

Don’t be put off by the title of the section, which makes it sound more confusing than it is.
Essentially, all this section is really asking for is the use of logic. As long as we keep our
cool, that’s something all of us are capable of!

© 2017 edulink GmbH www.edu-link.de ALL RIGHTS RESERVED 75


Buyer: Jan Hiller (janhiller@gmx.net)
Transaction ID: 8BE25485KM511652L

Humanities, Cultural Studies and


Social Sciences Module

3.2 THINGS TO KEEP IN MIND

As with other sections of this test, all the knowledge you need will be contained within the
question. You will never need to use any of your own knowledge to answer a question. In
other words, if you come across a topic you’re not familiar with, don’t panic! Keep your cool
and read the text slowly and calmly, underlining key words if you think it will help. Likewise, if
you come across unfamiliar vocabulary, don’t automatically assume that you won’t be able to
answer the question. Focus on what you can understand and try to come to terms with the
basic ideas the text is conveying. Don’t get bogged down with names and figures – nobody
is asking you to memorise the information and repeat it back!

A handy tip here is to always finish reading the text before you move on to look at the
diagram(s). While studying the two simultaneously might seem like a good way to save time,
you’re more likely to end up confusing yourself. If you feel like there’s simply too much
information to process in a question, do not panic! The questions are not designed to trick
you. Some of them might just require you consider more relationships than others - and
jotting down key facts and ideas on the question sheet or the notepaper provided can
really help with this.

Look out for more handy tips as you work through the practice questions and answers
below.

3.3 TYPES OF DIAGRAMS ENCOUNTERED IN THE EXAM

There are different types of visuals encountered in the exam. Let’s review these.

3.3.1 DEPENDENCIES

The first diagram type shows the overall structure of a group (such as a family, company)
and the relationships between the different parts within that group. The information is usually
shown in rectangles that are connected by straightlines. The lower rectangles are subgroups
of the connected upper rectangles.

EXAMPLE 1

Nowadays, we can differentiate between 9 species of honeybees. The most common bee
species worldwide is the western honeybee, Apis mellifera. This type, however, has

© 2017 edulink GmbH www.edu-link.de ALL RIGHTS RESERVED 76


Buyer: Jan Hiller (janhiller@gmx.net)
Transaction ID: 8BE25485KM511652L

Humanities, Cultural Studies and


Social Sciences Module

approximately 25 sub-species. The Apis mellifera mellifera, Apis mellifera iberica and Apis
mellifera major belong, for example, to the dark honeybees from Northern and Western
Europe. The Apis mellifera carnica, Apis mellifera ligustica, Apis mellifera cecopria as well as
the Apis mellifera sicula belong to the Carnica group more commonly found in Southern
Europe. The Apis mellifera syriaca, Apis mellifera meda, Apis mellifera anatolica and the
Apis mellifera caucasica are representatives of bees from the Middle East. Furthermore,
there are even more sub-species of the Apis mellifera.

These affiliations are displayed within the following illustration.

Apis mellifera

Dark honeybees Carnica group Bees from the Middle East

Apis mellifera iberica Apis mellifera carnica Apis mellifera anatolica


Apis mellifera ligustica Apis mellifera cecopria Apis mellifera caucasia
Apis mellifera major Apis mellifera sicula Apis mellifera syriaca

Which of the following two statements about this diagram is or are correct?

I. At least one of the Apis mellifera sub-species mentioned within the text is not found
within the graph.

II. According to the text, one of the sub-species found under the "Carnica group" field,
does not belong here.

(A) Only statement (I) is correct.

(B) Only statement (II) is correct.

(C) Both statements are correct.

(D) Neither of the two statements is correct.

ANSWER A

Only statement (I) is correct. Two of the species mentioned within the text are not found
within the graph. Furthermore, there is no wrong assignment in the "Carnica group" field.

© 2017 edulink GmbH www.edu-link.de ALL RIGHTS RESERVED 77


Buyer: Jan Hiller (janhiller@gmx.net)
Transaction ID: 8BE25485KM511652L

Humanities, Cultural Studies and


Social Sciences Module

3.3.2 CHRONOLOGICAL PROCESSES

Some of the diagrams in the test will show chronological processes (processes happening in
time order). The flow of events is represented simply by arrows, as shown in the example
below.

EXAMPLE 2

THE TUDOR SUCCESSION

In 1509, Henry VIII became King of England on the death of his father, Henry VII, the first
monarch of the Tudor dynasty. Henry VIII made Catherine of Aragon his Queen, and their
union resulted in the birth of his first child, Mary. In 1533, after having his first marriage
annulled, Henry married Anne Boleyn. His new Queen gave birth to a girl, but she was also
unable to produce a male heir to the throne. Elizabeth was Henry’s second daughter and
would later be known as one of England’s greatest sovereigns. In 1536, Henry had Anne
arrested on charges of adultery, and she was executed 4 days later. Henry’s dream of a
male heir was finally realized in 1537, when his next wife, Jane Seymour, gave birth to a
boy. Because of the rule of male succession Edward took precedence over his two half-
sisters, and at 9 years old he became the third monarch of the Tudor dynasty.

Which of the following diagrams correctly shows the Tudor succession of ruling monarchs?

© 2017 edulink GmbH www.edu-link.de ALL RIGHTS RESERVED 78


Buyer: Jan Hiller (janhiller@gmx.net)
Transaction ID: 8BE25485KM511652L

Humanities, Cultural Studies and


Social Sciences Module

(A)

Henry VII Henry VIII Edward Mary Elizabeth

(B)

Catherine of 
Henry VIII Anne Boleyn Mary Elizabeth Edward
Aragon

(C)

Henry VII Henry VIII Mary Elizabeth Edward

(D)

Catherine of 
Henry VII Henry VIII Anne Boleyn Edward Mary Elizabeth
Aragon

Answer A

Here, a good approach is to eliminate the answers that you know cannot be true. Henry VII
was the first Tudor monarch; therefore, B cannot be correct. Catherine of Aragon and Anne
Boleyn, as wives of King Henry VIII, both had the title "Queen" but did not rule as monarchs;
therefore, B and D are incorrect. C includes all the monarchs but the order of succession is
wrong, because the text clearly states that Edward ruled before his half-sisters. (Because of
the rule of male succession, Edward took precedence over his two half-sisters).

The exam may also feature some family tree-style diagrams such as the one shown below.

© 2017 edulink GmbH www.edu-link.de ALL RIGHTS RESERVED 79


Buyer: Jan Hiller (janhiller@gmx.net)
Transaction ID: 8BE25485KM511652L

Humanities, Cultural Studies and


Social Sciences Module

EXAMPLE 3

THE SUCCESSOR TO ELIZABETH I

Henry VII

Henry VIII Margaret

James V

Edward I Mary I Elizabeth I

Mary Queen 
of Scots

James I

When Henry VIII’s youngest daughter, Queen Elizabeth I, died, the question of succession to
the English throne became a pressing issue. Like her half-sister Mary and half-brother
Edward, Elizabeth had died childless. Descendants of Henry VII’s daughter, Margaret, had
strong claims to the English throne. Margaret had married James IV of Scotland, and their
son had been James V of Scotland. Mary Stuart became Queen of Scotland when she was
just six days old, following the death of her father, James V. Mary, Queen of Scots, as she
was known, had a son, also named James, who was king of Scotland at the time of
Elizabeth’s death. It was decided that James VI of Scotland would succeed Elizabeth and in
so doing become James I of England.

Which of the following two statements about this diagram is or are correct?

I. The diagram correctly shows the lines of succession from Henry VII to James I.

II. The diagram correctly shows the line of succession from Henry VII to Elizabeth I.

© 2017 edulink GmbH www.edu-link.de ALL RIGHTS RESERVED 80


Buyer: Jan Hiller (janhiller@gmx.net)
Transaction ID: 8BE25485KM511652L

Humanities, Cultural Studies and


Social Sciences Module

(A) Only statement (I) is correct.

(B) Only statement (II) is correct.

(C) Both statements are correct.

(D) Neither of the two statements is correct.

Answer C

The diagram shows both lines of succession correctly. Elizabeth was the last monarch
directly descended from Henry VIII and was the youngest of three children, including her
older half-brother and half-sister Edward and Mary. When James VI of Scotland succeeded
to the English throne, he became James I of England and was a direct descendant of Henry
VIII’s sister, Margaret. (His mother was Mary, Queen of Scots, whose father was James V,
whose mother was Margaret).

EXAMPLE 4

The Merovingian Dynasty was the first to rule France and their reign lasted nearly 300 years
(486-751).

Clovis I was the first king and ruled between 481 and 511. Childebert I was the son of Clovis
I and he became the next king, ruling from 511 to 558. While he did have children,
namedChrodoberge and Chrodesinde, his brother was first in line to the throne when he
died. Therefore, upon Childebert I’s death, Chlothar I was declared the next king, though he
occupied this post only for 3 years. Chlothar I is reputed to have had seven sons. Two of the
sons died before the father and never had the chance to reign. Among his remaining
children, Charibert I was able to position himself as king following the death of his father. He
was replaced by his half-brother Chilperic I after only 7 years. Chilperic I ruled as the King of
Soissons from 567 to 584. The son of Chilperic I, Chlothar II, then began his reign, first with
the help of his mother Fredegund until her death in 597 and afterwards by himself until 629.

Which of the following figures represent the ruling reign of Merovingian Dynasty?

© 2017 edulink GmbH www.edu-link.de ALL RIGHTS RESERVED 81


Buyer: Jan Hiller (janhiller@gmx.net)
Transaction ID: 8BE25485KM511652L

Humanities, Cultural Studies and


Social Sciences Module

(A) Clovis I Childebert I Clothar I Chilperic I Clothar II Charibert I

(B) Clovis I Childebert I Clothar II Charibert I Fredegvig

(C) Clovis I Childebert I Clothar I Charibert I Chilperic I Clothar II

(D) Clovis I Childebert I Clothar I Clothar II Charibert I Chilperic I

Answer C

If you’re having difficulty getting your head around this question, try making your own "line of
succession" diagram from the text. From this, you should be able to see that the third ruler
was Chlothar I; thus, only diagrams with this name as the third option can be correct. This
rules out Answer B. From your line of succession, you should also be able to see that the
fourth ruler was Charibert I, which means that only diagrams with this name as the fourth
option can be correct. This is the case in only one of the three remaining diagrams, Answer
C, thus Answer C must be correct.

TIP

Note that if you use the process of elimination described above, you don’t have to go
to the end of the line of succession to determine which is the correct answer.

3.3.3 POSITIVE AND NEGATIVE EFFECTS

On the diagrams in this test, relationships between different factors or variables are shown
using arrows. You’ll see "+" or "-" written over the arrow to indicate whether the relationship
is positive or negative. But what are positive and negative relationships? It’s very simple:

© 2017 edulink GmbH www.edu-link.de ALL RIGHTS RESERVED 82


Buyer: Jan Hiller (janhiller@gmx.net)
Transaction ID: 8BE25485KM511652L

Humanities, Cultural Studies and


Social Sciences Module

 A positive relationship is one where an increase in one variable (or factor) in a


relationship produces an increase in the other variable in the relationship. An
example of two variables in a positive relationship would be "number of hours spent
studying" (H) and "score on a test" (S). This could be represented in diagrammatic
form as follows:

Number of hours  +  Score on a test
spent studying

Alternatively:

H +  S

Note also that if the number of hours spent studying decreases, the score on the test
will also decrease.

 A negative relationship is one in which in an increase in one variable (or factor) in a


relationship produces a decrease in the other variable in the relationship. An example
of two variables in a negative relationship would be "number of hours spent studying"
(H) and "number of mistakes on the exam" (M). This could be represented in
diagrammatic form as follows:

Number of 
Number of hours  ‐  mistakes on the 
spent studying
exam

Alternatively:

H ‐  M

Note also that if the number of hours spent studying decreases, the number of
mistakes will increase.

Even if a text is describing more than one variable – which it frequently will be – you should
consider the diagram one relationship at a time and interpret it exactly as described above.
Try to get your head around this example:

If a student’s interest in a subject increases (I), their number of hours spent studying (H) will
also increase. This will lead to a decrease in their number of mistakes (M), which will lead to

© 2017 edulink GmbH www.edu-link.de ALL RIGHTS RESERVED 83


Buyer: Jan Hiller (janhiller@gmx.net)
Transaction ID: 8BE25485KM511652L

Humanities, Cultural Studies and


Social Sciences Module

an increase in their level of motivation (Mo), which will in turn lead to a further increase in the
number of hours spent studying:

Interest in the  +  Number of hours  ‐ Number of 


mistakes on the  ‐  Level of 
subject spent studying motivation
exam

I +  H ‐ M ‐  Mo

EXAMPLE

CLIMATE CHANGE 5

The Earth absorbs energy from the sun, which is necessary to warm the planet. Much of this
energy is reflected back into space, which helps to stop the planet from overheating. Carbon
dioxide (CO2) is released into the atmosphere as a result of the burning of fossil fuels, and is
known as a greenhouse gas. These types of gases linger in the Earth’s atmosphere and
prevent or slow down the escape of energy back into space. As a result, temperatures on
Earth are rising.

How can one show these effects in a diagram?

I. The burning of fossil fuels is connected to the release of CO2 using which is

connected to a build-up of greenhouse gases in the atmosphere using .

II. The build-up of greenhouse gases in the atmosphere is connected to a change in the

amount of energy escaping using which is connected to the temperatures on

Earth using .

(A) Only statement (I) is correct.

(B) Only statement (II) is correct.

(C) Both statements are correct.

(D) Neither of the two statements is correct.

© 2017 edulink GmbH www.edu-link.de ALL RIGHTS RESERVED 84


Buyer: Jan Hiller (janhiller@gmx.net)
Transaction ID: 8BE25485KM511652L

Humanities, Cultural Studies and


Social Sciences Module

Answer C

Statement (I) is correct, as the sentence "Carbon dioxide (CO2) is released into the
atmosphere as a result of the burning of fossil fuels, and is known as a greenhouse gas"
points to a positive relationship between the burning of fossil fuels and the release of CO2
into the atmosphere. The sentence "These types of gases linger in the Earth’s
atmosphere…" points to a positive relationship between the release of CO2 and a build-up of
greenhouse gases in the atmosphere. Both these relationships are described correctly in the
statement, thus the statement as a whole is correct.

Statement (II) is also correct, as a negative relationship between a build-up of greenhouse


gases in the atmosphere and the amount of energy escaping is given by the following two
sentences: "These types of gases linger in the Earth’s atmosphere and prevent or slow
down the escape of energy back into space. As a result, temperatures on Earth are rising."
The energy escaping is also connected to the temperatures on Earth in a negative
relationship. Be careful not to get confused by this. In a negative relationship, the dominant
(effect-causing) variable can be increasing or decreasing – the rule is simply that it always
affects the target variable in the opposite way (in this case, the less energy escaping, the
warmer the temperatures on Earth; if more energy was escaping, temperatures would be
cooler).

© 2017 edulink GmbH www.edu-link.de ALL RIGHTS RESERVED 85


Buyer: Jan Hiller (janhiller@gmx.net)
Transaction ID: 8BE25485KM511652L

Humanities, Cultural Studies and


Social Sciences Module

3.4 SUMMARY: TEN ESSENTIAL FACTS, TIPS AND TRICKS – READ THIS (AGAIN)
BEFORE THE EXAM

This section of the exam tests your ability to understand a piece of information (a
relationship between two variables) in both written and diagrammatic form.

A positive relationship (represented by “+”) is one in which an increase in one variable


also produces an increase in the other.

A negative relationship (represented by “-“) is one in which an increase in one variable


produces a decrease in the other.

To avoid confusion, always finish reading the text completely before moving on to look at
the diagram.

If you’re feeling overwhelmed by the amount of information being presented to you, don’t
panic – simply go back to the text and think about the relationships one at a time. Use
your note paper to get your thoughts in order!

The test contains 22 questions, which are designed to start easy and become
progressively harder. You’ll have 55 minutes to answer them.

Remember that the harder questions will probably take longer, so make sure you leave
enough time at the end of the test.

You won’t lose points for a wrong answer, so if you feel like you’re spending too long on
a question, you can guess an answer and come back to it at the end if time allows.

All the information you need to reach the correct answer will be provided in the question.

Though it might seem like it, the questions are not designed to trick you! Harder
questions simply describe a greater number of relationships than easier ones, and can
be answered in exactly the same way!

© 2017 edulink GmbH www.edu-link.de ALL RIGHTS RESERVED 86


Buyer: Jan Hiller (janhiller@gmx.net)
Transaction ID: 8BE25485KM511652L

Humanities, Cultural Studies and


Social Sciences Module

3.5 PRACTICE QUESTIONS

You have a total of 55 minutes during the TestAS exam for the 22 questions belonging to
the "Using Representation Systems Flexibly" section.

3.5.1 EXAM

1.1.

WINE AND FOOD MATCHING

Although, ultimately, the matching of food with wine will always be a matter of personal taste,
certain wine varieties can be matched with particular types of food to enhance the dining
experience. Riesling is a slightly sweet, white wine that is best served with white meat such
as fish and chicken. Sauvignon Blanc grapes tend to grow best in warm climates, and the
resulting wine makes a great match for seafood and salads. If you are looking for a wine to
pair with red meat such as steak or wild game, Syrah is a great choice.

The following diagram is intended to show the relationships described above. Relationships
are shown by connecting lines.

Types of Wine

Riesling Sauvignon Blanc Syrah

Chicken Steak Salads Seafood Wild Game Fish

Which of the following statements is or are correct?

I. The Riesling food-wine pairing is shown correctly.

II. The Sauvignon Blanc food-wine pairing is shown correctly.

(A) Only statement (I) is correct.

(B) Only statement (II) is correct.

© 2017 edulink GmbH www.edu-link.de ALL RIGHTS RESERVED 87


Buyer: Jan Hiller (janhiller@gmx.net)
Transaction ID: 8BE25485KM511652L

Humanities, Cultural Studies and


Social Sciences Module

(C) Both statements are correct.

(D) Neither of the two statements is correct.

1.2.

Cabernet Sauvignon is widely considered to be one of the world’s greatest grape varieties. It
grows best in moderately warm, semi-arid regions; these areas allow for a long growing
season, which increases the quality of the grapes. However, the higher the quality of
Cabernet Sauvignon grapes, the lower the quantity of grapes harvested. Vintages that
experience extreme temperatures, too little sunlight or early harvesting are often found to
have a less fruity and more vegetative character. Some of the best Cabernet Sauvignon
wines come from France’s Médoc region and are considered amongst the finest of the great
Bordeaux wines. The wine is best enjoyed with simple red meat dishes. Because of the long
growing season, Cabernet Sauvignon grape and wine producers obtain only one harvest per
year, even if weather conditions are optimal.

The following diagram is intended to show the effects described in the text.

Moderately warm,  +  Length of the  +  Quality of 


Cabernet  ‐  Quantity of the 
semi arid regions growing season grapes harvested
Sauvignon grapes

Which of the following statements is or are correct?

I. The diagram correctly shows the effects that a moderately warm, semi-arid climate,
and a long growing season have on Cabernet Sauvignon grapes in Bordeaux.

II. The diagram correctly shows the suitability of matching a Cabernet Sauvignon wine
from Bordeaux with simply prepared red meat.

(A) Only statement (I) is correct.

(B) Only statement (II) is correct.

(C) Both statements are correct.

(D) Neither of the two statements is correct.

© 2017 edulink GmbH www.edu-link.de ALL RIGHTS RESERVED 88


Buyer: Jan Hiller (janhiller@gmx.net)
Transaction ID: 8BE25485KM511652L

Humanities, Cultural Studies and


Social Sciences Module

1.3.

TYPES OF CROCODILIAN

Crocodilians (or Crocodylia) are a group of reptiles that includes crocodiles, alligators,
caiman and gharials. There are 23 species of Crocodylia, which are divided into three
families. The family that contains the most Crocodilian species is the Crocodylidae.
Members of this family are referred to as "true crocodiles", and their longer heads and v-
shaped snouts differentiate them from other Crocodilians. "True crocodiles" account for 14
species of Crocodilyae, and include the American Crocodile, the Australian Freshwater
Crocodile, the Nile Crocodile and the Philippine Crocodile. Another grouping of Crocodylia,
known as the Alligatoridae, is found predominantly in North and South America. Family
members include the American Alligator, the Spectacled Caiman, the Black Caiman and, the
only species located outside America, the Chinese Alligator. The remaining family of
Crocodylia, the Gavialidae, contains a single species: the gharial, which is characterized by
its very thin snout.

(A) Crocodylidae Crocodilian Crocodylia

American  Australian Nile Philippine American  Chinese Black  Spectacled 


Gavialidae
Caiman Caiman

(B) Crocodylia
Crocodylidae Alligatoridae

American  Chinese Nile Philippine American  Australian Black  Spectacled  Gharial Gavialidae
Caiman Caiman

(C) Gavialidae
Crocodylia Alligatoridae

American  Chinese Nile Philippine American  Australian Black  Spectacled  Gharial


Caiman Caiman

(D) Crocodylidae Alligatoridae Gavialidae

American  Australian Nile Philippine American  Chinese Black  Spectacled 


Caiman Caiman Gharial

© 2017 edulink GmbH www.edu-link.de ALL RIGHTS RESERVED 89


Buyer: Jan Hiller (janhiller@gmx.net)
Transaction ID: 8BE25485KM511652L

Humanities, Cultural Studies and


Social Sciences Module

1.4.

ANCIENT CHINA

In history books, the pre-imperial period in China is referred to as "Ancient China". It can be
divided into the Xia Dynasty, the Shang Dynasty, and the Zhou Dynasty. Yu the Great, the
first dynastic ruler of China, and his son, Qi of Xia, ruled during the Xia Dynasty. Tang was
the first king of the Shang dynasty; 600 years later, Di Xin became the dynasty’s last ruler.
The Zhou lineage began with Zhou Wu Wang and ended with King Nan.

The following diagram is intended to show the affiliations described above. Affiliations are
shown using connecting lines:

Ancient Chinese Dynasties

Xia Dynasty Shang  Dynasty Zhou Dynasty

Yu the Great Tang Di Xin Qi of Xia Zhou Wu Wong King Nan

Which of the following statements is or are correct?

I. The Xia Dynasty is shown correctly.

II. The Zhou Dynasty is shown correctly.

(A) Only statement (I) is correct.

(B) Only statement (II) is correct.

(C) Both statements are correct.

(D) Neither of the two statements is correct.

© 2017 edulink GmbH www.edu-link.de ALL RIGHTS RESERVED 90


Buyer: Jan Hiller (janhiller@gmx.net)
Transaction ID: 8BE25485KM511652L

Humanities, Cultural Studies and


Social Sciences Module

1.5.

ECONOMIC DEVELOPMENT

Chinese economic development has led to an increase in job opportunities in the


manufacturing industry, which has caused millions of the country’s young workers to migrate
to urban areas. The economic growth in China has resulted in a huge increase in the
number and size of urban centers and a vast reduction in the size of the country’s rural
population.

How can one show these effects in a diagram?

I. Chinese economic development is connected to both an increase in job opportunities

using and is connected to migration to urban areas using .

II. The economic growth in China is connected to both an increase in the number and

size of urban centers using and is connected to a change in the size of the

country’s rural population using .

(A) Only statement (I) is correct.

(B) Only statement (II) is correct.

(C) Both statements are correct.

(D) Neither of the two statements is correct.

1.6.

The director of the Berlin Philharmonic wants to try a new technique for setting up the
orchestra for their premiere of Gustav Mahler´s Symphony 5. He decides that he wants to
put together three groups on the stage, each consisting of some string players, a wind
section and some percussionists. Percussion instruments include rainstick, tambourine and
cymbal. String players include cello, guitar and violin. The wind section includes trumpet,
saxophone and flute.

The following diagram is intended to show the new stage layout:

© 2017 edulink GmbH www.edu-link.de ALL RIGHTS RESERVED 91


Buyer: Jan Hiller (janhiller@gmx.net)
Transaction ID: 8BE25485KM511652L

Humanities, Cultural Studies and


Social Sciences Module

Berlin Philharmonic

Group 1 Group 2 Group 3

Cello Guitar Rainstick Trumpet Saxophone Tambourine Violin Flute Cymbal

Which of the following statements is or are correct?

I. The setup of group 1 could be shown correctly.

II. The setup of group 3 could be shown correctly.

(A) Only statement (I) is correct.

(B) Only statement (II) is correct.

(C) Both statements are correct.

(D) Neither of the two statements is correct.

1.7.

According to a recent study published in an American medical magazine, the younger a


consumer of cannabis is, the greater the apparent down-regulation of cannabinoid receptors
in the brain through tetrahydrocannabinol (THC). This means consumers are more likely to
get addicted. The study claims that this leads to lower probability of gaining a high school
degree and a higher suicide rate among the youngsters concerned. In addition, the risk of
trying heroin is also higher.

The following diagram is intended to show the described effects:

© 2017 edulink GmbH www.edu-link.de ALL RIGHTS RESERVED 92


Buyer: Jan Hiller (janhiller@gmx.net)
Transaction ID: 8BE25485KM511652L

Humanities, Cultural Studies and


Social Sciences Module

Chance for 
high school 
degree

‐ + Likelihood of  +  Likelihood of 
Influence of THC 
Age becoming  committing 
on brain
addicted suicide

Likelihood of 
trying 
heroine

Which of the following statements is or are correct?

I. The diagram correctly shows the effect of age on the influence of


tetrahydrocannabinol on the cannabinoid receptors.

II. The diagram correctly shows the effect of cannabis addiction on the chance of
receiving a high school degree.

(A) Only statement (I) is correct.

(B) Only statement (II) is correct.

(C) Both statements are correct.

(D) Neither of the two statements is correct.

1.8.

THE GREAT KHANS

Genghis Khan founded the Mongol Empire and divided it between his four sons before he
died. Before his death in 1227, the Great Khan declared that his third son, Ögedei, should
succeed him, and Ögedei’s two older brothers, Jochi and Chagatai, were passed over.
Under this second Great Khan, the territory of the Mongol Empire reached its farthest extent
west and south. Güyük became the next Great Khan on the death of his father, in 1246, but
he ruled for less than two years. Möngke was the eldest son of Genghis Khan’s youngest
son Tolui, and succeeded his cousin as the next Great Khan. Möngke was in the process of
conquering Song China when he died in 1259. He was succeeded by his brother Kublai, who
united conquered Chinese territory and became founder and Emperor of the Yuan Dynasty.

© 2017 edulink GmbH www.edu-link.de ALL RIGHTS RESERVED 93


Buyer: Jan Hiller (janhiller@gmx.net)
Transaction ID: 8BE25485KM511652L

Humanities, Cultural Studies and


Social Sciences Module

Which of the following diagrams correctly shows the Great Khan succession?

(A) Genghis Jochi Chagatai Ogedei Guyuk Mongke Kublai

(B) Genghis Ogedei Guyuk Tolui Mongke Kublai

(C) Genghis Ogedei Guyuk Mongke Kublai

(D) Genghis Guyuk Ogedei Mongke Kublai

1.9.

GENGHIS KHAN FAMILY TREE

Use the information supplied in question 9 - The Great Khans - to help you answer the
following:

© 2017 edulink GmbH www.edu-link.de ALL RIGHTS RESERVED 94


Buyer: Jan Hiller (janhiller@gmx.net)
Transaction ID: 8BE25485KM511652L

Humanities, Cultural Studies and


Social Sciences Module

Genghis Khan

Jochi Chagatai Ogedei Tolui

Guyuk Mongke Kublai

Which of the following statements is or are correct?

I. The diagram correctly shows the affiliations between Tolui and his sons.

II. The diagram correctly shows the affiliations between Genghis Khan and his sons.

(A) Only statement (I) is correct.

(B) Only statement (II) is correct.

(C) Both statements are correct.

(D) Neither of the two statements is correct.

1.10.

EATING BLUEBERRIES

According to research, health conscious people eat more blueberries than those who do not
take so much care of their bodies. Blueberries contain high levels of a chemical called
anthocyanin. It is believed that anthocyanin is particularly beneficial for the heart because it
lowers blood pressure and makes blood vessels more elastic, reducing the risk of heart
attack. Because of the higher level of anthocyanin, eating lots of blueberries also leads to a
higher level of fitness in general.

© 2017 edulink GmbH www.edu-link.de ALL RIGHTS RESERVED 95


Buyer: Jan Hiller (janhiller@gmx.net)
Transaction ID: 8BE25485KM511652L

Humanities, Cultural Studies and


Social Sciences Module

Health 
conscious

Number of 
Level of  +  Blueberries 
+  Levels of  +  Risk of heart 
fitness anthocyanin attack
eaten


Level of 
income

Which of the following statements is or are correct?

I. The diagram correctly shows the effects of fitness on the number of blueberries
eaten.

II. The diagram correctly shows the effects high-levels of anthocyanin have on the risk
of heart attack.

(A) Only statement (I) is correct.

(B) Only statement (II) is correct.

(C) Both statements are correct.

(D) Neither of the two statements is correct.

1.11.

URBANIZATION

Many Asian countries are experiencing a period of economic growth and development. This
economic growth and development has led to an increase in employment opportunities. The
Asian city populations have increased in response to the rise in employment opportunities.
As city populations increased, there was a corresponding need for new land to build housing
and services on in order to meet the needs of the new inhabitants. This kind of growth is
known as urbanization. This urbanization impacts the environment by leading to a loss of
natural habitats and a rise in air and water pollution.

© 2017 edulink GmbH www.edu-link.de ALL RIGHTS RESERVED 96


Buyer: Jan Hiller (janhiller@gmx.net)
Transaction ID: 8BE25485KM511652L

Humanities, Cultural Studies and


Social Sciences Module

Number of 
natural habitats
‐ 

Economic growth & 
+ Employment 
+ +
City Population Urbanization
Development Opportunities

Air & water 
pollution levels

Which of the following statements is or are correct?

I. The diagram correctly shows the effects of employment opportunities on city


populations.

II. The diagram correctly shows the effects of urbanization on natural habitats and
pollution levels.

(A) Only statement (I) is correct.

(B) Only statement (II) is correct.

(C) Both statements are correct.

(D) Neither of the two statements is correct.

1.12.

MANAGEMENT PLAN

The Water Quality and Salinity Management Plan aims to use water from the Murray-Darling
Basin River system in a more sustainable way. Limiting the amount of water extracted from
its rivers will benefit the area’s wetlands and the ecosystems that depend on them. Allowing
the rivers to flow naturally will reduce the build-up of sediment and help improve water
quality.

How can one show these effects in a diagram?

I. The amount of water extracted is connected to benefits for the area’s wetlands and

their ecosystems using .

II. Allowing the rivers to flow naturally is connected to a reduction in the build-up of

sediment using .

© 2017 edulink GmbH www.edu-link.de ALL RIGHTS RESERVED 97


Buyer: Jan Hiller (janhiller@gmx.net)
Transaction ID: 8BE25485KM511652L

Humanities, Cultural Studies and


Social Sciences Module

(A) Only statement (I) is correct.

(B) Only statement (II) is correct.

(C) Both statements are correct.

(D) Neither of the two statements is correct.

1.13.

MURRAY-DARLING BASIN

Water extraction severely reduces the flow of the Murray-Darling Basin River system. This
causes the area’s wetlands to dry up, which creates favorable conditions for blue-green
algae to flourish. These algae-like bacteria have a potential for toxicity and are a serious
threat to water quality. The diversion of the natural flow of the rivers causes sediment to
build up in areas that are permanently flooded. Sedimentation reduces the movement of the
water and results in high salinity levels and a deterioration in water quality.

Driving‐up of 
+ Blue – Green algae 
wetland areas blooms +

Water Quality


Sedimentation Water salinity levels

Which of the following statements is or are correct?

I. The diagram correctly shows the effects of blue-green algae blooms on water quality.

II. The diagram correctly shows the effects of sedimentation on water salinity levels.

(A) Only statement (I) is correct.

(B) Only statement (II) is correct.

© 2017 edulink GmbH www.edu-link.de ALL RIGHTS RESERVED 98


Buyer: Jan Hiller (janhiller@gmx.net)
Transaction ID: 8BE25485KM511652L

Humanities, Cultural Studies and


Social Sciences Module

(C) Both statements are correct.

(D) Neither of the two statements is correct.

1.14.

DEFORESTATION AND GLOBAL WARMING

Deforestation for agricultural and logging purposes is threatening much of the earth’s
remaining tropical rainforest. Large forested areas play a vital role in regulating the world’s
climate patterns by acting as a carbon sink. Rainforests soak up carbon dioxide (CO2) that
would otherwise enter the atmosphere and convert it to oxygen. Increased levels of CO2 in
the atmosphere are linked to long-term trends in global warming (rising global temperatures).

Agriculture

Tropical  ‐ Levels of CO2 in  ‐ Global 


rainforests the atmosphere  temperatures


Logging

Which of the following statements is or are correct?

I. The diagram does not show the effects that agriculture and logging are having on
tropical rainforests correctly.

II. The diagram correctly shows the effects that levels of CO2 are having on global
temperatures.

(A) Only statement (I) is correct.

(B) Only statement (II) is correct.

(C) Both statements are correct.

(D) Neither of the two statements is correct.

© 2017 edulink GmbH www.edu-link.de ALL RIGHTS RESERVED 99


Buyer: Jan Hiller (janhiller@gmx.net)
Transaction ID: 8BE25485KM511652L

Humanities, Cultural Studies and


Social Sciences Module

1.15.

Refer back to the information in Exercise 14 to help you answer this exercise.

Agroforestry is an approach to agriculture and logging that looks to encourage long-term


production of wood through the sustainable management of tropical rainforests. Sustainable
farming practices – such as the simultaneous growing of crops and planting of trees and the
selective felling of trees – allow areas of rainforest to reach full maturity. Mature rainforests
are far more effective at reducing levels of CO2 in the atmosphere than areas that have not
been used in a sustainable way. Lower levels of CO2 in the atmosphere will help slow down
the rate of global warming.

Which of the following statements is or are correct?

I. The diagram correctly shows the effects that agriculture and logging are having on
tropical rainforests (negative arrow), which is in turn linked to levels of CO2 in the

atmosphere using (negative arrow).

II. The levels of CO2 in the atmosphere are connected to the rate of global warming

using , where the positive arrow points to the rate of global warming.

(A) Only statement (I) is correct.

(B) Only statement (II) is correct.

(C) Both statements are correct.

(D) Neither of the two statements is correct.

1.16.

JAPAN’S AGEING POPULATION

Because the average age in Japan (A) is steadily rising, there is an increase in the demand
(D) for nursing staff to help look after the country’s ageing population. Due to the rise in the
demand for nursing staff, nurses’ pay (P) started to rise in order to encourage more young
people to take-up a career in the healthcare industry. This policy was a success, resulting in
a twofold increase in the number of students (S) taking healthcare related courses, and a
similar surge in the number of graduates entering the nursing profession (N). The growth in
nurses’ pay (P), however, has begun to slow down as a result of the increase in the supply
of nursing staff.

© 2017 edulink GmbH www.edu-link.de ALL RIGHTS RESERVED 100


Buyer: Jan Hiller (janhiller@gmx.net)
Transaction ID: 8BE25485KM511652L

Humanities, Cultural Studies and


Social Sciences Module

A
+  D
+  P

+  + 

N
+  S

Which of the following statements is or are correct?

I. The diagram correctly shows the effects of an ageing population on the demand for
nursing staff.

II. The diagram correctly shows the effects the supply of newly recruited nurses is
having on pay.

(A) Only statement (I) is correct.

(B) Only statement (II) is correct.

(C) Both statements are correct.

(D) Neither of the two statements is correct.

1.17.

LIFE EXPECTANCY IN JAPAN

Japan has the highest life expectancy in the world. Higher life expectancy has caused an
increase in the country’s dependency ratio: the number of people not of working age to those
who are economically active. The growth in the dependency ratio is causing fiscal problems
for the government because lower tax revenue negatively impacts government spending.

Life Expectancy
+  Dependency Ratio
+  Tax Revenue
+  Government Spending

Which of the following statements is or are correct?

I. The diagram correctly shows the effects of life expectancy on the dependency ratio.

II. The diagram correctly shows the effects of the dependency ratio on tax revenue and
the effects of tax revenue on government spending.

© 2017 edulink GmbH www.edu-link.de ALL RIGHTS RESERVED 101


Buyer: Jan Hiller (janhiller@gmx.net)
Transaction ID: 8BE25485KM511652L

Humanities, Cultural Studies and


Social Sciences Module

(A) Only statement (I) is correct.

(B) Only statement (II) is correct.

(C) Both statements are correct.

(D) Neither of the two statements is correct.

1.18.

LOWERING MORTALITY RATES IN JAPAN

Life expectancy in Japan has benefitted from a reduction in salt content in the Japanese diet,
which has helped to remove stroke as the country’s leading cause of death. Improvements
to the country’s healthcare system and raised public awareness of health issues have also
contributed to the downward trend in stroke mortality rates. Naturally, a lower death rate
from stroke has also resulted in a higher life expectancy in general.

How can one show these effects in a diagram?

I. The amount of salt in the Japanese diet is connected to a change in the death rate

from stroke using .

II. Improvements to the country’s healthcare system and raised public awareness of

health issues are both connected to a change in the death rate from stroke using .

The change in the death rate from stroke is also connected to life expectancy using

(A) Only statement (I) is correct.

(B) Only statement (II) is correct.

(C) Both statements are correct.

(D) Neither of the two statements is correct.

1.19.

According to the German tale, the "Nibelungen", Siegmund zu Xanten, grandson of Odin,
has a child named Siegfried with his wife Sieglinde. Siegfried is called the "Dragon Slayer”
and he is the main character of the medieval German saga. Siegfried's wife Krimhild is

© 2017 edulink GmbH www.edu-link.de ALL RIGHTS RESERVED 102


Buyer: Jan Hiller (janhiller@gmx.net)
Transaction ID: 8BE25485KM511652L

Humanities, Cultural Studies and


Social Sciences Module

Dankrat's and Uta’s daughter and also the niece of Hagen von Tronje. Siegfried and Krimhild
are the parents of Gunther zu Worms; however, Krimhild also has a son with Etzel, named
Ortlieb. Etzel has a daughter, Brynhild, with Helche.

The following diagram is intended to show the family affiliations described:

Odin

Siegt Wels

Siegmund zu 
Sieglinde Dankrat Hagen Uta
Xanten

Siegfried Krimhild Etzel Helche

Gunther zu 
Ortlieb Brynhild
Worms

Which of the following two statements about this diagram is or are correct?

I. The affiliation between Gunther zu Worms, Ortlieb, and Brynhild is shown correctly.

II. The affiliation between Ortlieb and Hagen is shown correctly.

(A) Only statement (I) is correct.

(B) Only statement (II) is correct.

(C) Both statements are correct.

(D) Neither of the two statements is correct.

© 2017 edulink GmbH www.edu-link.de ALL RIGHTS RESERVED 103


Buyer: Jan Hiller (janhiller@gmx.net)
Transaction ID: 8BE25485KM511652L

Humanities, Cultural Studies and


Social Sciences Module

1.20.

CLOSED ECOLOGICAL SYSTEM

In a closed ecological system, natural elements interact to produce a flow of nutrients that
remain in the ecosystem as they are recycled within the environment. Plants introduce
nutrients – chemical elements – into the soil through the biomass they produce. As the
biomass decomposes, microorganisms in the soil break down the plant matter and release
nutrients that help maintain the health – or rather, the quality of the soil. The higher the
nutrient content of the soil, the higher the soil quality. Healthy soil is essential for sustaining
plant growth, producing biomass and keeping the nutrient cycle running smoothly.

Nutrient content of 
soil
+ ‐

+
Plant biomass Soil Quality

Which of the following two statements about this diagram is or are correct?

I. The diagram correctly shows the effects of plant biomass on the nutrient content of
soil.

II. The diagram correctly shows the effects of the nutrient content of soil on soil quality.

(A) Only statement (I) is correct.

(B) Only statement (II) is correct.

(C) Both statements are correct.

(D) Neither of the two statements is correct.

1.21.

See question 20 for additional information.

© 2017 edulink GmbH www.edu-link.de ALL RIGHTS RESERVED 104


Buyer: Jan Hiller (janhiller@gmx.net)
Transaction ID: 8BE25485KM511652L

Humanities, Cultural Studies and


Social Sciences Module

OPEN ECOLOGICAL SYSTEM

In an open ecological system, a large proportion of the energy and nutrients produced does
not remain inside the system. Instead it is harvested in the form of fruit, grain, and
vegetables. The quality of the soil is adversely affected by this loss of nutrients from the
system. This means that fertilizers are required to prevent soil degradation and to ensure
that soil has a high nutrient content, which in turn leads a high soil quality.

+ +
Plant biomass  Nutrient content of 
Soil Quality
(decomposing) soil

What additions have to be made to the diagram to show the effects of an open ecological
system?

Pla = Plant biomass Harv = Harvest

Nut = Nutrient content of soil Fer = Fertilizer

Soil = Soil quality

Which of the following two statements about this diagram is or are correct?

I. Harv is positioned before Pla and is linked to it by .

II. Fer is positioned between Nut and Soil. One points from Nut to Fer, one
points from Fer to Soil.

(A) Only statement (I) is correct.

(B) Only statement (II) is correct.

(C) Both statements are correct.

(D) Neither of the two statements is correct.

1.22.

SUPER FOOD

Blueberries are often described as super food because of the many health benefits the fruit
provides. Blueberries contain high levels of antioxidant nutrients, which help limit the
damage caused by free radicals to cellular structures. Research has shown that a blueberry-

© 2017 edulink GmbH www.edu-link.de ALL RIGHTS RESERVED 105


Buyer: Jan Hiller (janhiller@gmx.net)
Transaction ID: 8BE25485KM511652L

Humanities, Cultural Studies and


Social Sciences Module

rich diet may protect the brain from oxidative stress, which helps to improve memory and
other cognitive functions in older people.

How can one show these effects in a diagram?

I. Eating blueberries is connected to high levels of antioxidant nutrients using and is

connected to the limiting of the damage caused to cells using . Both of these
arrows point to eating blueberries.

II. Eating blueberries is connected to a development of oxidative stress in the brain

using and is also connected to improved memory using . points to the

oxidative stress, and points to improved memory.

(A) Only statement (I) is correct.

(B) Only statement (II) is correct.

(C) Both statements are correct.

(D) Neither of the two statements is correct.

© 2017 edulink GmbH www.edu-link.de ALL RIGHTS RESERVED 106


Buyer: Jan Hiller (janhiller@gmx.net)
Transaction ID: 8BE25485KM511652L

Humanities, Cultural Studies and


Social Sciences Module

3.5.2 ANSWER KEY

Question Answer

1.1. B

1.2. A

1.3. D

1.4. B

1.5. D

1.6. B

1.7. A

1.8. C

1.9. B

1.10. D

1.11. C

1.12. C

1.13. D

1.14. A

1.15. B

1.16. A

1.17. A

1.18. C

1.19. D

1.20. A

1.21. C

1.22. B

© 2017 edulink GmbH www.edu-link.de ALL RIGHTS RESERVED 107


Buyer: Jan Hiller (janhiller@gmx.net)
Transaction ID: 8BE25485KM511652L

Humanities, Cultural Studies and


Social Sciences Module

3.5.3 DETAILED ANSWERS

1.1.

WINE AND FOOD MATCHING

Although, ultimately, the matching of food with wine will always be a matter of personal taste,
certain wine varieties can be matched with particular types of food to enhance the dining
experience. Riesling is a slightly sweet, white wine that is best served with white meat such
as fish and chicken. Sauvignon Blanc grapes tend to grow best in warm climates, and the
resulting wine makes a great match for seafood and salads. If you are looking for a wine to
pair with red meat such as steak or wild game, Syrah is a great choice.

The following diagram is intended to show the relationships described above. Relationships
are shown by connecting lines.

Types of Wine

Riesling Sauvignon Blanc Syrah

Chicken Steak Salads Seafood Wild Game Fish

Which of the following statements is or are correct?

I. The Riesling food-wine pairing is shown correctly.

II. The Sauvignon Blanc food-wine pairing is shown correctly.

(A) Only statement (I) is correct.

(B) Only statement (II) is correct.

(C) Both statements are correct.

(D) Neither of the two statements is correct.

© 2017 edulink GmbH www.edu-link.de ALL RIGHTS RESERVED 108


Buyer: Jan Hiller (janhiller@gmx.net)
Transaction ID: 8BE25485KM511652L

Humanities, Cultural Studies and


Social Sciences Module

Answer B

Consider the statements one at a time in order to reach the correct answer. Statement I is
incorrect, since the text states that Riesling wine is best served with fish and chicken, NOT
chicken and steak. Statement II, however, is correct, since the text mentions that Sauvignon
Blanc makes a great match with seafood and salads. This is represented correctly in the
diagram, which means that Statement II is correct. Statement I, however, is incorrect, since
the text states that Riesling wine is best served with fish and chicken, NOT chicken and
steak.

1.2.

Cabernet Sauvignon is widely considered to be one of the world’s greatest grape varieties. It
grows best in moderately warm, semi-arid regions; these areas allow for a long growing
season, which increases the quality of the grapes. However, the higher the quality of
Cabernet Sauvignon grapes, the lower the quantity of grapes harvested. Vintages that
experience extreme temperatures, too little sunlight or early harvesting are often found to
have a less fruity and more vegetative character. Some of the best Cabernet Sauvignon
wines come from France’s Médoc region and are considered amongst the finest of the great
Bordeaux wines. The wine is best enjoyed with simple red meat dishes. Because of the long
growing season, Cabernet Sauvignon grape and wine producers obtain only one harvest per
year, even if weather conditions are optimal.

The following diagram is intended to show the effects described in the text.

Moderately warm,  +  Length of the  +  Quality of 


Cabernet  ‐  Quantity of the 
semi arid regions growing season grapes harvested
Sauvignon grapes

Which of the following statements is or are correct?

I. The diagram correctly shows the effects that a moderately warm, semi-arid climate,
and a long growing season have on Cabernet Sauvignon grapes in Bordeaux.

II. The diagram correctly shows the suitability of matching a Cabernet Sauvignon wine
from Bordeaux with simply prepared red meat.

(A) Only statement (I) is correct.

(B) Only statement (II) is correct.

© 2017 edulink GmbH www.edu-link.de ALL RIGHTS RESERVED 109


Buyer: Jan Hiller (janhiller@gmx.net)
Transaction ID: 8BE25485KM511652L

Humanities, Cultural Studies and


Social Sciences Module

(C) Both statements are correct.

(D) Neither of the two statements is correct.

Answer A

Statement (I) is correct, as the sentences "It grows best in moderately warm, semi-arid
regions; these areas allow for a long growing season, which increases the quality of
Cabernet Sauvignon grapes" and "However, the higher the quality of Cabernet Sauvignon
grapes, the lower the quantity of grapes harvested" point to the positive effects that a
moderately warm, semi-arid region has on a long growing season and the cumulative effect
these conditions have on the quality and quantity of Cabernet Sauvignon grapes. This is
what the diagram shows. Hence, statement (I) is correct.

However, the information "The wine is best enjoyed with simple red meat dishes" (positive
relationship) is not represented in the diagram. Hence, statement (II) is false and A is the
correct answer.

1.3.

TYPES OF CROCODILIAN

Crocodilians (or Crocodylia) are a group of reptiles that includes crocodiles, alligators,
caiman and gharials. There are 23 species of Crocodylia, which are divided into three
families. The family that contains the most Crocodilian species is the Crocodylidae.
Members of this family are referred to as "true crocodiles", and their longer heads and v-
shaped snouts differentiate them from other Crocodilians. "True crocodiles" account for 14
species of Crocodilyae, and include the American Crocodile, the Australian Freshwater
Crocodile, the Nile Crocodile and the Philippine Crocodile. Another grouping of Crocodylia,
known as the Alligatoridae, is found predominantly in North and South America. Family
members include the American Alligator, the Spectacled Caiman, the Black Caiman and, the
only species located outside America, the Chinese Alligator. The remaining family of
Crocodylia, the Gavialidae, contains a single species: the gharial, which is characterized by
its very thin snout.

© 2017 edulink GmbH www.edu-link.de ALL RIGHTS RESERVED 110


Buyer: Jan Hiller (janhiller@gmx.net)
Transaction ID: 8BE25485KM511652L

Humanities, Cultural Studies and


Social Sciences Module

(A) Crocodylidae Crocodilian Crocodylia

American  Australian Nile Philippine American  Chinese Black  Spectacled 


Gavialidae
Caiman Caiman

(B) Crocodylia
Crocodylidae Alligatoridae

American  Chinese Nile Philippine American  Australian Black  Spectacled  Gharial Gavialidae
Caiman Caiman

(C) Gavialidae
Crocodylia Alligatoridae

American  Chinese Nile Philippine American  Australian Black  Spectacled  Gharial


Caiman Caiman

(D) Crocodylidae Alligatoridae Gavialidae

American  Australian Nile Philippine American  Chinese Black  Spectacled 


Caiman Caiman Gharial

Answer D

The text mentions three families of crocodilians, namely Crocodylidae, Alligatoridae and
Gavialidae. Then the respective species are described:

Australian Freshwater Crocodile, the Nile Crocodile and the Philippine Crocodile belong to
the Crocodylidae family.

American and Chinese Alligators, the Spectacled Caiman and the Black Caiman belong to
the Alligatoridae family.

The gharial belongs to the Gavialidae family.

TIPS

When presented with a text like this, you should begin by identifying which terms
refer to the superordinate categories and which refer to the subordinate categories.
Jotting down rough "family tree"-style diagrams (like the ones above) as you read
can help with this. A superordinate category is simply one which comes higher in a
system of classification. In this case, the superordinate categories are the three
families of Crocodilians (or Crocodylia) – the Crocodilyae, the Alligatoridae and the
Gavialidia – and the subordinate terms are the different species (or family members).

© 2017 edulink GmbH www.edu-link.de ALL RIGHTS RESERVED 111


Buyer: Jan Hiller (janhiller@gmx.net)
Transaction ID: 8BE25485KM511652L

Humanities, Cultural Studies and


Social Sciences Module

In addition, be sure to look carefully at the spellings – "Crocodilyae" should not be


confused with "Crocodylia".

Though this information might seem like a lot to take in, this kind of question can be
solved very easily by using a process of elimination. Pick ONE of the three families
described in the text (so the Crocodilyae, Alligatoridae OR the Gavialidia) and
visualize what would be written in the boxes for the family tree. Now, check the
possible answers (A, B, C and D) and disregard any answers where the family you’ve
picked is not represented correctly. Since ALL of the relationships described in the
text must be shown correctly in order for an answer to be correct, it’s really easy to
rule out incorrect answers without studying all the diagrams.

1.4.

ANCIENT CHINA

In history books, the pre-imperial period in China is referred to as "Ancient China". It can be
divided into the Xia Dynasty, the Shang Dynasty, and the Zhou Dynasty. Yu the Great, the
first dynastic ruler of China, and his son, Qi of Xia, ruled during the Xia Dynasty. Tang was
the first king of the Shang dynasty; 600 years later, Di Xin became the dynasty’s last ruler.
The Zhou lineage began with Zhou Wu Wang and ended with King Nan.

The following diagram is intended to show the affiliations described above. Affiliations are
shown using connecting lines:

Ancient Chinese Dynasties

Xia Dynasty Shang  Dynasty Zhou Dynasty

Yu the Great Tang Di Xin Qi of Xia Zhou Wu Wong King Nan

Which of the following statements is or are correct?

I. The Xia Dynasty is shown correctly.

© 2017 edulink GmbH www.edu-link.de ALL RIGHTS RESERVED 112


Buyer: Jan Hiller (janhiller@gmx.net)
Transaction ID: 8BE25485KM511652L

Humanities, Cultural Studies and


Social Sciences Module

II. The Zhou Dynasty is shown correctly.

(A) Only statement (I) is correct.

(B) Only statement (II) is correct.

(C) Both statements are correct.

(D) Neither of the two statements is correct.

Answer B

Statement (I) is incorrect. The Xia Dynasty is not shown correctly - although Yu the Great
and Qi of Xia belonged to this dynasty, Tang did not. Statement (II), however, is correct. The
Zhou Dynasty is shown correctly, since the text states that Zhou Wu Wang and King Nan
belonged to this dynasty.

1.5.

ECONOMIC DEVELOPMENT

Chinese economic development has led to an increase in job opportunities in the


manufacturing industry, which has caused millions of the country’s young workers to migrate
to urban areas. The economic growth in China has resulted in a huge increase in the
number and size of urban centers and a vast reduction in the size of the country’s rural
population.

How can one show these effects in a diagram?

I. Chinese economic development is connected to both an increase in job opportunities

using and is connected to migration to urban areas using .

II. The economic growth in China is connected to both an increase in the number and

size of urban centers using and is connected to a change in the size of the

country’s rural population using .

(A) Only statement (I) is correct.

(B) Only statement (II) is correct.

(C) Both statements are correct.

© 2017 edulink GmbH www.edu-link.de ALL RIGHTS RESERVED 113


Buyer: Jan Hiller (janhiller@gmx.net)
Transaction ID: 8BE25485KM511652L

Humanities, Cultural Studies and


Social Sciences Module

(D) Neither of the two statements is correct.

Answer D

Statement (I) is incorrect, as the sentence "Chinese economic development has led to an
increase in job opportunities in the manufacturing industry, which has caused millions of the
country’s young workers to migrate to urban areas" points to a positive relationship between
economic development and an increase in job opportunities and a positive relationship
between economic development and migration to urban areas. However, while the

statement correctly uses a to point to a positive relationship between economic

development and an increase in job opportunities, it incorrectly uses a to point to the


relationship between economic development and migration to urban areas.

Statement (II) is incorrect, as the sentence "Economic growth has resulted in a huge
increase in the number and size of urban centers and a vast reduction in the size of the
country’s rural population" points to a positive relationship between economic growth and the
increase in the number and size of urban centers, and a negative relationship between
economic growth and the size of the country’s rural population.

TIP

Once you have read the text, read statement (I) and make sure you understand the
relationship that is described between each of the variables in the statement. In
statement (I), the variable "Economic development (or economic growth)" is the
principle cause of change for both relationships (in mathematics, this is known as a
dominant variable). Thus, we can expand statement (I) as follows:

o Chinese economic development is connected to an increase in job


opportunities using (arrow with plus symbol)

o Chinese economic development is connected to migration to urban areas


using (arrow with minus symbol)

Both of these statements must be correct in order for Statement (I) to be correct
overall. As we saw from the explanation above, this is not the case.

Thus, the right way to approach this kind of question is to break EACH STATEMENT
down into each of the relationships it describes and identify which part (or parts)
is/are incorrect, if any. If you are having trouble understanding the relationships
between variables, you can visualize such relationships by drawing boxes, writing the
names of the variables inside and connecting the boxes with arrows. Next, look at the
variables you have identified and try to determine the relationship between them.

© 2017 edulink GmbH www.edu-link.de ALL RIGHTS RESERVED 114


Buyer: Jan Hiller (janhiller@gmx.net)
Transaction ID: 8BE25485KM511652L

Humanities, Cultural Studies and


Social Sciences Module

Does an increase in one cause a decrease in the other, or does an increase in one
also produce an increase in the other? Always remember that when an increase in
the dominant variable (in this example, Chinese economic development) leads to a
decrease in the dependent variable (in this example, job opportunities and migration
to urban areas), the relationship between them is represented by a "-".

1.6.

The director of the Berlin Philharmonic wants to try a new technique for setting up the
orchestra for their premiere of Gustav Mahler´s Symphony 5. He decides that he wants to
put together three groups on the stage, each consisting of some string players, a wind
section and some percussionists. Percussion instruments include rainstick, tambourine and
cymbal. String players include cello, guitar and violin. The wind section includes trumpet,
saxophone and flute.

The following diagram is intended to show the new stage layout:

Berlin Philharmonic

Group 1 Group 2 Group 3

Cello Guitar Rainstick Trumpet Saxophone Tambourine Violin Flute Cymbal

Which of the following statements is or are correct?

I. The setup of group 1 could be shown correctly.

II. The setup of group 3 could be shown correctly.

(A) Only statement (I) is correct.

(B) Only statement (II) is correct.

(C) Both statements are correct.

© 2017 edulink GmbH www.edu-link.de ALL RIGHTS RESERVED 115


Buyer: Jan Hiller (janhiller@gmx.net)
Transaction ID: 8BE25485KM511652L

Humanities, Cultural Studies and


Social Sciences Module

(D) Neither of the two statements is correct.

Answer B

The text states that each group must contain some percussion instruments, some wind
instruments and some percussion instruments. Thus, only statement (II) can be correct,
since Group 3 is the only group that consists of a string section (violins), a wind section
(flutes) and a percussion section (cymbal).

1.7.

According to a recent study published in an American medical magazine, the younger a


consumer of cannabis is, the greater the apparent down-regulation of cannabinoid receptors
in the brain through tetrahydrocannabinol (THC). This means consumers are more likely to
get addicted. The study claims that this leads to lower probability of gaining a high school
degree and a higher suicide rate among the youngsters concerned. In addition, the risk of
trying heroin is also higher.

The following diagram is intended to show the described effects:


Chance for 
high school 
degree

‐ + Likelihood of  +  Likelihood of 
Influence of THC 
Age becoming  committing 
on brain
addicted suicide

Likelihood of 
trying 
heroine

Which of the following statements is or are correct?

I. The diagram correctly shows the effect of age on the influence of


tetrahydrocannabinol on the cannabinoid receptors.

II. The diagram correctly shows the effect of cannabis addiction on the chance of
receiving a high school degree.

© 2017 edulink GmbH www.edu-link.de ALL RIGHTS RESERVED 116


Buyer: Jan Hiller (janhiller@gmx.net)
Transaction ID: 8BE25485KM511652L

Humanities, Cultural Studies and


Social Sciences Module

(A) Only statement (I) is correct.

(B) Only statement (II) is correct.

(C) Both statements are correct.

(D) Neither of the two statements is correct.

Answer A

The diagram shows that the lower the age of the smoker, the greater the effect of THC on
the brain (negative relationship). This corresponds with what is written in the text. Statement
(II) is incorrect, since addiction to cannabis lowers the smoker’s chance of receiving a high
school degree.

1.8.

THE GREAT KHANS

Genghis Khan founded the Mongol Empire and divided it between his four sons before he
died. Before his death in 1227, the Great Khan declared that his third son, Ögedei, should
succeed him, and Ögedei’s two older brothers, Jochi and Chagatai, were passed over.
Under this second Great Khan, the territory of the Mongol Empire reached its farthest extent
west and south. Güyük became the next Great Khan on the death of his father, in 1246, but
he ruled for less than two years. Möngke was the eldest son of Genghis Khan’s youngest
son Tolui, and succeeded his cousin as the next Great Khan. Möngke was in the process of
conquering Song China when he died in 1259. He was succeeded by his brother Kublai, who
united conquered Chinese territory and became founder and Emperor of the Yuan Dynasty.

Which of the following diagrams correctly shows the Great Khan succession?

© 2017 edulink GmbH www.edu-link.de ALL RIGHTS RESERVED 117


Buyer: Jan Hiller (janhiller@gmx.net)
Transaction ID: 8BE25485KM511652L

Humanities, Cultural Studies and


Social Sciences Module

(A) Genghis Jochi Chagatai Ogedei Guyuk Mongke Kublai

(B) Genghis Ogedei Guyuk Tolui Mongke Kublai

(C) Genghis Ogedei Guyuk Mongke Kublai

(D) Genghis Guyuk Ogedei Mongke Kublai

Answer C

We can surmise from the text that the first Great Khan was Genghis. Not far into the text, we
learn that he chose Ögedei to succeed him; thus, any diagrams that do not show Ögedei in
second place can be disregarded. This leaves B and C as possible answers. Both show the
next Great Khan, Guyuk, correctly, but C rightly names Mongke as the one who succeeded
him ("Möngke was the eldest son of Genghis Khan’s youngest son Tolui, and succeeded his
cousin as the next Great Khan"). This means that Answer C is correct.

TIP

While reading a text, try looking for clues that might help you better understand the
information presented. In this case, Genghis is described as the founder of the
Mongol Empire and the Great Khan. From this, we can presume Genghis was the
first Great Khan. Later in the text, Ögedei is referred to as the second Great Khan.
Also, be aware that some names and terms mentioned in a text may not be relevant
to the solution. In this case, the eldest sons of Genghis, Jochi and Chagatai were
passed over and so never became Great Khans. This will often be the case, which is
why you should not be overwhelmed when a text presents a large number of names,
dates or figures. You will rarely have to consider them all at the same time to get the
answer.

© 2017 edulink GmbH www.edu-link.de ALL RIGHTS RESERVED 118


Buyer: Jan Hiller (janhiller@gmx.net)
Transaction ID: 8BE25485KM511652L

Humanities, Cultural Studies and


Social Sciences Module

1.9.

GENGHIS KHAN FAMILY TREE

Use the information supplied in question 9 - The Great Khans - to help you answer the
following:

Genghis Khan

Jochi Chagatai Ogedei Tolui

Guyuk Mongke Kublai

Which of the following statements is or are correct?

I. The diagram correctly shows the affiliations between Tolui and his sons.

II. The diagram correctly shows the affiliations between Genghis Khan and his sons.

(A) Only statement (I) is correct.

(B) Only statement (II) is correct.

(C) Both statements are correct.

(D) Neither of the two statements is correct.

Answer B

The affiliations between Genghis Khan and his sons are shown correctly by the diagram:
Jochi, Chagatai, Ögedei and Tolui are all sons of Genghis Khan, although Tolui is not
mentioned along with the other names near the beginning of the text. However, the sons of
Tolui are not shown correctly. We know this because the text states that Guyuk is Ögedei’s
son ("Under this second Great Khan (Ögedei), the territory of the Mongol Empire reached its

© 2017 edulink GmbH www.edu-link.de ALL RIGHTS RESERVED 119


Buyer: Jan Hiller (janhiller@gmx.net)
Transaction ID: 8BE25485KM511652L

Humanities, Cultural Studies and


Social Sciences Module

farthest extent west and south. Güyük became the next Great Khan on the death of his
father, in 1246…").

TIP

When dealing with a question like this, remember that information that you would
logically expect to be presented together can sometimes be spread throughout the
text. For example, we learn in the first 2-3 lines that Ögedei, Jochi and Chagatai are
the names of three of Genghis Khan’s sons, but we do not learn the name of the
fourth one until nearer the end of the text.

1.10.

EATING BLUEBERRIES

According to research, health conscious people eat more blueberries than those who do not
take so much care of their bodies. Blueberries contain high levels of a chemical called
anthocyanin. It is believed that anthocyanin is particularly beneficial for the heart because it
lowers blood pressure and makes blood vessels more elastic, reducing the risk of heart
attack. Because of the higher level of anthocyanin, eating lots of blueberries also leads to a
higher level of fitness in general.

Health 
conscious

Number of 
Level of  +  Blueberries 
+  Levels of  +  Risk of heart 
fitness anthocyanin attack
eaten


Level of 
income

Which of the following statements is or are correct?

© 2017 edulink GmbH www.edu-link.de ALL RIGHTS RESERVED 120


Buyer: Jan Hiller (janhiller@gmx.net)
Transaction ID: 8BE25485KM511652L

Humanities, Cultural Studies and


Social Sciences Module

I. The diagram correctly shows the effects of fitness on the number of blueberries
eaten.

II. The diagram correctly shows the effects high-levels of anthocyanin have on the risk
of heart attack.

(A) Only statement (I) is correct.

(B) Only statement (II) is correct.

(C) Both statements are correct.

(D) Neither of the two statements is correct.

Answer D

Statement (I) is incorrect, as the text states that the relationship between the level of fitness
and the amount of blueberries eaten is the other way round: an increase in the number of
blueberries eaten (the dominant variable) leads to an increase in levels of fitness (the target
variable). Statement (II) is incorrect as well, as the sentence "It is believed that anthocyanin
is particularly beneficial for the heart because it lowers blood pressure and makes blood
vessels more elastic reducing the risk of heart attack" points to a negative relationship
between levels of anthocyanin and the risk of heart attack (remember – negative relationship
means that as one goes up, the other goes down). The diagram, however, shows a positive
relationship.

TIP

Remember that the only information you should use to answer the question is the
information provided in the text. For example, in the case of statement A, you could
argue that higher levels of fitness leads to an increase in the number of blackberries
eaten, because fit people make healthier eating choices. However, the text does not
suggest that the relationship could function in this direction, even though you may
believe this to be true in real life. Thus, you have to say that according to the
information provided in the text, statement (I) is incorrect. Moral of the story: don’t
overcomplicate things, and remember this is a logic exam, not a test of your ability to
discuss or analyse!

© 2017 edulink GmbH www.edu-link.de ALL RIGHTS RESERVED 121


Buyer: Jan Hiller (janhiller@gmx.net)
Transaction ID: 8BE25485KM511652L

Humanities, Cultural Studies and


Social Sciences Module

1.11.

URBANIZATION

Many Asian countries are experiencing a period of economic growth and development. This
economic growth and development has led to an increase in employment opportunities. The
Asian city populations have increased in response to the rise in employment opportunities.
As city populations increased, there was a corresponding need for new land to build housing
and services on in order to meet the needs of the new inhabitants. This kind of growth is
known as urbanization. This urbanization impacts the environment by leading to a loss of
natural habitats and a rise in air and water pollution.

Number of 
natural habitats
‐ 

Economic growth & 
+ Employment 
+ +
City Population Urbanization
Development Opportunities

Air & water 
pollution levels

Which of the following statements is or are correct?

I. The diagram correctly shows the effects of employment opportunities on city


populations.

II. The diagram correctly shows the effects of urbanization on natural habitats and
pollution levels.

(A) Only statement (I) is correct.

(B) Only statement (II) is correct.

(C) Both statements are correct.

(D) Neither of the two statements is correct.

Answer C

Statement (I) is correct, as the sentence "The Asian city populations have increased in
response to the rise in employment opportunities" points to a positive relationship between
employment opportunities and city populations. This is what the diagram shows.

Statement (II) is also correct, as the sentence "This urbanization impacts the environment by
leading to a loss of natural habitats and a rise in air and water pollution" points to a negative

© 2017 edulink GmbH www.edu-link.de ALL RIGHTS RESERVED 122


Buyer: Jan Hiller (janhiller@gmx.net)
Transaction ID: 8BE25485KM511652L

Humanities, Cultural Studies and


Social Sciences Module

relationship between urbanization and natural habitats, and a positive relationship between
urbanization and pollution levels. This is what the diagram shows.

TIP

Once you have read the text, read statement (I) and make sure you understand the
inferred relationship between each variable in the statement (does an increase in the
first variable lead to an increase or a decrease in the second variable?). Remember
that when an increase in a dominant variable (the variable that comes first) leads to a
decrease in the target variable, the relationship between them is represented by a "-".
Follow the same procedure for statement (II).

1.12.

MANAGEMENT PLAN

The Water Quality and Salinity Management Plan aims to use water from the Murray-Darling
Basin River system in a more sustainable way. Limiting the amount of water extracted from
its rivers will benefit the area’s wetlands and the ecosystems that depend on them. Allowing
the rivers to flow naturally will reduce the build-up of sediment and help improve water
quality.

How can one show these effects in a diagram?

I. The amount of water extracted is connected to benefits for the area’s wetlands and

their ecosystems using .

II. Allowing the rivers to flow naturally is connected to a reduction in the build-up of

sediment using .

(A) Only statement (I) is correct.

(B) Only statement (II) is correct.

(C) Both statements are correct.

(D) Neither of the two statements is correct.

Answer C

© 2017 edulink GmbH www.edu-link.de ALL RIGHTS RESERVED 123


Buyer: Jan Hiller (janhiller@gmx.net)
Transaction ID: 8BE25485KM511652L

Humanities, Cultural Studies and


Social Sciences Module

Statement (I) is correct, as the sentence "Limiting the amount of water extracted from its
rivers will benefit the area’s wetlands and the ecosystems that depend on them" points to a
negative relationship between the amount of water extracted and the benefits for the area’s
wetlands and their ecosystems. This is described correctly in the statement.

Statement (II) is correct, as the sentence "Allowing the rivers to flow naturally will reduce the
build-up of sediment and help improve water quality" points to a negative relationship
between allowing the rivers to flow naturally and the build-up of sediment.

TIP

Once you have read the text, read statement (I) and make sure you understand the
inferred relationship between the two variables in the statement. In this case, the
phrase "A reduction in the amount of water extracted" is placed at the beginning of
the statement and is the principle cause of the effect that follows. If you are having
problems following the relationships between variables, then draw boxes to represent
the variables and connect them using arrows. Next, determine the relationship
between the variables you have identified. Always remember that when an increase
in the dominant variable leads to a decrease in the target variable, the relationship

between them is represented by a " ". Follow the same procedure for statement
(II).

1.13.

MURRAY-DARLING BASIN

Water extraction severely reduces the flow of the Murray-Darling Basin River system. This
causes the area’s wetlands to dry up, which creates favorable conditions for blue-green
algae to flourish. These algae-like bacteria have a potential for toxicity and are a serious
threat to water quality. The diversion of the natural flow of the rivers causes sediment to
build up in areas that are permanently flooded. Sedimentation reduces the movement of the
water and results in high salinity levels and a deterioration in water quality.

© 2017 edulink GmbH www.edu-link.de ALL RIGHTS RESERVED 124


Buyer: Jan Hiller (janhiller@gmx.net)
Transaction ID: 8BE25485KM511652L

Humanities, Cultural Studies and


Social Sciences Module

Driving‐up of 
+ Blue – Green algae 
wetland areas blooms +

Water Quality


Sedimentation Water salinity levels

Which of the following statements is or are correct?

I. The diagram correctly shows the effects of blue-green algae blooms on water quality.

II. The diagram correctly shows the effects of sedimentation on water salinity levels.

(A) Only statement (I) is correct.

(B) Only statement (II) is correct.

(C) Both statements are correct.

(D) Neither of the two statements is correct.

Answer D

Statement (I) is incorrect, as the sentence "These algae-like bacteria have a potential for
toxicity and are a serious threat to water quality” points to a negative relationship between
blue-green algae and water quality. The diagram, however, shows a positive relationship
between blue-green algae and water quality.

Statement (II) is incorrect, as the sentence "Sedimentation reduces the movement of the
water and results in high salinity levels and deterioration in water quality” points to a positive
relationship between sedimentation and water salinity levels. The diagram, however, shows
a negative relationship between sedimentation and water salinity levels.

© 2017 edulink GmbH www.edu-link.de ALL RIGHTS RESERVED 125


Buyer: Jan Hiller (janhiller@gmx.net)
Transaction ID: 8BE25485KM511652L

Humanities, Cultural Studies and


Social Sciences Module

1.14.

DEFORESTATION AND GLOBAL WARMING

Deforestation for agricultural and logging purposes is threatening much of the earth’s
remaining tropical rainforest. Large forested areas play a vital role in regulating the world’s
climate patterns by acting as a carbon sink. Rainforests soak up carbon dioxide (CO2) that
would otherwise enter the atmosphere and convert it to oxygen. Increased levels of CO2 in
the atmosphere are linked to long-term trends in global warming (rising global temperatures).

Agriculture

Tropical  ‐ Levels of CO2 in  ‐ Global 


rainforests the atmosphere  temperatures


Logging

Which of the following statements is or are correct?

I. The diagram does not show the effects that agriculture and logging are having on
tropical rainforests correctly.

II. The diagram correctly shows the effects that levels of CO2 are having on global
temperatures.

(A) Only statement (I) is correct.

(B) Only statement (II) is correct.

(C) Both statements are correct.

(D) Neither of the two statements is correct.

Answer D

Statement (I) is incorrect. The sentence "Deforestation for agricultural and logging purposes
is threatening much of the earth’s land surface still covered by tropical rainforests" points to
a negative relationship between agriculture and logging and tropical rainforests (as
agriculture and logging rise, the area of rainforest falls). This is what the diagram shows
correctly.

© 2017 edulink GmbH www.edu-link.de ALL RIGHTS RESERVED 126


Buyer: Jan Hiller (janhiller@gmx.net)
Transaction ID: 8BE25485KM511652L

Humanities, Cultural Studies and


Social Sciences Module

Statement (II) is incorrect, as the sentence "Increased levels of CO2 in the atmosphere are
linked to long-term trends in global warming" points to a positive relationship between
increased levels of CO2 in the atmosphere and global warming. The diagram, however,
shows a negative relationship.

TIP

When studying this text and diagram, it is important to remember that when an
increase in the dominant variable (effect-causing variable) leads to a decrease in the
target variable, the relationship between them is represented by a "-". In this case,
increased levels of agriculture and logging lead to a reduction in tropical rainforests -
a negative relationship. Likewise, it is important to keep in mind that when an
increase in a dominant variable leads to an increase in the target variable, the
relationship between them is represented by a "+". In this case, increased levels of
CO2 in the atmosphere are linked to long-term trends in global warming (rising global
temperatures) – a positive effect.

1.15.

Refer back to the information in Exercise 14 to help you answer this exercise.

Agroforestry is an approach to agriculture and logging that looks to encourage long-term


production of wood through the sustainable management of tropical rainforests. Sustainable
farming practices – such as the simultaneous growing of crops and planting of trees and the
selective felling of trees – allow areas of rainforest to reach full maturity. Mature rainforests
are far more effective at reducing levels of CO2 in the atmosphere than areas that have not
been used in a sustainable way. Lower levels of CO2 in the atmosphere will help slow down
the rate of global warming.

Which of the following statements is or are correct?

I. The diagram correctly shows the effects that agriculture and logging are having on
tropical rainforests (negative arrow), which is in turn linked to levels of CO2 in the

atmosphere using (negative arrow).

II. The levels of CO2 in the atmosphere are connected to the rate of global warming

using , where the positive arrow points to the rate of global warming.

(A) Only statement (I) is correct.

© 2017 edulink GmbH www.edu-link.de ALL RIGHTS RESERVED 127


Buyer: Jan Hiller (janhiller@gmx.net)
Transaction ID: 8BE25485KM511652L

Humanities, Cultural Studies and


Social Sciences Module

(B) Only statement (II) is correct.

(C) Both statements are correct.

(D) Neither of the two statements is correct.

Answer B

Statement (I) is incorrect, as the sentence "Sustainable farming practices - such as the
simultaneous growing of crops and planting of trees and the selective felling of trees - allows
areas of rainforest to reach full maturity" points to a positive relationship between
sustainable farming practices and rainforests reaching full maturity. Even though the second
part of the statement is correct (negative relationship between area of rainforest reaching full
maturity and levels of CO2 in the atmosphere), the statement as a whole is therefore
incorrect.

Statement (II) is correct, as the sentence "Lower levels of CO2 in the atmosphere will help
slow down the rate of global warming" points to a positive relationship between levels of CO2
in the atmosphere and the rate of global warming (remember that in a positive relationship, a

decrease in one variable will also produce a decrease in the other). Therefore must
point to the rate of global warming.

1.16.

JAPAN’S AGEING POPULATION

Because the average age in Japan (A) is steadily rising, there is an increase in the demand
(D) for nursing staff to help look after the country’s ageing population. Due to the rise in the
demand for nursing staff, nurses’ pay (P) started to rise in order to encourage more young
people to take-up a career in the healthcare industry. This policy was a success, resulting in
a twofold increase in the number of students (S) taking healthcare related courses, and a
similar surge in the number of graduates entering the nursing profession (N). The growth in
nurses’ pay (P), however, has begun to slow down as a result of the increase in the supply
of nursing staff.

© 2017 edulink GmbH www.edu-link.de ALL RIGHTS RESERVED 128


Buyer: Jan Hiller (janhiller@gmx.net)
Transaction ID: 8BE25485KM511652L

Humanities, Cultural Studies and


Social Sciences Module

A
+  D
+  P

+  + 

N
+  S

Which of the following statements is or are correct?

I. The diagram correctly shows the effects of an ageing population on the demand for
nursing staff.

II. The diagram correctly shows the effects the supply of newly recruited nurses is
having on pay.

(A) Only statement (I) is correct.

(B) Only statement (II) is correct.

(C) Both statements are correct.

(D) Neither of the two statements is correct.

Answer A

Statement (I) is correct, as the sentence "Because the average age in Japan (A) is steadily
rising, there is an increase in the demand (D) for nursing staff to help look after the country’s
aged population" points to a positive relationship between an ageing population and an
increase in the demand for nurses. This is what the diagram shows.

Statement (II) is incorrect, as the sentence "The growth in nurses’ pay (P), however, has
begun to slow down as a result of the increase in the supply of nursing staff" points to a
negative relationship between the increase in the supply of nursing staff and the average
pay for nurses. The diagram, however, shows a positive relationship.

TIP

To understand this diagram, it is important to be aware of the concept of supply and


demand. The variable pay (P) is affected by both the demand (D) and the supply
(N). Only one of these variables, (D), has a positive effect on pay (P) - the greater the
demand for nurses, the greater the growth in pay; or, the lesser the demand for

© 2017 edulink GmbH www.edu-link.de ALL RIGHTS RESERVED 129


Buyer: Jan Hiller (janhiller@gmx.net)
Transaction ID: 8BE25485KM511652L

Humanities, Cultural Studies and


Social Sciences Module

nurses, the lesser the growth in pay. Supply (N) has a negative effect on pay (P): the
greater the supply of nurses, the lesser the growth in pay; or, the lesser the supply of
nurses, the greater the growth in pay.

1.17.

LIFE EXPECTANCY IN JAPAN

Japan has the highest life expectancy in the world. Higher life expectancy has caused an
increase in the country’s dependency ratio: the number of people not of working age to those
who are economically active. The growth in the dependency ratio is causing fiscal problems
for the government because lower tax revenue negatively impacts government spending.

Life Expectancy
+  Dependency Ratio
+  Tax Revenue
+  Government Spending

Which of the following statements is or are correct?

I. The diagram correctly shows the effects of life expectancy on the dependency ratio.

II. The diagram correctly shows the effects of the dependency ratio on tax revenue and
the effects of tax revenue on government spending.

(A) Only statement (I) is correct.

(B) Only statement (II) is correct.

(C) Both statements are correct.

(D) Neither of the two statements is correct.

Answer A

Statement (I) is correct, as the sentence "Higher life expectancy has caused an increase in
the country’s dependency ratio: the number of people not of working age to those who are
economically active" points to a positive relationship between life expectancy and the
dependency ratio. This is what the diagram shows.

Statement (II) is incorrect, as the sentence "The growth in the dependency ratio is causing
fiscal problems for the government because lower tax revenue negatively impacts
government spending" points to a negative relationship between the dependency ratio and
tax revenue. The diagram, however, shows a positive relationship between the dependency
ratio and tax revenue.

© 2017 edulink GmbH www.edu-link.de ALL RIGHTS RESERVED 130


Buyer: Jan Hiller (janhiller@gmx.net)
Transaction ID: 8BE25485KM511652L

Humanities, Cultural Studies and


Social Sciences Module

TIP

To solve this question, it is important to understand the meaning of the sentence


"The growth in the dependency ratio is causing fiscal problems for the government
because lower tax revenue negatively impacts government spending". This sentence
is actually describing two relationships:

o It is suggesting that as the dependency ratio increases, the government


receives less tax revenue (fiscal problems = financial problems = lower tax
revenue), and;

o Lower tax revenue leads to lower government spending (remember that even
though we are talking about decreases, this is actually a positive relationship,
since a change in one variable produces a change in the same direction in
the other).

Be aware that sometimes, as in the first statement, a relationship will not always be
described explicitly.

1.18.

LOWERING MORTALITY RATES IN JAPAN

Life expectancy in Japan has benefitted from a reduction in salt content in the Japanese diet,
which has helped to remove stroke as the country’s leading cause of death. Improvements
to the country’s healthcare system and raised public awareness of health issues have also
contributed to the downward trend in stroke mortality rates. Naturally, a lower death rate
from stroke has also resulted in a higher life expectancy in general.

How can one show these effects in a diagram?

I. The amount of salt in the Japanese diet is connected to a change in the death rate

from stroke using .

II. Improvements to the country’s healthcare system and raised public awareness of

health issues are both connected to a change in the death rate from stroke using .

The change in the death rate from stroke is also connected to life expectancy using

© 2017 edulink GmbH www.edu-link.de ALL RIGHTS RESERVED 131


Buyer: Jan Hiller (janhiller@gmx.net)
Transaction ID: 8BE25485KM511652L

Humanities, Cultural Studies and


Social Sciences Module

(A) Only statement (I) is correct.

(B) Only statement (II) is correct.

(C) Both statements are correct.

(D) Neither of the two statements is correct.

Answer C

Statement (I) is correct, as the sentence "Life Expectancy in Japan has benefitted from a
reduction in salt content in the Japanese diet, which has helped remove stroke as the
country’s leading cause of death” points to a negative relationship between dietary salt and
the death rate from stroke. This is what the statement shows.

Statement (II) describes two different relationships, both of which must be correct in order for
the overall statement to be correct. Let’s consider the two parts separately:

○ Sentence 1: From the text, we know that there is a negative relationship between
each of the dominant variables (improvements to the healthcare system / raised
public awareness of health issues) and the target variable (death rate from stroke).
For each of these cases, an increase in the dominant variable leads to a decrease in
the target variable. This is what the sentence says.

○ Sentence 2: From the text, we also know that there is a negative relationship
between the death rate from stroke and the general life expectancy. This is what the
sentence says.

Both parts of the statement are correct, thus the statement as a whole is correct.

1.19.

According to the German tale, the "Nibelungen", Siegmund zu Xanten, grandson of Odin,
has a child named Siegfried with his wife Sieglinde. Siegfried is called the "Dragon Slayer”
and he is the main character of the medieval German saga. Siegfried's wife Krimhild is
Dankrat's and Uta’s daughter and also the niece of Hagen von Tronje. Siegfried and Krimhild
are the parents of Gunther zu Worms; however, Krimhild also has a son with Etzel, named
Ortlieb. Etzel has a daughter, Brynhild, with Helche.

The following diagram is intended to show the family affiliations described:

© 2017 edulink GmbH www.edu-link.de ALL RIGHTS RESERVED 132


Buyer: Jan Hiller (janhiller@gmx.net)
Transaction ID: 8BE25485KM511652L

Humanities, Cultural Studies and


Social Sciences Module

Odin

Siegt Wels

Siegmund zu 
Sieglinde Dankrat Hagen Uta
Xanten

Siegfried Krimhild Etzel Helche

Gunther zu 
Ortlieb Brynhild
Worms

Which of the following two statements about this diagram is or are correct?

I. The affiliation between Gunther zu Worms, Ortlieb, and Brynhild is shown correctly.

II. The affiliation between Ortlieb and Hagen is shown correctly.

(A) Only statement (I) is correct.

(B) Only statement (II) is correct.

(C) Both statements are correct.

(D) Neither of the two statements is correct.

Answer A

Statement I is correct as the affiliation between Gunther zu Worms, Ortlieb, and Brynhild is
shown correctly. Gunther zu Worms is the son of Siegfried and Krimhild, Ortlieb is the sone
of Krimhild and Etzel, and Brynhild is the daughter of Etzel and Helche.

Statement II, however, is incorrect, since the affiliation between Ortlieb and Hagen is shown
incorrectly in the diagram. Hagen is not Ortlieb's grandfather, but his great uncle (Ortlieb’s
mother, Krimhild, is the niece of Hagen). Dankrat is Ortlieb's grandfather.

© 2017 edulink GmbH www.edu-link.de ALL RIGHTS RESERVED 133


Buyer: Jan Hiller (janhiller@gmx.net)
Transaction ID: 8BE25485KM511652L

Humanities, Cultural Studies and


Social Sciences Module

1.20.

CLOSED ECOLOGICAL SYSTEM

In a closed ecological system, natural elements interact to produce a flow of nutrients that
remain in the ecosystem as they are recycled within the environment. Plants introduce
nutrients – chemical elements – into the soil through the biomass they produce. As the
biomass decomposes, microorganisms in the soil break down the plant matter and release
nutrients that help maintain the health – or rather, the quality of the soil. The higher the
nutrient content of the soil, the higher the soil quality. Healthy soil is essential for sustaining
plant growth, producing biomass and keeping the nutrient cycle running smoothly.

Nutrient content of 
soil
+ ‐

+
Plant biomass Soil Quality

Which of the following two statements about this diagram is or are correct?

I. The diagram correctly shows the effects of plant biomass on the nutrient content of
soil.

II. The diagram correctly shows the effects of the nutrient content of soil on soil quality.

(A) Only statement (I) is correct.

(B) Only statement (II) is correct.

(C) Both statements are correct.

(D) Neither of the two statements is correct.

Answer A

Statement (I) is correct, as the sentence "Plants introduce nutrients – chemical elements –
into the soil through the biomass they produce” points to a positive relationship between
plant biomass and the nutrient content of soil. This is what the diagram shows.

© 2017 edulink GmbH www.edu-link.de ALL RIGHTS RESERVED 134


Buyer: Jan Hiller (janhiller@gmx.net)
Transaction ID: 8BE25485KM511652L

Humanities, Cultural Studies and


Social Sciences Module

Statement (II) is incorrect, as the sentence "The higher the nutrient content of the soil, the
higher the soil quality” points to a positive relationship between the nutrient content of soil
and soil quality. The diagram, however, shows a negative relationship between nutrient
content of soil and soil quality.

TIPS

This question requires you to use exactly the same process you have already used in
several previous questions! Read Statement (I) carefully and decide whether the
relationship shown in the diagram corresponds to what is written in the text, drawing
out your own diagram if you think it will help. Then repeat the procedure for
Statement (II).

1.21.

See question 20 for additional information.

OPEN ECOLOGICAL SYSTEM

In an open ecological system, a large proportion of the energy and nutrients produced does
not remain inside the system. Instead it is harvested in the form of fruit, grain, and
vegetables. The quality of the soil is adversely affected by this loss of nutrients from the
system. This means that fertilizers are required to prevent soil degradation and to ensure
that soil has a high nutrient content, which in turn leads a high soil quality.

+ +
Plant biomass  Nutrient content of 
Soil Quality
(decomposing) soil

What additions have to be made to the diagram to show the effects of an open ecological
system?

Pla = Plant biomass Harv = Harvest

Nut = Nutrient content of soil Fer = Fertilizer

Soil = Soil quality

Which of the following two statements about this diagram is or are correct?

© 2017 edulink GmbH www.edu-link.de ALL RIGHTS RESERVED 135


Buyer: Jan Hiller (janhiller@gmx.net)
Transaction ID: 8BE25485KM511652L

Humanities, Cultural Studies and


Social Sciences Module

I. Harv is positioned before Pla and is linked to it by .

II. Fer is positioned between Nut and Soil. One points from Nut to Fer, one
points from Fer to Soil.

(A) Only statement (I) is correct.

(B) Only statement (II) is correct.

(C) Both statements are correct.

(D) Neither of the two statements is correct.

Answer C

Statement (I) is correct: the new information places Harv before Pla. The sentence, "In an
open ecological system, a large proportion of the energy and nutrients produced does not
remain inside the system, instead it is harvested in the form of fruit, grain and vegetables”
points to a negative relationship between harvesting (the dominant variable, or the variable
that causes the effect) and the amount of plant biomass left to decompose in the soil (the

target variable). Thus, this relationship should be represented using (negative arrow).

Statement (II) is correct: the new information places Fer between Nut and Soil. Fertilizers are
used to replace nutrients in the soil lost to harvesting, ultimately increasing the quality of the
soil. The following sentences point to a negative relationship between the nutrient content of

the soil and fertilizer, and are correctly represented by : "The quality of the soil is
adversely affected by this loss of nutrients from the system. This means that fertilizers are
required to prevent soil degradation and to ensure that soil has a high nutrient content, which
in turn leads to high soil quality.” It also points to a positive relationship between fertilizer and

soil quality, which is correctly represented by .

TIP

This question is linked to the question Closed Ecological System (Exercise 20). In the
current question, the diagram representing the closed ecological system has been
opened up and now runs from left to right. New variables (harvest and fertilizer) have
been introduced because we are now considering an open system (a system with an
unlimited number of variables). Though we are dealing with more variables than we
have seen in previous questions, the process for arriving at the answer is exactly the
same. Read statement (I) and make sure you understand the inferred relationship
between each variable in the statement: is it negative or positive, and have the
dominant and target variables been put in the correct order? If you are having

© 2017 edulink GmbH www.edu-link.de ALL RIGHTS RESERVED 136


Buyer: Jan Hiller (janhiller@gmx.net)
Transaction ID: 8BE25485KM511652L

Humanities, Cultural Studies and


Social Sciences Module

problems following the relationships between variables, then draw boxes to represent
the variables and connect them using arrows. Next, determine the relationship
between the variables you have identified. Always remember that when an increase
in a dominant (effect-causing) variable leads to a decrease in the target variable, the
relationship between them is represented by a "-". Follow the same procedure for
statement (II).

1.22.

SUPER FOOD

Blueberries are often described as super food because of the many health benefits the fruit
provides. Blueberries contain high levels of antioxidant nutrients, which help limit the
damage caused by free radicals to cellular structures. Research has shown that a blueberry-
rich diet may protect the brain from oxidative stress, which helps to improve memory and
other cognitive functions in older people.

How can one show these effects in a diagram?

I. Eating blueberries is connected to high levels of antioxidant nutrients using and is

connected to the limiting of the damage caused to cells using . Both of these
arrows point to eating blueberries.

II. Eating blueberries is connected to a development of oxidative stress in the brain

using and is also connected to improved memory using . points to the

oxidative stress, and points to improved memory.

(A) Only statement (I) is correct.

(B) Only statement (II) is correct.

(C) Both statements are correct.

(D) Neither of the two statements is correct.

Answer B

Statement (I) is incorrect. The sentence "Blueberries contain high levels of antioxidant
nutrients, which help limit the damage caused by free radicals to cellular structures” points to
a positive relationship between eating blueberries and high levels of antioxidant nutrients

© 2017 edulink GmbH www.edu-link.de ALL RIGHTS RESERVED 137


Buyer: Jan Hiller (janhiller@gmx.net)
Transaction ID: 8BE25485KM511652L

Humanities, Cultural Studies and


Social Sciences Module

and a negative relationship between eating blueberries and damage to cellular structures.
Thus, the first part of the statement is correct. However, the arrows should point FROM
eating blueberries TOWARDS high levels of antioxidant nutrients and damage to cellular
structures, not the other way round. Thus, the overall statement is incorrect.

Statement (II) is correct. The sentence "Research has shown that a blueberry-rich diet may
protect the brain from oxidative stress, which could help improve memory and other
cognitive functions in older people” points to a positive relationship between eating
blueberries and protection of the brain from oxidative stress, and a positive relationship
between eating blueberries and improved memory. Eating blueberries improves the

protection of the brain against oxidative stress; therefore, must point towards improved
memory. A higher protection of the brain against oxidative stress improves memory;

therefore, points to improved memory.

TIP

Be aware that in these kinds of questions, the statement must correctly describe both
the nature of the relationship (positive or negative) and the direction of the
relationship (which is the dominant variable and which is the target one). In statement
(I), even though the type of relationship is described correctly, the statement wrongly
identifies high levels of antioxidant nutrients and damage to cellular structures as
dominant variables, not target ones. Thus, the statement as a whole is incorrect.
Remember to draw out your own diagram if you get stuck!

© 2017 edulink GmbH www.edu-link.de ALL RIGHTS RESERVED 138


Buyer: Jan Hiller (janhiller@gmx.net)
Transaction ID: 8BE25485KM511652L

Humanities, Cultural Studies and


Social Sciences Module

4 RECOGNISING LINGUISTIC STRUCTURES

The subtest "Recognising Linguistic Structures" is the last and arguably the most difficult
section of the Humanities, Cultural Studies and Social Sciences Module. In this question
type you will be given brief sentences in English and a fictitious language that does not exist
in real life. The given words/sentences enable you to decipher the grammatical structure of
the language to the extent that you can translate the English sentence in the question.

Let’s illustrate this unique question type with an easy example.

EXAMPLE 1

(I) tuludola = I called.

(II) tolodola = We called.

"He called" is expressed in the foreign language by:

(A) tulufola

(B) tuludolu

(C) tolodulu

(D) tulodola

Answer D

When we look at the given English text, we see that the verb and the tense are the same:
called, i.e. "call" in the past tense. Let’s see what part of the word is the same in the foreign
language.

(I) tuludola = I called.

(II) tolodola = We called.

Looking at the repeated words in these 2 sentences, "dola" means "called" in English. In the
answer choices, only answer choice D ends with "dola".

© 2017 edulink GmbH www.edu-link.de ALL RIGHTS RESERVED 139


Buyer: Jan Hiller (janhiller@gmx.net)
Transaction ID: 8BE25485KM511652L

Humanities, Cultural Studies and


Social Sciences Module

Additionally, we can deduce the following rules of the fictitious language based on these
simple sentences.

 Looking at the location of “dola”, we can already deduce the sentence structure. Verb
(v) and tense are together and these follow the subject(s) in one word. 
s+(v+tense)

 Consequently, "tulu" must mean "I".

 "tolo" means "we".

 What we do not know is how the verb and tense are sequenced in the word. "dola"
means "called", but "dol" could mean "call" and "a" could indicate the past tense. Or
d+ola, do+la, etc.

EXAMPLE 2

(I) tuludola = I called.

(II) tolodola = We called.

(III) tolodol = We call.

"I will call" is expressed in the foreign language by:

(A) tolodolu

(B) tuludolu

(C) tuludol

(D) toludolu

Answer B

As analysed in Example 1 from the statements (I) and (II), we know that the subject is
followed by the verb and tense in one word.

X X X X X X X X

Subject Verb+tense

© 2017 edulink GmbH www.edu-link.de ALL RIGHTS RESERVED 140


Buyer: Jan Hiller (janhiller@gmx.net)
Transaction ID: 8BE25485KM511652L

Humanities, Cultural Studies and


Social Sciences Module

When we compare statements (II) and (III), we see that the only difference is the ending “a”,
which must indicate the past tense.

(I) tolodola = We called.

(II) tolodol = We call.

"dol" is the present tense of the verb "to call", and to make this past tense, we add at the end
of the verb the letter "a". This means that we have the following structure.

X X X X X X X X

Subject Verb Tense

In Example 2 the translation of "I will call" is asked.

"I call" would be "tuludol". We need an ending in order to convert this to the future. We need
to look at the answer to find this out, as in the given statements there is no reference to the
future tense.

Answer choices A and D can be eliminated because they do not start with "tuludol".

Answer choice C does not have an ending, indicating present tense.

Consequently, answer choice B must be correct. "u" at the end of the verb indicates the
future tense in the fictitious language.

While answering the question type "Recognising Linguistic Structures", you are looking to
identify the following three aspects of the new language:

1) The sentence structure: Where is the subject / verb / object? Which are written
together and in which sequence?

2) The meaning of certain words: It is often necessary to know the meaning of the verb
and/or subject.

3) The way the tenses are built (present, future, past): Is it an ending after the verb? A
separate word?

We will show you with a few examples possible ways to solve this question type.
Additionally, at the end of this section, you will have 2 full-size tests to practice and refine
your answering approach. This question type is quite unique, but once you get the gist of it, it
is actually easy to solve.

© 2017 edulink GmbH www.edu-link.de ALL RIGHTS RESERVED 141


Buyer: Jan Hiller (janhiller@gmx.net)
Transaction ID: 8BE25485KM511652L

Humanities, Cultural Studies and


Social Sciences Module

4.1 GENERAL RULES FOR THIS SECTION

○ You have 50 minutes to answer the 22 questions.

○ You will be given sentences in English and in different fictitious languages. You need
to figure out the rules and build the sentence in the question based on these rules.
Every few questions you will be given a new fictitious language, which is not
connected in any way to the ficticious languages in the previous questions.

○ Questions increase in difficulty throughout the section. The first ones are very easy,
while the last ones are very tiring and time consuming. Quickly solve the first
questions and move on, without wasting time double-checking your result.

○ Only correct answers impact your score. Do not leave questions blank. There is no
penalty for wrong answers.

○ You will likely run out of time in this section. If you are stuck on a question, eliminate
the most obvious incorrect answer choices and guess from the remainder. You can
always come back if time remains (though this is highly unlikely).

○ The fictitious languages are not based on any real language or language rules. You
cannot and should not try to draw parallels between English and the ficticious
languages. For example, the word "chocolate" is a long word in many languages, but
it’s possible that in the fictitious language it’s only a few letters long (such as "au").
Similarly, "and" is a short word in English, but it can be very long, such as "hapus", in
the new language. Words may also be combined with other words as opposed to
being standalone. Rules in the fictitious language are independent of any rules you
may know in the English language.

○ There are no irregularities in the fictitious languages. If the rule of the fictitious
language is that a past tense is formed by adding the letters "sey" to a verb, this is
true for all verbs consistently, even if these are irregular in English.

 Example for irregular verbs: eat  ate (as opposed to eated)

 Example for irregular plural: leaf  leaves (leafs)

4.1.1 THERE ARE NO IRREGULARITIES IN THE FICTITIOUS LANGUAGES.

In English, as with most other languages, there are irregularities. Taking the verb tense as
an example, normally the ending "ed" in English converts a present tense verb to past tense.
However, there are many exceptions:

© 2017 edulink GmbH www.edu-link.de ALL RIGHTS RESERVED 142


Buyer: Jan Hiller (janhiller@gmx.net)
Transaction ID: 8BE25485KM511652L

Humanities, Cultural Studies and


Social Sciences Module

○ to fly  flew (as opposed to flyed)

○ to have  had (as opposed to haved)

○ to catch  caught (as opposed to catched)

Other irregularities can be found in the plural tense of words. In most cases an "s" ending
converts a singular noun to plural. But there are exceptions:

○ fish  fish (as opposed to fishes)

○ life  lives (as opposed to lifes)

○ foot  feet (as opposed to foots)

In the fictitious languages shown in this subtest, there are no irregularities, even if the
English translation has an irregularity. There is only one rule to convert a present tense to
past tense, which is consistent across all verbs.

EXAMPLE 3

(I) tuludola = I called.

(II) tolodola = We called.

(III) tolodol = We call.

(IV) tuludolu = I will call.

"We fell" is expressed in the foreign language by:

(A) tolukutba

(B) tolokuta

(C) tolokutt

(D) tolokutu

Answer B

© 2017 edulink GmbH www.edu-link.de ALL RIGHTS RESERVED 143


Buyer: Jan Hiller (janhiller@gmx.net)
Transaction ID: 8BE25485KM511652L

Humanities, Cultural Studies and


Social Sciences Module

Let’s repeat what we already know:

X X X X X X X X

Subject Verb Tense

I= tulu call=dol past tense= a

we=tolo future tense= u

This means that we already know part of the answer: (tolo)+(???)+(a). Given "to fall" is not
listed in the statements, we need to derive the solution from the answers.

Looking at the answers, only answer choice B uses the above structure. We now know that
"to fall" means "kut" in the fictitious language.

EXAMPLE 4

(I) tuludola = I called.

(V) tuludola busma = I called our friend.

(VI) tolodola busmad = We called our friends.

(VII) tuludola fonma = I called our child.

"I called our children" is expressed in the foreign language by:

(A) tuludola fonmas

(B) tolodola fonmas

(C) tuludola fonmad

(D) tolodola fonmad

Answer C

We know from statement (I) that the answer needs to start with "tuludola". We can already
eliminate answer choices B and D.

We now need to figure out what "our children" means. We can compare sentences (V) and
(VI) to decipher how the object is created. The addition of an object to the sentence resulted
in a second word.

© 2017 edulink GmbH www.edu-link.de ALL RIGHTS RESERVED 144


Buyer: Jan Hiller (janhiller@gmx.net)
Transaction ID: 8BE25485KM511652L

Humanities, Cultural Studies and


Social Sciences Module

(V) tuludola busma = I called our friend.

I call past-tense our friend

(VI) tolodola busmad = We called our friends.

We call past-tense our friends

Comparing "busma" with "busmad", we see that the ending "d" makes the word plural. (Of
course in a real language, there could be several different endings for plural words as well
as irregularities. But in this subtest, we know that the fictitious languages do not have
irregularities.)

Consequently, to answer the question, we need to take statement (VII) and add a “d” to the
end to make "child" plural. The correct answer is C.

(One could argue that answer choice A could also be correct because it uses an "s" ending
to show that the word is plural, which could indicate the plural of an irregular word. But as
previously stated, there are no irregularities in the fictitious languages.)

4.2 THIS QUESTION TYPE CAN BE MASTERED

This type of question is unusual for secondary school graduates since it requires you to
translate between English and a made-up (non-existing) language. The good news is – this
skill can be learned!

Your ability to solve these types of problems will be determined to a great extent by how
often you practice. The following sections contain plenty of practice questions. We
recommend you work through one test first, without any help, and then consult the
explanations. This way you can find out where your most pressing weaknesses are and
which type of question you need to focus on the most.

4.3 TIME MANAGEMENT IS ESSENTIAL

You have a total of 50 minutes to answer the 22 questions in the section "Recognising
Linguistic Structures". Keep in mind, you will already be tired at the very end of this long
exam and this section requires a high level of concentration – many students report not even
making it halfway through this section.

© 2017 edulink GmbH www.edu-link.de ALL RIGHTS RESERVED 145


Buyer: Jan Hiller (janhiller@gmx.net)
Transaction ID: 8BE25485KM511652L

Humanities, Cultural Studies and


Social Sciences Module

As in the rest of the TestAS sections, the easier questions come first. Since you will have
approximately two minutes per question, one strategy we recommend is to complete the
easy questions first.

It might help to remember that every question answered correctly increases your overall
humanities score. (The test average in this bird-language section is generally low. We
assume each correctly answered question from this section has more impact on your overall
score than a correctly answered question in the other two humanities sections.) With some
practice you could score above average in this difficult section and achieve a higher overall
score in the Humanities subject module.

4.4 TOP-DOWN APPROACH

In this section we explain how to decipher the rules of the language to the extent possible,
so that the sentence in question can be translated.

Please note that the top-down approach can be rather time-consuming because you are
deciphering rules without knowing which of these rules will be needed to answer the question.
For example, in a given question with sentences using different tenses, you may be able
decipher the rules for building tenses. But if the question does not require all of these rules, or
if you could have eliminated answers without using them, then you have probably wasted
precious time.

In any case, it’s good to go over a few examples with the top-down approach so that you
understand the principles of this question type.

We started this section with very easy questions. Now we move on to a medium difficulty
question.

EXAMPLES 5-9

(I) zanatlulu autim = The boy feeds the cat.

(II) uduldieda lulum = The girl calls the boy.

(III) zanatnana krabim = The mother feeds the children.

(IV) aidilsuka nanam = The neighbor visits the mother.

© 2017 edulink GmbH www.edu-link.de ALL RIGHTS RESERVED 146


Buyer: Jan Hiller (janhiller@gmx.net)
Transaction ID: 8BE25485KM511652L

Humanities, Cultural Studies and


Social Sciences Module

EXAMPLE 5

"The girl visits the cat" is expressed in the foreign language by:

(A) luluzanat auti

(B) aidildieda autim

(C) udulsuka nana

(D) aidildiedam auti

EXAMPLE 6

"The neighbor calls the children" is expressed in the foreign language by:

(A) udulsuka krabim

(B) aidilnana dieda

(C) aidilsuka krabi

(D) udulsukam krabim

EXAMPLE 7

"The boy visits the girl" is expressed in the foreign language by:

(A) aidiludul autim

(B) zanatlulu diedam

(C) aidillulu diedam

(D) zanatlulu dieda

EXAMPLE 8

"The neighbor feeds the children and the cat" is expressed in the foreign language by:

(A) zanatlulu autim diedam

© 2017 edulink GmbH www.edu-link.de ALL RIGHTS RESERVED 147


Buyer: Jan Hiller (janhiller@gmx.net)
Transaction ID: 8BE25485KM511652L

Humanities, Cultural Studies and


Social Sciences Module

(B) zanatsuka autim

(C) zanatlulu krabi auti

(D) zanatsuka krabimsautim

EXAMPLE 9

"The mother visits the neighbor" is expressed in the foreign language by:

(A) zanatnana sukam

(B) aidilnana sukam

(C) zanatsuka nanam

(D) aidilsuka krabim

Answer 5: B

Answer 6: A

Answer 7: C

Answer 8: D

Answer 9: B

Step 1: Identify the repeating words

 "the boy" in statements (I) and (II)


 "feeds" in statements (I) and (III)
 "the mother" in statements (III) and (IV)

Step 2: Look at each of the statements that use the repeating word to determine its
translation, and also use these to begin drawing conclusions about sentence structure.

We start with the word "the boy".

(I) zanatlulu autim = The boy feeds the cat.

© 2017 edulink GmbH www.edu-link.de ALL RIGHTS RESERVED 148


Buyer: Jan Hiller (janhiller@gmx.net)
Transaction ID: 8BE25485KM511652L

Humanities, Cultural Studies and


Social Sciences Module

(II) uduldieda lulum = The girl calls the boy.

We can conclude the following:

○ "lulu" means "the boy".


○ The subject is at the end of the first word, and the object is at the beginning of the
second word.

Next we look at "feeds".

(I) zanatlulu autim = The boy feeds the cat.


(III) zanatnana krabim = The mother feeds the children.

We can conclude the following:

○ "zanat" means "feeds".


○ The verb forms the beginning of the first word.
○ So the sentence structure is:

o First word: Verb(v)+Subject(s).

The third repeating word is "mother".

(III) zanatnana krabim = The mother feeds the children.


(IV) aidilsuka nanam = The neighbor visits the mother.

We can conclude the following:

○ "nana" means "mother".


○ We see that there is an "m" at the end of the object, as we saw in statement (II). It
looks like "m" indicates the direct object.
○ So the sentence structure is: Verb(v)+Subject(s) Object(o)+ "m"

Step 3: Top-down

Recap of what we already know:

○ the boy = lulu


○ feeds = zanat
○ the mother = nana
○ sentence structure: v+s (first word) o+"m" (second word)

Let’s go from statement to statement to discover the other words.

© 2017 edulink GmbH www.edu-link.de ALL RIGHTS RESERVED 149


Buyer: Jan Hiller (janhiller@gmx.net)
Transaction ID: 8BE25485KM511652L

Humanities, Cultural Studies and


Social Sciences Module

(I) zanatlulu autim = The boy feeds the cat.

We know the second word is the object. I.e., "auti" must mean the cat.

(II) uduldieda lulum = The girl calls the boy.

The first word in the fictitious language is formed by the verb followed by the subject. This
means the verb "to call" starts with the first few letters "ud…", while the word "the girl" ends
with "…da". (We still do not know the exact words. It could be ud+uldieda or udul+dieda or
u+duldieda, etc.)

(III) zanatnana krabim = The mother feeds the children.

The second word is the object, in this case "the children" and "krabi".

(IV) aidilsuka nanam = The neighbor visits the mother.

Looking at this sentence, we know that that "to visit" starts with the letters "ai…". Similarly,
"the neighbor" ends with "…ka".

To recap our discoveries thus far:

○ the boy = lulu


○ feeds = zanat
○ the mother = nana
○ sentence structure  v+s (first word) o+"m" (second word)
○ the cat = auti
○ to call = ud…
○ the girl = …da
○ the children = krabi
○ to visit = ai…
○ the neighbor = …ka

It’s always advisable to write the rules neatly on a clean piece of paper so that you can add
to the list as you discover new rules or vocabulary.

Now that we’ve written all the rules we can find from the given statements, let’s look at the
first question to find out more.

EXAMPLE 5

"The girl visits the cat" is expressed in the foreign language by:

(A) luluzanat auti

© 2017 edulink GmbH www.edu-link.de ALL RIGHTS RESERVED 150


Buyer: Jan Hiller (janhiller@gmx.net)
Transaction ID: 8BE25485KM511652L

Humanities, Cultural Studies and


Social Sciences Module

(B) aidildieda autim

(C) udulsuka nana

(D) aidildiedam auti

Based on our findings thus far, the answer needs to fulfill the following descriptions:

○ The first word should start with "ai…".


○ The first word should end with "…da".
○ The second word should be "autim".

Only answer B fits these criteria.

EXAMPLE 6

"The neighbor calls the children" is expressed in the foreign language by:

(A) udulsuka krabim

(B) aidilnana dieda

(C) aidilsuka krabi

(D) udulsukam krabim

Based on our findings thus far, the answer needs to fulfill the following descriptions:

○ The first word should start with "ud…".


○ The first word should end with "…ka".
○ The second word should be "krabim".

Only answer A fits these criteria.

EXAMPLE 7

"The boy visits the girl" is expressed in the foreign language by:

(A) aidiludul autim

(B) zanatlulu diedam

(C) aidillulu diedam

© 2017 edulink GmbH www.edu-link.de ALL RIGHTS RESERVED 151


Buyer: Jan Hiller (janhiller@gmx.net)
Transaction ID: 8BE25485KM511652L

Humanities, Cultural Studies and


Social Sciences Module

(D) zanatlulu dieda

Based on our findings thus far, the answer needs to fulfill the following descriptions:

○ The first word should start with "ai…"


○ The first word should end with "…lulu"
○ The second word ends with "da" plus the "m" ending for the direct object, i.e. "dam".

Only answer C fits these criteria.

We now know a new fact about the fictitious language: "to visit" means "aidil", and "the girl"
means "dieda".

To recap our discoveries so far:

○ the boy = lulu


○ feeds = zanat
○ the mother = nana
○ sentence structure  v+s (first word) o+“m” (second word)
○ the cat = auti
○ to call = ud…
○ the girl = dieda
○ the children = krabi
○ to visit = aidil
○ the neighbor = …ka

Looking at the first word of statement (II), we now know:

○ to call = udul
○ the neighbor = suka

Now let’s move on to the next question.

EXAMPLE 8

"The neighbor feeds the children and the cat" is expressed in the foreign language by:

(A) zanatlulu autim diedam

(B) zanatsuka autim

(C) zanatlulu krabi auti

© 2017 edulink GmbH www.edu-link.de ALL RIGHTS RESERVED 152


Buyer: Jan Hiller (janhiller@gmx.net)
Transaction ID: 8BE25485KM511652L

Humanities, Cultural Studies and


Social Sciences Module

(D) zanatsuka krabimsautim

Based on our findings so far, the sentence should look as follows:

○ first word is zanat+suka

○ The following words should have "krabi" (=the children) and "auti" (=the cat) in it.

Only answer D fulfills these criteria. Apparently "krabimsautim" means "the children and the
cat". Now we know 2 new rules:

○ and = s
○ If there are 2 objects connected with the word "and", these form 1 word.

The last question is very easy given all that we know.

EXAMPLE 9

"The mother visits the neighbor" is expressed in the foreign language by:

(A) zanatnana sukam

(B) aidilnana sukam

(C) zanatsuka nanam

(D) aidilsuka krabim

We can build the sentence without looking at the answer choices for further information:
aidil+nana suka+m . The answer is B.

© 2017 edulink GmbH www.edu-link.de ALL RIGHTS RESERVED 153


Buyer: Jan Hiller (janhiller@gmx.net)
Transaction ID: 8BE25485KM511652L

Humanities, Cultural Studies and


Social Sciences Module

4.5 OUR SUGGESTION FOR SOLUTION APPROACH

Below we present our suggestion for a possible solution approach. Of course, this approach
is totally subjective. If you can answer the question with a different approach, then by all
means use your own approach to solve the questions quickly.

Be sure to practice often using whatever approach you have selected. You will see that such
questions can be quickly answered once you know what to look for.

Step 1 - Look for repetition. Write down what you find out.

Read all given sentences, and look for repeating words. Write down the words and the
rule(s) that you deciphered. The fictitious languages are quite confusing, so writing will
help you keep track of the rules that you discover.

Step 2 - Eliminate the most obvious incorrect answers.

Use the discovered rules to eliminate the wrong answers. Cross these out visibly in
the question booklet to avoid confusion. (Having solved questions for the last four/five
hours, students report serious concentration problems.)

Step 3 - Look at the remaining answers to determine what to focus on. Focus on
deciphering these (for instance using the bottom-up approach explained below).

You focus on the areas that enable you to make a decision. For instance, if only two
answer choices remain and the first word is the same in both, focus on the second
word.

Step 4 - If you have already spent 2 minutes, consider eliminating wrong answers and
guessing.

While eliminating, look for quick clues in the answer choices, such as a word that
should be in a sentence but is not.

© 2017 edulink GmbH www.edu-link.de ALL RIGHTS RESERVED 154


Buyer: Jan Hiller (janhiller@gmx.net)
Transaction ID: 8BE25485KM511652L

Humanities, Cultural Studies and


Social Sciences Module

PLEASE NOTE: FOR EACH FICTITIOUS LANGUAGE THERE CAN BE A FEW


QUESTIONS. IN THESE CASES, IF YOU ARE STUCK, JUST GUESS AND MOVE
ON TO THE NEXT QUESTION. THE NEW KNOWLEDGE FROM THE NEXT
QUESTION MAY THEN HELP YOU SOLVE THE PRIOR QUESTION.

We will now show you a few example where we used this approach. After studying this, you
can come up with your own approach of how to master this question type.

EXAMPLES 10-12

(I) mitekoppo mät = The teacher talks loudly.

(II) saniixansch miteen = The student calls the teacher.

(III) kazmkoppo saot = The librarian talks fast.

(IV) Xundiperdi kazmen = The visitor annoys the librarian.

EXAMPLE 10

"The teacher calls the visitor" is expressed in the foreign language by:

(A) mitekoppo xundien

(B) xundiperdi miteen

(C) mitexansch saniien

(D) mitexansch xundien

EXAMPLE 11

"The student annoys the librarian" is expressed in the foreign language by:

(A) saniixundi saot

(B) saniiperdi kazmen

(C) saniixansch kazmen

© 2017 edulink GmbH www.edu-link.de ALL RIGHTS RESERVED 155


Buyer: Jan Hiller (janhiller@gmx.net)
Transaction ID: 8BE25485KM511652L

Humanities, Cultural Studies and


Social Sciences Module

(D) saniiperdi xundien

EXAMPLE 12

"The visitor talks fast and loudly in front of the librarian" is expressed in the foreign language
by:

(A) xundikappo kazmsaot mat

(B) xundiperdi saot mat

(C) xundikoppo kazmmät saot

(D) xundiperdi kazmsaotmat

Answer 10: D

Answer 11: B

Answer 12: C

We start with identifying the repeating words.

Step 1: repeating words

○ "the teacher" in statements (I) and (II)


○ "talks" in statements (I) and (III)
○ "the librarian" in statements (III) and (IV)
○ Based on the above, we know that “the teacher talks" means "mitekoppo".

Consequently, "loudly" must be "mät".

(I) mitekoppo mät = The teacher talks loudly.

(II) saniixansch miteen = The student calls the teacher.

(III) kazmkoppo saot = The librarian talks fast.

(IV) xundiperdi kazmen = The visitor annoys the librarian.

So we know:

© 2017 edulink GmbH www.edu-link.de ALL RIGHTS RESERVED 156


Buyer: Jan Hiller (janhiller@gmx.net)
Transaction ID: 8BE25485KM511652L

Humanities, Cultural Studies and


Social Sciences Module

○ the teacher = mite


○ talks = koppo
○ the librarian = kazm
○ loudly = mät
○ The direct objects end with “en” (compare the use of teacher and librarian as object
and subject.)
○ Sentence structure
‐ subject+verb (first word)
‐ object+”en” or adverb (second word)

In the bottom-up approach, you want to be pragmatic and solve the questions by looking at
the questions and the answer choices right away, rather than trying to first find out all the
known facts about the fictitious language. So let’s look at the 3 questions and eliminate
already what we can.

EXAMPLE 10

"The teacher calls the visitor" is expressed in the foreign language by:

(A) mitekoppo xundien

(B) xundiperdi miteen

(C) mitexansch saniien

(D) mitexansch xundien

Given the sentence structure s+v o+"en", we can eliminate all choices that do not start with
"mite" (= the teacher). Answer choice B is out.

We can also eliminate A because "koppo" (= to talk) is mentioned there.

We are left with the answers C and D. The front part is the same, so "xansch" must mean "to
talk". Let’s add this to our list of known rules. The question is whether “saniien” or "xundien"
means visitor. So we are on the hunt for the word "visitor". Looking at the given 4
statements, statement (IV) gives us the information that we need. The subject, i.e. the first
word in the fictitious language, is "xundi" which must mean "the visitor".

The correct answer is D.

Looking at statement (IV), we also now know that perdi must mean "to annoy".

To recap, we know the following and update our list accordingly:

© 2017 edulink GmbH www.edu-link.de ALL RIGHTS RESERVED 157


Buyer: Jan Hiller (janhiller@gmx.net)
Transaction ID: 8BE25485KM511652L

Humanities, Cultural Studies and


Social Sciences Module

○ the teacher = mite


○ talks = koppo
○ the librarian = kazm
○ loudly = mät
○ to call = xansch
○ the visitor = xundi
○ to annoy = perdi
○ The direct objects end with "en" (compare the use of teacher and librarian as object
and subject.)
○ Sentence structure
‐ subject+verb (first word)
‐ object+"en" or adverb (second word).

THE BOTTOM-UP APPROACH REQUIRES THAT YOU HAVE A FAIR AMOUNT


OF COMFORT WITH UNCERTAINTY. YOU MAKE YOUR DECISIONS BASED ON
WHAT YOU KNOW IS WRONG, RATHER THAN FINDING OUT WHAT’S RIGHT.
BUT IT SAVES TIME, AND TIME IS PRECIOUS IN THIS SUBTEST.

EXAMPLE 11

"The student annoys the librarian" is expressed in the foreign language by:

(A) saniixundi saot

(B) saniiperdi kazmen

(C) saniixansch kazmen

(D) saniiperdi xundien

We see that all answers start with "sanii" which probably means "the student". Before we
forget, it’s best to update our list of known rules.

We are looking for the verb "perdi" (= to annoy) in the answer choices, so "saniperdi" should
be the first word. Options A and C are eliminated.

Additionally, we know that "the librarian" means "kazm". Therefore, the second word is
"kazm+en".

Consequently, the answer is B.

To recap, we know the following and update our list accordingly:

© 2017 edulink GmbH www.edu-link.de ALL RIGHTS RESERVED 158


Buyer: Jan Hiller (janhiller@gmx.net)
Transaction ID: 8BE25485KM511652L

Humanities, Cultural Studies and


Social Sciences Module

○ the teacher = mite


○ talks = koppo
○ the librarian = kazm
○ loudly = mät
○ to call = xansch
○ the visitor = xundi
○ to annoy = perdi
○ the student = sanii
○ The direct objects end with "en" (compare the use of teacher and librarian as object
and subject.)
○ Sentence structure
‐ subject+verb (first word)
‐ object+"en" or adverb (second word)

EXAMPLE 12

"The visitor talks fast and loudly in front of the librarian" is expressed in the foreign language
by:

(A) xundikappo kazmsaot mat

(B) xundiperdi saot mat

(C) xundikoppo kazmmät saot

(D) xundiperdi kazmsaotmat

Looking at this question and our findings thus far, we expect the following words to be
included in the final sentence:

The visitor talks fast and loudly in front of the librarian.

xundi koppo … … mät …. kazm  The words will not be in this order; at this point we just
listed the correct vocabulary.

When we look at the answers, we see that only the answer choice C fulfills this requirement.

Note that a few answer choices have the word "mat", but we need "mät" with umlaut. In fact,
we expect cases like this (where the u is shown to confuse with the ü) to be in the exam to
test your concentration.

We are far from deciphering what "and" or "in front of" means in this fictitious language. But
it turns out that we don’t need this knowledge. Given that time is tight, you can quickly move
to the next set of questions.

© 2017 edulink GmbH www.edu-link.de ALL RIGHTS RESERVED 159


Buyer: Jan Hiller (janhiller@gmx.net)
Transaction ID: 8BE25485KM511652L

Humanities, Cultural Studies and


Social Sciences Module

In case you would like to have more proof for why answer C is the correct one,
let’s look at statement (III). We know that the second word is the adverb; in this
case, "fast" means "saot".

Now you need to compare the options A and C. We don’t know what "in front of"
means, but comparing A and C, we see that this fictitious language avoids the use
of prepositions. This may sound like an unrealistic rule, but remember that this is a
fictitious and overly simple language.

So we have no option but to compare A and C and notice that "mat" has the
umlauts missing in A.

Consequently, the sentence structure in this complex language looks as such:

○ first word: subject + verb


○ second word: object + first adverb OR direct object + "en"
○ third word: second adverb
○ Prepositions do not exist.

As can be seen, the bottom-up approach is oftentimes faster than the top-down, especially
with the harder questions. It comes with a weakness though. With some questions, you may
be unable to eliminate all 3 answers, and you will end up having to guess and do the
thinking to find out the necessary rule. Additionally, you need to be comfortable answering
questions based on elimination rather than constructing the answer with precise knowledge.

© 2017 edulink GmbH www.edu-link.de ALL RIGHTS RESERVED 160


Buyer: Jan Hiller (janhiller@gmx.net)
Transaction ID: 8BE25485KM511652L

Humanities, Cultural Studies and


Social Sciences Module

4.6 THE PROCESS OF ELIMINATION IS YOUR FRIEND IN THIS SECTION

This section is really draining to many students because it’s at the very end of the long exam
and because it requires heavy thinking. The average student answers only a limited number
of questions correctly. Each correct answer strongly improves your score as you are rated
compared to the performance of other students. Eliminating answers can therefore be a
powerful strategy in this section.

You can partly or fully eliminate the wrong answers without fully deciphering the questions.
When would this make sense?

○ You have slightly over 2 minutes per question. If this time has passed and you are
stuck, consider eliminating wrong answers and proceeding.

○ If you are very tired and unable to concentrate, you might also consider this
approach. The subtest "Recognising Linguistic Structures" starts after 4-5 hours of
solving the prior subtests. Moreover, most students find this question type to be
mentally draining. If you are out of patience or having a difficult time concentrating on
the questions, you can still eliminate wrong answers and guess, which is much easier
to think about than deciphering the entire ruleset.

Please remember:

○ If you leave an answer empty, you have 0% likelihood to answer the


question correctly.

○ If you randomly guess from the 4 choices, you have 25% likelihood to
answer the question correctly.

○ If you can eliminate 2 out of the 4 answer choices, your likelihood to


answer the question correctly increases to 50%.

Let’s try to solve examples 5-9 by using the elimination process. Please compare how much
faster and how much less precise you are with this method than with the top-down approach.

EXAMPLES 5-9

(I) zanatlulu autim = The boy feeds the cat.

(II) uduldieda lulum = The girl calls the boy.

© 2017 edulink GmbH www.edu-link.de ALL RIGHTS RESERVED 161


Buyer: Jan Hiller (janhiller@gmx.net)
Transaction ID: 8BE25485KM511652L

Humanities, Cultural Studies and


Social Sciences Module

(III) zanatnana krabim = The mother feeds the children.

(IV) aidilsuka nanam = The neighbor visits the mother.

So based on the repeating words, we know already the following rules. Please always write
down the rules that you discover, as you discover them, as solving these questions can be
complicated.

‐ boy = lulu

‐ feeds = zanat

‐ mother = nana

‐ sentence structure: v+s (first word) o+"m" (second word)

Let’s see how many of the wrong answers we can already eliminate. You will notice that
elimination is a much quicker process than the more detailed analysis involved with the top-
down approach. It can therefore be of great value when time is running out.

EXAMPLE 5

"The girl visits the cat" is expressed in the foreign language by:

(A) luluzanat auti

(B) aidildieda autim

(C) udulsuka nana

(D) aidildiedam auti

Looking at example 5, we can eliminate A, C and D because the object does not end with an
"m". The answer must be B.

EXAMPLE 6

"The neighbor calls the children" is expressed in the foreign language by:

(A) udulsuka krabim

(B) aidilnana dieda

(C) aidilsuka krabi

© 2017 edulink GmbH www.edu-link.de ALL RIGHTS RESERVED 162


Buyer: Jan Hiller (janhiller@gmx.net)
Transaction ID: 8BE25485KM511652L

Humanities, Cultural Studies and


Social Sciences Module

(D) udulsukam krabim

Looking at example 6, we can eliminate B and C because they lack the object ending "m".

B is also eliminated because the word "nana" (= the mother) is mentioned, though the
English sentence does not include "mother".

Now you can guess between A and D, giving you a 50% chance to guess right.

EXAMPLE 7

"The boy visits the girl" is expressed in the foreign language by:

(A) aidiludul autim

(B) zanatlulu diedam

(C) aidillulu diedam

(D) zanatlulu dieda

Looking at example 7, we can eliminate A and D because the word "lulu" (= the boy) is not
included.

B can be eliminated because "zanat" means "feeds", which does not form part of the English
sentence. The answer is C.

EXAMPLE 8

"The neighbor feeds the children and the cat" is expressed in the foreign language by:

(A) zanatlulu autim diedam

(B) zanatsuka autim

(C) zanatlulu krabi auti

(D) zanatsuka krabimsautim

Looking at example 8, we can eliminate C because it lacks the object ending "m".

A and C can be eliminated because the word "lulu" (= the boy) should not have been
included.

© 2017 edulink GmbH www.edu-link.de ALL RIGHTS RESERVED 163


Buyer: Jan Hiller (janhiller@gmx.net)
Transaction ID: 8BE25485KM511652L

Humanities, Cultural Studies and


Social Sciences Module

You need to guess between B and D, giving you a 50% likelihood to guess correctly.

EXAMPLE 9

"The mother visits the neighbor" is expressed in the foreign language by:

(A) zanatnana sukam

(B) aidilnana sukam

(C) zanatsuka nanam

(D) aidilsuka krabim

Looking at example 9, we can eliminate C and D because the word "nana" (= the mother) is
not at the end of the first word.

A is eliminated because "zanat" (="feeds") should not have been included in the sentence.
The correct answer is B.

© 2017 edulink GmbH www.edu-link.de ALL RIGHTS RESERVED 164


Buyer: Jan Hiller (janhiller@gmx.net)
Transaction ID: 8BE25485KM511652L

Humanities, Cultural Studies and


Social Sciences Module

4.7 PRACTICE QUESTIONS

In the following practice questions, you can test both approaches and determine which one
works best for you.

You have a total of 50 minutes during the TestAS exam for the 22 questions belonging to
the "Recognising Linguistic Structures" section.

4.7.1 EXAM 1

1.1.

kamatu = I drink

kamapa = she drinks

rewapa = she eats

"I eat" is expressed in the foreign language by:

(A) rewapu

(B) rewatu

(C) rewapa

(D) kawapa

1.2.-1.5.

yllömzemko = I jumped

llünzimku = He plays

yllömzutko = We will jump

yllömzitke = You jump (you in the plural)

llünzetku = They played

© 2017 edulink GmbH www.edu-link.de ALL RIGHTS RESERVED 165


Buyer: Jan Hiller (janhiller@gmx.net)
Transaction ID: 8BE25485KM511652L

Humanities, Cultural Studies and


Social Sciences Module

1.2.

"I jump" is expressed in the foreign language by:

(A) llünzemko

(B) llünzitko

(C) yllömzetko

(D) yllömzimko

1.3.

"You jump" (you in singular) is expressed in the foreign language by:

(A) yllömzimke

(B) yllömzemke

(C) llünzimke

(D) yllömzitke

1.4.

"We played" is expressed in the foreign language by:

(A) llünzumku

(B) unünzetko

(C) llünzetko

(D) llünzitko

1.5.

"She will play" is expressed in the foreign language by:

(A) llünzumkü

(B) llünzutku

© 2017 edulink GmbH www.edu-link.de ALL RIGHTS RESERVED 166


Buyer: Jan Hiller (janhiller@gmx.net)
Transaction ID: 8BE25485KM511652L

Humanities, Cultural Studies and


Social Sciences Module

(C) yllömzimkü

(D) llünzemkü

1.6.-1.9.

tekizän lide = The girl works carefully.

samcüq tekix = The dog loves the girl.

petizän ledö = The woman works fast.

allenlüb petix = The horse likes the woman.

1.6.

"The girl loves the horse" is expressed in the foreign language by:

(A) tekilüb allen

(B) allencüq teki

(C) petilüb allenx

(D) tekicüq allenx

1.7.

"The dog likes the woman" is expressed in the foreign language by:

(A) samlüb petix

(B) petilüb samx

(C) samcüq peti

(D) samlüb peti

1.8.

"The boy works carefully but fast" is expressed in the foreign language by:

© 2017 edulink GmbH www.edu-link.de ALL RIGHTS RESERVED 167


Buyer: Jan Hiller (janhiller@gmx.net)
Transaction ID: 8BE25485KM511652L

Humanities, Cultural Studies and


Social Sciences Module

(A) maguzän lide op ledö

(B) magulüb lide op ledö

(C) magucüq lide op

(D) maguzän lide op peti

1.9.

"The woman likes the horse, and the girl likes the woman" is expressed in the foreign
language by:

(A) petizän lüb og tekilüb peti

(B) petilüb allenx og tekilüb petix

(C) teki og petix zänlüb

(D) peticüq zäni og tekicüq petix

1.10.

oreta tarti neducas = The dog shakes its tail

geno rasa endomino = The cat eats the rat

rasa soque endominopo = The rat ate the cheese

borme oretate nalitomi = The man feeds his dog

"The man fed his cat" is expressed in the foreign language by:

(A) borme genote nalitomi

(B) nalitomipo genote borme

(C) borme genote nalitomipo

(D) borme geno nalitomipo

© 2017 edulink GmbH www.edu-link.de ALL RIGHTS RESERVED 168


Buyer: Jan Hiller (janhiller@gmx.net)
Transaction ID: 8BE25485KM511652L

Humanities, Cultural Studies and


Social Sciences Module

1.11.

dekerambu muffer reetan = The mother noticed the father

hasarte reetan fundary = The father is introducing his friends

delukke chastmann gundans = The business man invited the customers

kerambu zimanfer gundan = The shopkeeper is noticing the customer

"The customer is inviting the father" is expressed in the foreign language by:

(A) delukke gundan reetan

(B) lukke gundans reetan

(C) delukke gundan muffer

(D) lukke gundan reetan

1.12.

iredingey relasky negart = The mothers are carrying their children

irenesaty volegy bloos = The birds are building their nests

oditas rubad moulli = The master is coaching the student

voleg dinge repyet = The child is feeding the bird

"The students are carrying their bags" is expressed in the foreign language by:

(A) ireflimky oditasy negart

(B) irenesat oditasy negart

(C) ireflimka negart oditasy

(D) flimka dinge bloos

1.13.

trabug lenos ta nikal = The uncle is advising his son

veralu pa ta jannal ta nos = The son was painting the room

© 2017 edulink GmbH www.edu-link.de ALL RIGHTS RESERVED 169


Buyer: Jan Hiller (janhiller@gmx.net)
Transaction ID: 8BE25485KM511652L

Humanities, Cultural Studies and


Social Sciences Module

thundan levalur ta pikkay = The seller is punishing his worker

trabug pa ta manigar ta regged = The principal was advising the students

"The worker is painting his room" is expressed in the foreign language by:

(A) veralu ta jannal ta valur

(B) valur ta jannal veralu

(C) veralu lejannal ta valur

(D) veralu pa lejannal ta valur

1.14.-1.16.

koto lemüx = The man works always.

kata qilem = The men will work.

loso qilemu = The boy will never work.

lemu kata = Do the men never work?

qipenüx fala = Will the students always study?

1.14.

"Does the man work rarely?" is expressed in the foreign language by:

(A) koto zölem

(B) lemax koto

(C) qilemzö kota

(D) koto qiüxlem

1.15.

"The boy studies always" is expressed in the foreign language by:

(A) loso penüx

© 2017 edulink GmbH www.edu-link.de ALL RIGHTS RESERVED 170


Buyer: Jan Hiller (janhiller@gmx.net)
Transaction ID: 8BE25485KM511652L

Humanities, Cultural Studies and


Social Sciences Module

(B) loso qiüxlem

(C) qiüxpen loso

(D) qipenüx loso

1.16.

"Will the students never work?" is expressed in the foreign language by:

(A) lemüx na üxlem fala

(B) qilemu fala

(C) fala zölem na üxlem

(D) lemzö na zölem fala

1.17.

Bodalpilla fib = The policeman is checking

Cattinfib ja poth = The man is carrying his book

Bodalgompat noyali = The doctor is checking the patient

Parogilpoth = The book is falling

"The doctor is carrying his patient" is expressed in the foreign language by:

(A) Cattingompat noyali

(B) Parogilgompat ja noyali

(C) Cattinoyali ja gompat

(D) Cattingompat ja noyali

1.18.

The dog is shaking its tail = torarkarooš uvas itsa

The cat is eating a lot = sijakēf a kul

© 2017 edulink GmbH www.edu-link.de ALL RIGHTS RESERVED 171


Buyer: Jan Hiller (janhiller@gmx.net)
Transaction ID: 8BE25485KM511652L

Humanities, Cultural Studies and


Social Sciences Module

The dog was eating the beef = nusijakarooš kurees

The servant is feeding = merašponta

"The dog was feeding the cat" is expressed in the foreign language by:

(A) numerašponta ēf

(B) numeraškarooš itsa

(C) meraškarooš ēf

(D) numeraškarooš ēf

1.19.

The driver is not sitting on the chair = Putuwani Otunar+ukkar

The painter is not drawing with a pencil = Gondaluda Oaviyan+varaiyu

The pencil is falling on the floor = Nilamni Gondaluwilupa

The man is falling with the table = Mesatda Tanmawilupa

"The driver is not drawing on the table" is expressed in the foreign language by:

(A) Nilamni Otunar+varaiyu

(B) Mesatda Otunarvaraiyu

(C) Mesatni Otunar+varaiyu

(D) Mesatni Oviyanvaraiyu

1.20.-1.22.

orisvar got = The cook cooks pasta.

mesvar as = I cook meat.

mespets got = The cook fries the meat.

pyln gotten = The cooking pot is full.

udo pets as = I fry frequently.

© 2017 edulink GmbH www.edu-link.de ALL RIGHTS RESERVED 172


Buyer: Jan Hiller (janhiller@gmx.net)
Transaction ID: 8BE25485KM511652L

Humanities, Cultural Studies and


Social Sciences Module

pyln bul bulten = The soup pot is full of soup.

pras gotten = The cooking pot is empty.

1.20.

"The cook cooks with the cooking pot" is expressed in the foreign language by:

(A) gotvar xogotten

(B) xogottenvar got

(C) xogottenvar gotten

(D) bultenvar got

1.21.

"I fry meat frequently" is expressed in the foreign language by:

(A) mespets udoni

(B) udo mespets as

(C) udo mesvar as

(D) udo petsmes as

1.22.

"meszuorisvar got" is expressed in the foreign language by:

(A) In the cooking pot meat and pasta cook.

(B) I frequently cook meat and pasta.

(C) The cook cooks meat and pasta.

(D) Fried meat with pasta

© 2017 edulink GmbH www.edu-link.de ALL RIGHTS RESERVED 173


Buyer: Jan Hiller (janhiller@gmx.net)
Transaction ID: 8BE25485KM511652L

Humanities, Cultural Studies and


Social Sciences Module

4.7.2 EXAM 2

2.1.

Ont tetraw = I am waiting

En letips = He is playing

En tetrawi = He was waiting

"I was playing" is expressed in the foreign language by:

(A) En letipsi

(B) Ont tetrawi

(C) Ont letopso

(D) Ont letipsi

2.2.-2.4.

qalogut zo = I drove

qelogut = you drive (you in singular)

qamopu = I sleep

2.2.

"You slept" (in singular) is expressed in the foreign language by:

(A) qamoput zo

(B) qelogut

(C) qamopu

(D) qemopu zo

2.3.

"Do you go?" (you in singular) is expressed in the foreign language by:

© 2017 edulink GmbH www.edu-link.de ALL RIGHTS RESERVED 174


Buyer: Jan Hiller (janhiller@gmx.net)
Transaction ID: 8BE25485KM511652L

Humanities, Cultural Studies and


Social Sciences Module

(A) tolutqe zo

(B) tolummqe

(C) toluqa maq

(D) qeloqu zo

2.4.

"I drive and you sleep" (you in singular) is expressed in the foreign language by:

(A) qalogut zo qemoput

(B) qelogut zö qamopu

(C) qelogut zö qamopu

(D) qalogut zö qemopu

2.5.

moloztipe boto xoxiq = The architect draws rapidly.

lumetepi moloztipe xax = The painter calls the architect.

maleztipe boto xuxy = The intern writes rapidly.

lumeztipe maleztipe xüxlim = The painters annoy the intern.

"The architect calls the painter" is expressed in the foreign language by:

(A) lumetepi xüxlim moloztipe

(B) moloztepi lumetipe xüxlim

(C) moloztipe lumetepi xax

(D) lumetepi molozlipe xax

2.6.

I closed = thechai

© 2017 edulink GmbH www.edu-link.de ALL RIGHTS RESERVED 175


Buyer: Jan Hiller (janhiller@gmx.net)
Transaction ID: 8BE25485KM511652L

Humanities, Cultural Studies and


Social Sciences Module

he closes = thecheu

I blame = culxoi

"He blamed" is expressed in the foreign language by:

(A) culxoeu

(B) culxoieu

(C) thechaeu

(D) culxoaeu

2.7.

qaramos = We wash

twouxram = They bring

qarauram = They washed

"We brought" is expressed in the foreign language by:

(A) twouxmos

(B) qararam

(C) twouxumos

(D) twouxram

2.8.-2.11.

ba ?zuxöm lem = will he read?

kito bao = she writes

kito nep ba = he wrote

?kito lem bao = she will write

© 2017 edulink GmbH www.edu-link.de ALL RIGHTS RESERVED 176


Buyer: Jan Hiller (janhiller@gmx.net)
Transaction ID: 8BE25485KM511652L

Humanities, Cultural Studies and


Social Sciences Module

2.8.

"We will read" is expressed in the foreign language by:

(A) zuxöm baoy

(B) ?zuxöm lem baoy

(C) zuxöm ?lem baoy

(D) ?kito lem

2.9.

"Did they write?" is expressed in the foreign language by:

(A) baoz kito nep

(B) baoz kito

(C) baoz zuxöm nep

(D) ?kito lem baz

2.10.

"He painted" is expressed in the foreign language by:

(A) kito ba

(B) sipo nep ba

(C) zuxöm nep ba

(D) kittom lem ba

2.11.

"She reads and he will write" is expressed in the foreign language by:

(A) zuxöm bao toki ?kito lem ba

© 2017 edulink GmbH www.edu-link.de ALL RIGHTS RESERVED 177


Buyer: Jan Hiller (janhiller@gmx.net)
Transaction ID: 8BE25485KM511652L

Humanities, Cultural Studies and


Social Sciences Module

(B) zuxöm bao ?toki kito lem ba

(C) zuxöm ba toki ?kito ba

(D) kito bao toki lem bam

2.12.

Desser ple = I am working

Elbisso gim = she is talking

ha og gi = he and she

"He is talking and working" is expressed in the foreign language by:

(A) ham Elbisso og Desser

(B) Elbisso og Desser ha

(C) Elbisso og Desser ham

(D) ha Elbisso og Desser

2.13.

Zopli lia odandig tausa = The policeman is driving the car

Ozagar domna tausaro = The boy is giving his car

Codimac lonatcon napenti = The doctor is checking the patient

napenti mendaco zopli lia = The patient is calling the policeman

"The doctor is calling his patient" is expressed in the foreign language by:

(A) Codimac mendaco napentiro

(B) Codimac mendaco napenti

(C) napenti mendaco codimac

(D) Ozagar mendaco tausaro

© 2017 edulink GmbH www.edu-link.de ALL RIGHTS RESERVED 178


Buyer: Jan Hiller (janhiller@gmx.net)
Transaction ID: 8BE25485KM511652L

Humanities, Cultural Studies and


Social Sciences Module

2.14.

Tū borplokkē rumpt = The uncle is advising his son

Lossēz boxty borp = The son was painting beautifully

Tū mantūpthadam untēad = The professor is motivating his student

mantūpLokkēz lenkēm = The principal was advising the student

"The principal was motivating his son" is expressed in the foreign language by:

(A) Tū rumptthadamz lenkēm

(B) Tū borpthadamz lenkēm

(C) borpthadamz lenkēm

(D) Tū borpthadam lenkēm

2.15.-2.17.

lomvelz roxalk nep = The cat is black.

lomnoly boking nep = The lawn is green.

lamvoxs peny komolx tos = The boy always plays soccer.

lemrasx peny minorz nep = The horse is always tired.

2.15.

"The boy is never tired" is expressed in the foreign language by:

(A) lemvoxs peny minorz nep

(B) lamvoxs nep tilute minorz

(C) lemrasx tilute minorz nep

(D) lamvoxs tilute minorz nep

© 2017 edulink GmbH www.edu-link.de ALL RIGHTS RESERVED 179


Buyer: Jan Hiller (janhiller@gmx.net)
Transaction ID: 8BE25485KM511652L

Humanities, Cultural Studies and


Social Sciences Module

2.16.

"The black horse plays on the lawn" is expressed in the foreign language by:

(A) roxalklamrasx lomnoly tos

(B) lemrasxroxalk lomnoly tos

(C) lamnoly lemrasxroxalk tos

(D) lemrasx roxalk nep lomnaly tos

2.17.

"The cat and the boy are tired" is expressed in the foreign language by:

(A) neplo roxalk lomvelz lamvoxs minorz

(B) minorz leamvelz lomvoxs neplo

(C) lomvelz at lamvoxs minorz neplo

(D) lemrasx at lamvoxs minorz neplo

2.18.

lupolani mo = She is telling nothing.

polanihuwi tum? = Are they telling a lot?

kipola joi = I will tell everything.

huwiristyni sur = They are travelling everywhere.

"Will she travel a lot?" is expressed in the foreign language by:

(A) luristy tum?

(B) ristynilu tum?

(C) ristylu tum?

(D) ristylu joi?

© 2017 edulink GmbH www.edu-link.de ALL RIGHTS RESERVED 180


Buyer: Jan Hiller (janhiller@gmx.net)
Transaction ID: 8BE25485KM511652L

Humanities, Cultural Studies and


Social Sciences Module

2.19.-2.22.

momle lomanozete rokoz = The father helps his son.

naqumi katoze talokoz = The mother greets the dog.

molde katozete sumolüm = The student hugs his dog.

ronume molde ranilüm = The professor likes the student.

2.19.

"The son greets his professor" is expressed in the foreign language by:

(A) rokoz talulüm ronume

(B) naqumi ronumeti talokoz

(C) lomanoze ronumete taloklüm

(D) lomanoze ronumete talokoz

2.20.

"The mother likes her dog" is expressed in the foreign language by:

(A) naqumi katozete ranilüm

(B) katozete naqumite rokozlüm

(C) naqumiti katozete ranilüm

(D) talokoz naqumite katozeti

2.21.

"The professor hugs the student and her son" is expressed in the foreign language by:

(A) moldexümoldi ronume sumokoz

(B) ronume moldexütakonete sumolüm

(C) lomanozexükatoze ronume sumolüm

© 2017 edulink GmbH www.edu-link.de ALL RIGHTS RESERVED 181


Buyer: Jan Hiller (janhiller@gmx.net)
Transaction ID: 8BE25485KM511652L

Humanities, Cultural Studies and


Social Sciences Module

(D) ronume moldexütakonete ronume

2.22.

"The boy greets his dog" is expressed in the foreign language by:

(A) ramilüm katozete takone

(B) romme katoze talokoz

(C) takone katozete talokoz

(D) katozete talokoz takone

© 2017 edulink GmbH www.edu-link.de ALL RIGHTS RESERVED 182


Buyer: Jan Hiller (janhiller@gmx.net)
Transaction ID: 8BE25485KM511652L

Humanities, Cultural Studies and


Social Sciences Module

4.8 ANSWER KEY

Exam 1 Exam 2

Question Answer Question Answer

1.1. B 2.1. D

1.2. D 2.2. D

1.3. A 2.3. B

1.4. C 2.4. A

1.5. A 2.5. C

1.6. D 2.6. D

1.7. A 2.7. C

1.8. A 2.8. B

1.9. B 2.9. A

1.10. C 2.10. B

1.11. D 2.11. A

1.12. A 2.12. C

1.13. C 2.13. A

1.14. B 2.14. B

1.15. A 2.15. D

1.16. B 2.16. B

1.17. D 2.17. C

1.18. D 2.18. C

1.19. C 2.19. D

1.20. B 2.20. A

1.21. B 2.21. B

1.22. C 2.22. C

© 2017 edulink GmbH www.edu-link.de ALL RIGHTS RESERVED 183


Buyer: Jan Hiller (janhiller@gmx.net)
Transaction ID: 8BE25485KM511652L

Humanities, Cultural Studies and


Social Sciences Module

4.9 DETAILED ANSWERS

4.9.1 EXAM 1

1.1.

kamatu = I drink

kamapa = she drinks

rewapa = she eats

"I eat" is expressed in the foreign language by:

(A) rewapu

(B) rewatu

(C) rewapa

(D) kawapa

Answer B

Verb: to eat = rewa. Consequently, the possible answer choices are A, B and C.

Personal pronoun: I = tu. The correct answer is B.

1.2.-1.5.

yllömzemko = I jumped

llünzimku = He plays

yllömzutko = We will jump

yllömzitke = You jump (you in the plural)

llünzetku = They played

© 2017 edulink GmbH www.edu-link.de ALL RIGHTS RESERVED 184


Buyer: Jan Hiller (janhiller@gmx.net)
Transaction ID: 8BE25485KM511652L

Humanities, Cultural Studies and


Social Sciences Module

1.2.

"I jump" is expressed in the foreign language by:

(A) llünzemko

(B) llünzitko

(C) yllömzetko

(D) yllömzimko

Answer D

Verb: to jump = yllöm (It can also be yllömz, but either way the elimination process and the
solution remain the same.) Possible answer choices: C, D

Verb tense: present tense = (-zi-) It can also be -i-, but either way the elimination process
and the solution remain the same. Past tense = (-ze-).

Personal pronoun: If we look at the first sentence in the question, we can conclude that the
last three letters of the word show us the personal pronoun (as we deciohered the
remainung letters). I = mko. The answer must be D.

Additional clues:

○ Sentence structure: Verb + Tense + Subject

1.3.

"You jump" (you in singular) is expressed in the foreign language by:

(A) yllömzimke

(B) yllömzemke

(C) llünzimke

(D) yllömzitke

Answer A

Verb: to jump = yllöm. Possible answer choices: A, B, D

Verb tense: Present tense = (-zi-). Possible answer choices (remaining): A, D

© 2017 edulink GmbH www.edu-link.de ALL RIGHTS RESERVED 185


Buyer: Jan Hiller (janhiller@gmx.net)
Transaction ID: 8BE25485KM511652L

Humanities, Cultural Studies and


Social Sciences Module

Personal pronoun: you (plural) = tke. D can be eliminated. The correct answer must be A.

Additional clues:

○ This also means: you (singular) = mke.

1.4.

"We played" is expressed in the foreign language by:

(A) llünzumku

(B) unünzetko

(C) llünzetko

(D) llünzitko

Answer C

Verb: to play = llün. Possible answer choices: A, C, D

Personal pronoun: we = tko. Possible answer choices (remaining): C, D

Verb tense: past tense = (-ze-). The correct answer is C.

1.5.

"She will play" is expressed in the foreign language by:

(A) llünzumkü

(B) llünzutku

(C) yllömzimkü

(D) llünzemkü

Answer A

Verb: play = llün. Possible answer choices: A, B, D

© 2017 edulink GmbH www.edu-link.de ALL RIGHTS RESERVED 186


Buyer: Jan Hiller (janhiller@gmx.net)
Transaction ID: 8BE25485KM511652L

Humanities, Cultural Studies and


Social Sciences Module

Sentence structure: Verb + Tense + Subject (as deciphered in question 1.2.). We can now
solve for the personal pronoun she which is found at the end of the word.

Personal pronoun: she = mkü. Possible answer choices (remaining): A, D

Verb tense: future tense = (-zu-). The correct answer is A.

1.6.-1.9.

tekizän lide = The girl works carefully.

samcüq tekix = The dog loves the girl.

petizän ledö = The woman works fast.

allenlüb petix = The horse likes the woman.

1.6.

"The girl loves the horse" is expressed in the foreign language by:

(A) tekilüb allen

(B) allencüq teki

(C) petilüb allenx

(D) tekicüq allenx

Answer D

Verb: to love = cüq. Possible answer choices: B, D

Object: the horse = allen –x (accusative). Possible answer choices: B, D

Subject: the girl = teki.

Sentence structure: Subject + Verb  Adverb. The correct answer is D because "the girl"
needs to be at the begining of the answer.

© 2017 edulink GmbH www.edu-link.de ALL RIGHTS RESERVED 187


Buyer: Jan Hiller (janhiller@gmx.net)
Transaction ID: 8BE25485KM511652L

Humanities, Cultural Studies and


Social Sciences Module

1.7.

"The dog likes the woman" is expressed in the foreign language by:

(A) samlüb petix

(B) petilüb samx

(C) samcüq peti

(D) samlüb peti

Answer A

Verb: to like = lüb. Possible answer choices: A, B, D

Object: the woman = peti-x (accusative). The correct answer is A.

Additional clues:

○ Subject: the dog = sam.

1.8.

"The boy works carefully but fast" is expressed in the foreign language by:

(A) maguzän lide op ledö

(B) magulüb lide op ledö

(C) magucüq lide op

(D) maguzän lide op peti

Answer A

Verb: to work = zän. Possible answer choices: A, D

Adverb: carefully, fast = lide, ledö. Remaining answer: A

Additional clues:

○ Subject: the boy = magu.

○ "but" = "op"

© 2017 edulink GmbH www.edu-link.de ALL RIGHTS RESERVED 188


Buyer: Jan Hiller (janhiller@gmx.net)
Transaction ID: 8BE25485KM511652L

Humanities, Cultural Studies and


Social Sciences Module

PLEASE NOTE

In the case of some questions, the right solution can be deduced at a relatively early
stage. In the above example, you can deduce the correct the answer at a relatively
early stage (after deciphering the verb and the adverb.) In our answer explanations
we continued the analysis of the langage for practice purposes. These steps are
actually not necessary. During the test, once you have the correct answer, you can
move to the next task immedately.

1.9.

"The woman likes the horse, and the girl likes the woman" is expressed in the foreign
language by:

(A) petizän lüb og tekilüb peti

(B) petilüb allenx og tekilüb petix

(C) teki og petix zänlüb

(D) peticüq zäni og tekicüq petix

Answer B

Verb: to like = lüb. Possible answer choices: A, B, C

Object: the horse, the woman = allen-x, peti-x (accusative). The correct answer is B.

Additional clues:

○ Subject: the woman, the girl = peti, teki.

○ "and" = "og".

1.10.

oreta tarti neducas = The dog shakes its tail

© 2017 edulink GmbH www.edu-link.de ALL RIGHTS RESERVED 189


Buyer: Jan Hiller (janhiller@gmx.net)
Transaction ID: 8BE25485KM511652L

Humanities, Cultural Studies and


Social Sciences Module

geno rasa endomino = The cat eats the rat

rasa soque endominopo = The rat ate the cheese

borme oretate nalitomi = The man feeds his dog

"The man fed his cat" is expressed in the foreign language by:

(A) borme genote nalitomi

(B) nalitomipo genote borme

(C) borme genote nalitomipo

(D) borme geno nalitomipo

Answer C

The question sentence is quite similar to the fourth sentence. Only 2 things differ: the object
(cat vs. dog) and the tense.

Sentence structure: Let’s first focus on the word dog because it repeats in the first and the
fourth sentence and is the only commonality between the two sentences. oreta = dog and
his = (-te). In the first sentence, dog is the subject and forms the first word in the foreign
language. In the fourth sentence, it is the object and is found in the second word.
Consequently the word order of the foreign language is Subject  Object  Verb.

Subject: Looking at the fourth sentence, the man = borme. Possible answer choices: A, C
and D.

Object: To find out what "his cat" means, we look at the first sentence. cat = gano. his cat =
ganote. Possible answer choices: A and C.

Verb: to feed = nalitomi (as seen in sentence four). In the question we are asked for the past
tense and given answer choice A has the present tense, the answer must be C.

Tense: To find out the equivalent of "fed", we can evaluate the verbs of the second and third
sentences. eats = endomino. ate = endominopo. This means that "po" has been added to
form the past tense. fed = nalitomipo.

1.11.

dekerambu muffer reetan = The mother noticed the father

© 2017 edulink GmbH www.edu-link.de ALL RIGHTS RESERVED 190


Buyer: Jan Hiller (janhiller@gmx.net)
Transaction ID: 8BE25485KM511652L

Humanities, Cultural Studies and


Social Sciences Module

hasarte reetan fundary = The father is introducing his friends

delukke chastmann gundans = The business man invited the customers

kerambu zimanfer gundan = The shopkeeper is noticing the customer

"The customer is inviting the father" is expressed in the foreign language by:

(A) delukke gundan reetan

(B) lukke gundans reetan

(C) delukke gundan muffer

(D) lukke gundan reetan

Answer D

Sentence structure: According to the first and second sentences the word order of the
foreign language is: Verb  Subject  Object.

Subject: the customer = gundan. A, C and D could be correct.

Tense: We need to find out what "is inviting" is in the foreign language. Consider the verbs of
the first and fourth sentences. is noticing = kerambu and noticed = dekerambu. The past
tense is constructed by adding the letters "de" at the beginning of the word.

Verb: "is inviting" corresponds to "lukke". D is the correct answer.

1.12.

iredingey relasky negart = The mothers are carrying their children

irenesaty volegy bloos = The birds are building their nests

oditas rubad moulli = The master is coaching the student

voleg dinge repyet = The child is feeding the bird

"The students are carrying their bags" is expressed in the foreign language by:

(A) ireflimky oditasy negart

(B) irenesat oditasy negart

© 2017 edulink GmbH www.edu-link.de ALL RIGHTS RESERVED 191


Buyer: Jan Hiller (janhiller@gmx.net)
Transaction ID: 8BE25485KM511652L

Humanities, Cultural Studies and


Social Sciences Module

(C) ireflimka negart oditasy

(D) flimka dinge bloos

Answer A

In this question there are only few repeating words. We need to focus on these: their,
child/children, bird/birds. (If we are stuck, we can also look for clues for the word "the
student" which occurs in the third sentence as well as the correct answer choice.)

Subject: Consider the word "bird" in the second and fourth sentences: Bird = voleg and Birds
= volegy (a letter "y" has been added at the end of the word to make it plural). Child = dinge
and Children = dingey. Given "the students" is plural, we know that this word needs to end
with the letter y. Possible answer choices: A, B, C.

Sentence structure: If we analyse the second and fourth sentences where birld/birds repeat,
we can conclude the following word order: Object  Subject  Verb. So we are looking for
an answer where both the first and the second words end with the letter "y". The correct
answer is A.

Additional clues:

○ Verb: are carrying = negart.

○ Additional clues: Their children = iredingerd. Their nests = irenesat. The common 3
letters in both words "ire" must mean "their".

1.13.

trabug lenos ta nikal = The uncle is advising his son

veralu pa ta jannal ta nos = The son was painting the room

thundan levalur ta pikkay = The seller is punishing his worker

trabug pa ta Manigar ta regged = The principal was advising the students

"The worker is painting his room" is expressed in the foreign language by:

(A) veralu ta jannal ta valur

(B) valur ta jannal veralu

(C) veralu lejannal ta valur

© 2017 edulink GmbH www.edu-link.de ALL RIGHTS RESERVED 192


Buyer: Jan Hiller (janhiller@gmx.net)
Transaction ID: 8BE25485KM511652L

Humanities, Cultural Studies and


Social Sciences Module

(D) veralu pa lejannal ta valur

Answer C

Sentence structure: Verb  Object  Subject (according to sentences 1 and 2)

Consider the subject of all 4 sentences in both languages. "The" is there in English, while
"ta" is in the foreign language. So, "the" means "ta".

Subject: The worker = ta valur. (Consider the object of first and third sentences. The
common letters "le", which means "his". So “worker” means “valur”.)

So, possible answer choices: A, C and D

Verb:

○ is painting = consider the verbs of the first and fourth sentences

○ is advising = trabug

○ was advising = trabug pa. The word "pa" has been added after trabug to make it past
tense.

○ is painting = veralu. The possible answer choices (remaining): A and C.

Object: his room = lejannal. The correct answer is C.

1.14.-1.16.

koto lemüx = The man works always.

kata qilem = The men will work.

loso qilemu = The boy will never work.

lemu kata = Do the men never work?

qipenüx fala = Will the students always study?

1.14.

"Does the man work rarely?" is expressed in the foreign language by:

© 2017 edulink GmbH www.edu-link.de ALL RIGHTS RESERVED 193


Buyer: Jan Hiller (janhiller@gmx.net)
Transaction ID: 8BE25485KM511652L

Humanities, Cultural Studies and


Social Sciences Module

(A) koto zölem

(B) lemax koto

(C) qilemzö kota

(D) koto qiüxlem

Answer B

Verb: to work = lem. Possible answer choices: A, B, C, D

Subject: the man = koto. Possible answer choices: A, B, D

Verb tense: Present tense. Possible answer choices: A, B

Sentence structure:

○ There are two words. Subject (first word)  Tense + Verb + Adverb (second word).
This structure is applicable for sentences which are not question.

○ In case of questions, the sequence of the words is switched. Tense + Verb + Adverb
 Subject (second word). The correct answer is B.

1.15.

"The boy studies always" is expressed in the foreign language by:

(A) loso penüx

(B) loso qiüxlem

(C) qiüxpen loso

(D) qipenüx loso

Answer A

Verb: to study = pen. Possible answer choices: A, C, D

Subject: the boy = loso. Possible answer choices (remaining): A, C, D

Verb tense: present tense. The correct is answer is A.

Additional clues:

© 2017 edulink GmbH www.edu-link.de ALL RIGHTS RESERVED 194


Buyer: Jan Hiller (janhiller@gmx.net)
Transaction ID: 8BE25485KM511652L

Humanities, Cultural Studies and


Social Sciences Module

○ always = üx.

1.16.

"Will the students never work?" is expressed in the foreign language by:

(A) lemüx na üxlem fala

(B) qilemu fala

(C) fala zölem na üxlem

(D) lemzö na zölem fala

Answer B

Verb: to work = lem. Possible answer choices: A, B, C, D

Subject: the students = fala. Possible answer choices: A, B, C, D

Verb tense: future tense = -qi-. The correct is answer is B.

Additional clues:

○ never = -u.

1.17.

Bodalpilla fib = The policeman is checking

Cattinfib ja poth = The man is carrying his book

Bodalgompat noyali = The doctor is checking the patient

Parogilpoth = The book is falling

"The doctor is carrying his patient" is expressed in the foreign language by:

(A) Cattingompat noyali

(B) Parogilgompat ja noyali

© 2017 edulink GmbH www.edu-link.de ALL RIGHTS RESERVED 195


Buyer: Jan Hiller (janhiller@gmx.net)
Transaction ID: 8BE25485KM511652L

Humanities, Cultural Studies and


Social Sciences Module

(C) Cattinoyali ja gompat

(D) Cattingompat ja noyali

Answer D

Verb: bodal = checking

cattin = is carrying (policeman = pilla fib). Consequently, the possible answer choices
are A, C and D.

Subject: the doctor = gompat. The remaining possible answer choices are A and D.

Object: patient = noyali

Possessive pronoun: his = ja. According to the second and fourth sentences, "poth" means
"book". Consequently, "ja" must mean his. The correct answer is D.

1.18.

The dog is shaking its tail = torarkarooš uvas itsa

The cat is eating a lot = sijakēf a kul

The dog was eating the beef = nusijakarooš kurees

The servant is feeding = merašponta

"The dog was feeding the cat" is expressed in the foreign language by:

(A) numerašponta ēf

(B) numeraškarooš itsa

(C) meraškarooš ēf

(D) numeraškarooš ēf

Answer D

Verb: sijak = eating (see sentences 2 and 3)

Subject: arooš = the dog (see sentences 1 and 3)

Consequently: nu = was. Possible answer choices are B and D.

Object: the cat = ēf (see sentence 2). The answer is D.

© 2017 edulink GmbH www.edu-link.de ALL RIGHTS RESERVED 196


Buyer: Jan Hiller (janhiller@gmx.net)
Transaction ID: 8BE25485KM511652L

Humanities, Cultural Studies and


Social Sciences Module

1.19.

The driver is not sitting on the chair = Putuwani Otunar+ukkar

The painter is not drawing with a pencil = Gondaluda Oaviyan+varaiyu

The pencil is falling on the floor = Nilamni Gondaluwilupa

The man is falling with the table = Mesatda Tanmawilupa

"The driver is not drawing on the table" is expressed in the foreign language by:

(A) Nilamni Otunar+varaiyu

(B) Mesatda Otunarvaraiyu

(C) Mesatni Otunar+varaiyu

(D) Mesatni Oviyanvaraiyu

Answer C

Sentence structure: Object  Subject  Verb (see sentences 2, 3 and 4)

Preposition: on = ni (from the first and third sentences). The possible answer choices are A,
C and D.

Negative form: not = + (between the subject and verb). The possible answer choices are A
and C.

Subject: The driver = Otunar. The possible answer choices continue to be A and C.

Object: To find out what the "the table" means, consider the second and fourth sentences.
The only similar letters of the series is "da", which means "with". the table = Mesat. C is the
correct answer.

1.20.-1.22.

orisvar got = The cook cooks pasta.

mesvar as = I cook meat.

mespets got = The cook fries the meat.

© 2017 edulink GmbH www.edu-link.de ALL RIGHTS RESERVED 197


Buyer: Jan Hiller (janhiller@gmx.net)
Transaction ID: 8BE25485KM511652L

Humanities, Cultural Studies and


Social Sciences Module

pyln gotten = The cooking pot is full.

udo pets as = I fry frequently.

pyln bul bulten = The soup pot is full of soup.

pras gotten = The cooking pot is empty.

1.20.

"The cook cooks with the cooking pot" is expressed in the foreign language by:

(A) gotvar xogotten

(B) xogottenvar got

(C) xogottenvar gotten

(D) bultenvar got

Answer B

Verb: to cook = var. Possible answer choices: A, B, C, D

Object: the cooking pot = gotten. Possible answer choices: A, B, C

Subject: the cook = got. Possible answer choices: A, B

Verb tense: present tense. Possible answer choices: A, B

with = xo. Possible answer choices (remaining): A, B

Sentence structure: The subject is always at the end of the sentence. The correct answer is
B.

1.21.

"I fry meat frequently" is expressed in the foreign language by:

(A) mespets udoni

(B) udo mespets as

© 2017 edulink GmbH www.edu-link.de ALL RIGHTS RESERVED 198


Buyer: Jan Hiller (janhiller@gmx.net)
Transaction ID: 8BE25485KM511652L

Humanities, Cultural Studies and


Social Sciences Module

(C) udo mesvar as

(D) udo petsmes as

Answer B

Verb: to fry = pets. Possible answer choices: A, B, D

Object: meat = mes. Possible answer choices: A, B, D

Personal pronoun: I = as. Possible answer choices (remaining): B, D

Adverb: frequently = udo. Possible answer choices (remaining): B, D.

Sentence structure: The object is always at the beginning of the sentence. The correct
answer is B.

1.22.

"meszuorisvar got" is expressed in the foreign language by:

(A) In the cooking pot meat and pasta cook.

(B) I frequently cook meat and pasta.

(C) The cook cooks meat and pasta.

(D) Fried meat with pasta

Answer C

Verb: var = to cook. Possible answer choices: A, B, C.

Object: mes, oris = meat, pasta. Possible answer choices: A, B, C.

Subject: got = the cook. The correct answer is C.

Additional clues:

○ zu = and.

© 2017 edulink GmbH www.edu-link.de ALL RIGHTS RESERVED 199


Buyer: Jan Hiller (janhiller@gmx.net)
Transaction ID: 8BE25485KM511652L

Humanities, Cultural Studies and


Social Sciences Module

4.9.2 EXAM 2

2.1.

Ont tetraw = I am waiting

En letips = He is playing

En tetrawi = He was waiting

"I was playing" is expressed in the foreign language by:

(A) En letipsi

(B) Ont tetrawi

(C) Ont letopso

(D) Ont letipsi

Answer D

Personal pronoun: He = En.

Verb:

○ am waiting = tetraw

○ was waiting = tetrawi. You need add "i" at the end of the verb for the past tense.

○ is playing = letips

○ was playing = letipsi. The possible answer choices are A and D.

Personal pronoun: I = ont. The answer is D.

2.2.-2.4.

qalogut zo = I drove

qelogut = you drive (you in singular)

qamopu = I sleep

© 2017 edulink GmbH www.edu-link.de ALL RIGHTS RESERVED 200


Buyer: Jan Hiller (janhiller@gmx.net)
Transaction ID: 8BE25485KM511652L

Humanities, Cultural Studies and


Social Sciences Module

2.2.

"You slept" (in singular) is expressed in the foreign language by:

(A) qamoput zo

(B) qelogut

(C) qamopu

(D) qemopu zo

Answer D

Personal pronoun: I = qa. (Compare the first and third sentences for repetitions.) We can
eliminate the answer choices that start with "qa". Possible answer choices: B, D

Sentence structure: The subject is at the beginning of the sentence followed by the verb (see
the third sentence.) Possible answer choices: B, D

Verb: to sleep = mopu. The correct answer is D.

Additional clues:

○ you = qe

○ Verb tense: past tense = "zo" (standalone word).

2.3.

"Do you go?" (you in singular) is expressed in the foreign language by:

(A) tolutqe zo

(B) tolummqe

(C) toluqa maq

(D) qeloqu zo

Answer B

Personal pronoun: you (singular) = qe. Possible answer choices: A, B

© 2017 edulink GmbH www.edu-link.de ALL RIGHTS RESERVED 201


Buyer: Jan Hiller (janhiller@gmx.net)
Transaction ID: 8BE25485KM511652L

Humanities, Cultural Studies and


Social Sciences Module

Verb tense: The sentence is in present tense. The word "zo", which indicates the past tense,
should not occur in this sentence. The correct answer is B.

Additional clues:

○ to go = tolumm.

2.4.

"I drive and you sleep" (you in singular) is expressed in the foreign language by:

(A) qalogut zo qemoput

(B) qelogut zö qamopu

(C) qelogut zö qamopu

(D) qalogut zö qemopu

Answer A

Personal pronoun & verb: I drive = qalogut. you sleep = qemopu (based on our findings in
the last 2 questions). Possible answer choices: A, D.

Verb tense: The sentence is in present tense. The word "zo", which indicates the past tense,
should not occur in this sentence. The correct answer is A.

Additional clues:

○ and = zö.

2.5.

moloztipe boto xoxiq = The architect draws rapidly.

lumetepi moloztipe xax = The painter calls the architect.

maleztipe boto xuxy = The intern writes rapidly.

lumeztipe maleztipe xüxlim = The painters annoy the intern.

"The architect calls the painter" is expressed in the foreign language by:

(A) lumetepi xüxlim moloztipe

© 2017 edulink GmbH www.edu-link.de ALL RIGHTS RESERVED 202


Buyer: Jan Hiller (janhiller@gmx.net)
Transaction ID: 8BE25485KM511652L

Humanities, Cultural Studies and


Social Sciences Module

(B) moloztepi lumetipe xüxlim

(C) moloztipe lumetepi xax

(D) lumetepi molozlipe xax

Answer C

Subject: the architect = moloztipe (See the repeating words in the first and the third
sentence.) Possible answer choices: A, C.

Sentence structure: The subject is found at the beginning of the sentence. (To find this out,
compare where "moloztipe" is in the first and the third sentence.) The correct answer is C.

Additional clues:

○ Verb: to call = xax. (We compare the second sentence with the answer choices and
look for repeptitions.)

○ Das Verb steht am Ende des Satzes.

○ Objekt: der Maler = lumetepi.

2.6.

notisu nadu jughane kok = I go tomorrow into the forest.

guvijur nadu jughani sai = I went yesterday to the see.

jassarasska nadu jughane jaf = I go today by the ocean.

guvijur nadu jughane kok = I go tomorrow to the see.

"I went yesterday into the forest" is expressed in the foreign language by:

(A) guvijur nadu jughane kok

(B) notisu nadu jughane jaf

(C) jassarasska nadu jughani sai

(D) notisu nadu jughani sai

Answer D

Verb & verb tense: jughane = I go. jughani = I went. Possible answer choices: C, D.

© 2017 edulink GmbH www.edu-link.de ALL RIGHTS RESERVED 203


Buyer: Jan Hiller (janhiller@gmx.net)
Transaction ID: 8BE25485KM511652L

Humanities, Cultural Studies and


Social Sciences Module

to the see = guvijur nadu. nadu is repeated in each sentence as the second word. It is likely
that nadu fulfils the function of a preposition (into, to, by).

When we review teh first sentence with this knowledge, we can conclude the following:
notisu = forest. notisu nadu = into the forest. The correct answer is D.

Additional clues:

○ sai = yesterday

○ kak = tomorrow

○ jaf = today

○ Sentence structure: Object  preposition  verb  time reference (yesterday,


today, tomorrow)

2.7.

qaramos = We wash

twouxram = They bring

qarauram = They washed

"We brought" is expressed in the foreign language by:

(A) twouxmos

(B) qararam

(C) twouxumos

(D) twouxram

Answer C

Verb:

○ We wash and They washed = Qaramos and Qarauram. Hence, “qara” must be the
root of the verb “wash”.

○ They bring and They washed = twouxram and twouxram.

© 2017 edulink GmbH www.edu-link.de ALL RIGHTS RESERVED 204


Buyer: Jan Hiller (janhiller@gmx.net)
Transaction ID: 8BE25485KM511652L

Humanities, Cultural Studies and


Social Sciences Module

Personal pronoun: Last 3 letters "ram" is the same. So, "ram" must be the word "they".

Tense: we wash and they bring = Qaramos and twouxram. There are no similar letters in
these words. So, there is no any particular letter for present tense in this foreign language.

We wash = Qaramos. Consequently: we = mos.

We brought = bring + a letter for past tense + We = twoux + u + mos = twouxumos. The
correct answer is C.

2.8.-2.11.

ba ?zuxöm lem = will he read?

kito bao = she writes

kito nep ba = he wrote

?kito lem bao = she will write

2.8.

"We will read" is expressed in the foreign language by:

(A) zuxöm baoy

(B) ?zuxöm lem baoy

(C) zuxöm ?lem baoy

(D) ?kito lem

Answer B

The second and fourth sentence differ only in the verb tense. Let’s start with analysing these
to decipher this language. We again look for common words/letters.

Verb: to write = kito. Answer D can be eliminated because kito should not be in the answer
choice. Additionally, we now know that the entence starts with the verb. Possible answer
choices: A, B, C.

Personal pronoun: she = bao. Subject comes after the verb.

© 2017 edulink GmbH www.edu-link.de ALL RIGHTS RESERVED 205


Buyer: Jan Hiller (janhiller@gmx.net)
Transaction ID: 8BE25485KM511652L

Humanities, Cultural Studies and


Social Sciences Module

Verb tense: future tense = the symbol "?" precedes the verb and "lem" follows the verb. The
answer is B.

Additional clues:

○ Sentence structure: Present tense: verb  subject.

○ Sentence structure: Future tense: "?" + Verb  lem  subject.

○ to read = zuxöm.

○ we = baoy

2.9.

"Did they write?" is expressed in the foreign language by:

(A) baoz kito nep

(B) baoz kito

(C) baoz zuxöm nep

(D) ?kito lem baz

Answer A

Verb: to write = kito. Possible answer choices: A, B, D

Verb tense: The sentence is in past tense. past tense = "nep" (compare second and third
sentence. The answer is A.

Additional clues:

○ they = baoz

○ Sentence structure in case of a question: The sentence starts with the subject:
Subject  (?) + Verb  Tense

© 2017 edulink GmbH www.edu-link.de ALL RIGHTS RESERVED 206


Buyer: Jan Hiller (janhiller@gmx.net)
Transaction ID: 8BE25485KM511652L

Humanities, Cultural Studies and


Social Sciences Module

2.10.

"He painted" is expressed in the foreign language by:

(A) kito ba

(B) sipo nep ba

(C) zuxöm nep ba

(D) kittom lem ba

Answer B

Verb tense: past tense = "nep". Possible answer choices: B, C

Verb: to read = zuxöm. We can eliminate answer C. The answer is B.

Additional clues:

○ to paint = sipo (We can only conclude this after determining that B is the answer.)

2.11.

"She reads and he will write" is expressed in the foreign language by:

(A) zuxöm bao toki ?kito lem ba

(B) zuxöm bao ?toki kito lem ba

(C) zuxöm ba toki ?kito ba

(D) kito bao toki lem bam

Answer A

Personal pronoun & Verb: she reads = zuxöm bao. Possible answer choices: A, B

Verb tense: future tense = the symbol "?" precedes the verb and "lem" follows the verb.This
means: he will write = ?kito lem ba. The answer is A.

Additional clues:

○ and = toki. (We can only conclude this after determining that A is the answer.)

© 2017 edulink GmbH www.edu-link.de ALL RIGHTS RESERVED 207


Buyer: Jan Hiller (janhiller@gmx.net)
Transaction ID: 8BE25485KM511652L

Humanities, Cultural Studies and


Social Sciences Module

2.12.

Desser ple = I am working

Elbisso gim = she is talking

ha og gi = he and she

"He is talking and working" is expressed in the foreign language by:

(A) ham Elbisso og Desser

(B) Elbisso og Desser ha

(C) Elbisso og Desser ham

(D) ha Elbisso og Desser

Answer C

Personal pronoun: she = gi. This means that letter "m" stands for "is". And the verb comes
before the pronoun. The possible answer choices are reduced to B and C.

Verb:

○ talking = Elbisso. The possible answer choices are still B and C.

○ working = Desser. The possible answer choices continue to be B and C.

Personal pronoun: He = ha.

He is = ham. The letter "m" is for "is".

And = og. C is the correct answer.

2.13.

Zopli lia odandig tausa = The policeman is driving the car

Ozagar domna tausaro = The boy is giving his car

Codimac lonatcon napenti = The doctor is checking the patient

napenti mendaco zopli lia = The patient is calling the policeman

© 2017 edulink GmbH www.edu-link.de ALL RIGHTS RESERVED 208


Buyer: Jan Hiller (janhiller@gmx.net)
Transaction ID: 8BE25485KM511652L

Humanities, Cultural Studies and


Social Sciences Module

"The doctor is calling his patient" is expressed in the foreign language by:

(A) Codimac mendaco napentiro

(B) Codimac mendaco napenti

(C) napenti mendaco codimac

(D) Ozagar mendaco tausaro

Answer A

If we analyse the third and fourth sentences, we can say the word order of the foreign
language is similar to the English language: Subject  Verb  Object

Subject: the doctor = Codimac. A and B could be correct.

Verb: is calling = mendaco. The possible answer choices continue to be A and B. We need
to figure out what the object is, i.e. what "his patient" is translated as. Let’s look at the
objects of first and second sentences.

Object:

○ the car = tausa

○ his car = tausaro (ro has been added to "tausa" to convert "the car" to "his car".)

Consequently, "his patient" must be "napentiro". A is correct.

2.14.

Tū borplokkē rumpt = The uncle is advising his son

Lossēz boxty borp = The son was painting beautifully

Tū mantūpthadam untēad = The professor is motivating his student

mantūpLokkēz lenkēm = The principal was advising the student

"The principal was motivating his son" is expressed in the foreign language by:

(A) Tū rumptthadamz lenkēm

(B) Tū borpthadamz lenkēm

© 2017 edulink GmbH www.edu-link.de ALL RIGHTS RESERVED 209


Buyer: Jan Hiller (janhiller@gmx.net)
Transaction ID: 8BE25485KM511652L

Humanities, Cultural Studies and


Social Sciences Module

(C) borpthadamz lenkēm

(D) Tū borpthadam lenkēm

Answer B

Sentence structure: Object  Verb  Subject (Based on the analysis of the first and second
sentences)

Possessive pronoun: his = Tū. Possible answer choices: A, B and D.

Subject: The principal = lenkēm. The possible answer choices continue to be A, B and D.

Object: son = borp. Possible answer choices (remaining): B and D

Verb:

○ Consider the verbs of the first and fourth sentences. Is advising = lokkē. Was
advising = lokkēz. "z" has been added at the end to make it past tense.

○ is motivating = thadam. According to third and fourth sentences, "mantūp" means


"the student".

○ was motivating = thadamz. The correct answer is B.

2.15.-2.17.

lomvelz roxalk nep = The cat is black.

lomnoly boking nep = The lawn is green.

lamvoxs peny komolx tos = The boy always plays soccer.

lemrasx peny minorz nep = The horse is always tired.

2.15.

"The boy is never tired" is expressed in the foreign language by:

(A) lemvoxs peny minorz nep

(B) lamvoxs nep tilute minorz

© 2017 edulink GmbH www.edu-link.de ALL RIGHTS RESERVED 210


Buyer: Jan Hiller (janhiller@gmx.net)
Transaction ID: 8BE25485KM511652L

Humanities, Cultural Studies and


Social Sciences Module

(C) lemrasx tilute minorz nep

(D) lamvoxs tilute minorz nep

Answer D

Verb: is = nep. Possible answer choices: A, B, C, D

Adjective: tired = minorz. Possible answer choices: A, B, C, D

Subject: the boy = lamvoxs. Possible answer choices: B, D

Sentence structure: The verb (like "nep") is always at the end of the sentence; consequently,
B cannot be correct. The correct answer is D.

Additional clues:

○ "peny" must correspond to "always".

2.16.

"The black horse plays on the lawn" is expressed in the foreign language by:

(A) roxalklamrasx lomnoly tos

(B) lemrasxroxalk lomnoly tos

(C) lamnoly lemrasxroxalk tos

(D) lemrasx roxalk nep lomnaly tos

Answer B

Verb:

○ to play = tos. Possible answer choices: A, B, C, D

○ „nep“ = is (see 2.14). „is“ as verb is not asked in this sentence and additionally verby
are at the end of the sentence in this language. The answer choice D can be
eliminated. Possible answer choices: A, B, C

Adjective: black = roxalk. Possible answer choices: A, B, C

Subject: the horse = lemrasx. Possible answer choices: B, C

© 2017 edulink GmbH www.edu-link.de ALL RIGHTS RESERVED 211


Buyer: Jan Hiller (janhiller@gmx.net)
Transaction ID: 8BE25485KM511652L

Humanities, Cultural Studies and


Social Sciences Module

"The lawn" = "lomnoly", which eliminates answer choice C. The correct answer must be B.

2.17.

"The cat and the boy are tired" is expressed in the foreign language by:

(A) neplo roxalk lomvelz lamvoxs minorz

(B) minorz leamvelz lomvoxs neplo

(C) lomvelz at lamvoxs minorz neplo

(D) lemrasx at lamvoxs minorz neplo

Answer C

Verb: are = neplo. Possible answer choices: A, B, C, D.

Adjective: tired = minorz. Possible answer choices: A, B, C, D.

Subject: the cat, the boy = lomvelz, lamvoxs. Possible answer choices: A, C.

The following words are in answer choice A (black = roxalk), but they are not in the question
sentence. The correct answer is C.

2.18.

lupolani mo = She is telling nothing.

polanihuwi tum? = Are they telling a lot?

kipola joi = I will tell everything.

huwiristyni sur = They are travelling everywhere.

"Will she travel a lot?" is expressed in the foreign language by:

(A) luristy tum?

(B) ristynilu tum?

© 2017 edulink GmbH www.edu-link.de ALL RIGHTS RESERVED 212


Buyer: Jan Hiller (janhiller@gmx.net)
Transaction ID: 8BE25485KM511652L

Humanities, Cultural Studies and


Social Sciences Module

(C) ristylu tum?

(D) ristylu joi?

Answer C

Verb:

○ telling = polani (based on the first and second sentences)

○ tell = pola (based on the third sentence)

○ travelling = ristyni (based on the second and fourth sentences). "huwi" means "they".
Consequently, “they are travelling” must mean "huwiristyni".

○ travel = risty. Possible answer choices: C and D.

Personal pronoun: she = lu (based on the first sentence). Possible answer choices
(remaining): C and D.

A lot = tum (based on the second sentence). The correct answer is C.

2.19.-2.22.

momle lomanozete rokoz = The father helps his son.

naqumi katoze talokoz = The mother greets the dog.

molde katozete sumolüm = The student hugs his dog.

ronume molde ranilüm = The professor likes the student.

2.19.

"The son greets his professor" is expressed in the foreign language by:

(A) rokoz talulüm ronume

(B) naqumi ronumeti talokoz

(C) lomanoze ronumete taloklüm

(D) lomanoze ronumete talokoz

© 2017 edulink GmbH www.edu-link.de ALL RIGHTS RESERVED 213


Buyer: Jan Hiller (janhiller@gmx.net)
Transaction ID: 8BE25485KM511652L

Humanities, Cultural Studies and


Social Sciences Module

Answer D

Verb: to greet = talokoz. Possible answer choices: B, D

Object: his professor = ronume –te (his / her). Remaining answer: D

Additional clues:

○ Subject: the son = lomanoze.

2.20.

"The mother likes her dog" is expressed in the foreign language by:

(A) naqumi katozete ranilüm

(B) katozete naqumite rokozlüm

(C) naqumiti katozete ranilüm

(D) talokoz naqumite katozeti

Answer A

Verb: to like = ranilüm. Possible answer choices: A, C

Object: her dog = katoze -te (his/her). The correct answer is A.

Additional clues:

○ Subject: the mother = naqumi.

2.21.

"The professor hugs the student and her son" is expressed in the foreign language by:

(A) moldexümoldi ronume sumokoz

(B) ronume moldexütakonete sumolüm

(C) lomanozexükatoze ronume sumolüm

© 2017 edulink GmbH www.edu-link.de ALL RIGHTS RESERVED 214


Buyer: Jan Hiller (janhiller@gmx.net)
Transaction ID: 8BE25485KM511652L

Humanities, Cultural Studies and


Social Sciences Module

(D) ronume moldexütakonete ronume

Answer B

Verb: to hug = sumolüm. Possible answer choices: B, C

Object: the student = molde. her son = takonete. Remaining answer B

Additional clues:

○ Subject: the professor = ronume.

○ Additional clues: and = xü.

2.22.

"The boy greets his dog" is expressed in the foreign language by:

(A) ramilüm katozete takone

(B) romme katoze talokoz

(C) takone katozete talokoz

(D) katozete talokoz takone

Answer C

Verb: to greet = talokoz. Possible answer choices: B, C, D.

Object: his dog = katoze -te (his/ her). Possible answer choices (remaining): C, D.

Subject: the boy = takone. The correct answer is C.

© 2017 edulink GmbH www.edu-link.de ALL RIGHTS RESERVED 215

You might also like